Criminal Law Final Exam Review - Multiple Choice Questions

Lakukan tugas rumah & ujian kamu dengan baik sekarang menggunakan Quizwiz!

Arden is angry with Riley and wants Riley dead. Arden decides to poison Riley later that evening. Before the evening comes, Arden is out driving around town. Unbeknownst to Arden, Riley is in a car directly in front of her. Arden's car slams into Riley's vehicle and Riley is pronounced dead on the scene. Arden did not intend to kill Riley at the time of this accident. Which of the following is most correct? a. Arden can be found guilty of purposely killing Riley b. Arden will be found not guilty of purposely killing Riley c. Because Arden had the intent to poison Riley, Arden can be found guilty of purposely killing Riley when Arden's car slammed into Riley's vehicle

b. Arden will be found not guilty of purposely killing Riley In order to have a crime, it is necessary that the actus reus, mens rea, and causation concur. In this case there is an actus reus (slamming into the car), a mens rea (the plan to poison Riley), and causation (but for Arden's car going into Riley, Riley would not be dead). The problem, however, is that each of the times for these events are not the same. Thus, lacking is the concurrence of the actus reus, mens rea, and causation. A is incorrect because Arden did not have a purposeful intent to kill Riley at the time of the automobile accident. B is incorrect because although Arden had the intent to poison Riley, he did not have the intent to kill Riley when the car slammed into him. The intent and act are therefore not at the same time.

Ariel is charged with violating 18 U.S.C. 5 1001, which provides in part that "whoever, in any matter within the jurisdiction of the executive, legislative, or judicial branch of the government . . . , knowingly and willfully . . . makes any materially false statements . . . shall be . . . imprisoned not more than 5 years . . ." Ariel admits to making a false statement to an FBI agent, but argues that he was unaware that making a false statement to an FBI agent was a crime. Which of the following is most correct? a. Ariel will be successful with this argument because there is no mens rea to commit this crime b. Ariel will be unsuccessful with this argument because Ariel violated the statute by making a false statement to the FBI agent c. Ariel will be successful with this argument because ignorance of the law is an excuse d. Ariel will be unsuccessful with this argument because ignorance of the law can never be an excuse e. Ariel will be successful with this argument because the statute is vague and ambiguous

b. Ariel will be unsuccessful with this argument because Ariel violated the statute by making a false statement to the FBI agent "The general rule that ignorance of the law or a mistake of law is no defense to criminal prosecution is deeply rooted in the American legal system." Cheek v. United States (see § 5.07(B)). A is incorrect because it is not a defense that Ariel does not know that it is illegal to make a false statement to the government. C is incorrect because ignorance of the law is not an excuse in almost all instances. So, this is not the most correct answer. E is incorrect because this statute is not vague and vagueness does not go to the defense that Ariel wants to present, a defense of not knowing the law. D is not correct because there are instances where ignorance of the law can be an excuse, such as when the statute uses the word willfully and it is a complicated tax statute or when the statute deprives an individual of due process

At 8:00PM, Ben walks in the front door of a very exclusive cash-only restaurant filled with wealthy diners. Ben pulled out a gun, aimed it at the ceiling, and yelled, "Alright, you all know what this is! Put the cash in the bag and nobody will get hurt!" The nervous diners all complied. Ben fled with over $5,000 in cash. Which of the following would be most correct? a. The most serious offense Ben has committed is multiple counts of larceny because he did not take the cash by force from any of the diners in the restaurant b. Ben has committed multiple counts of armed robbery c. The most serious offense Ben has committed is common law burglary because Ben entered the restaurant at night with the specific intent to commit a felony (larceny) d. Ben has committed multiple counts of robbery, but not robbery "while armed" because he never actually threatened anyone with the gun.

b. Ben has committed multiple counts of armed robbery Ben's act of wielding a gun and threatening bodily harm constitutes the requisite level of force needed for robbery. A gun does not have to be fired or pointed at the victim during a robbery in order for the robbery to be "while armed," or armed robbery. Although Ben had the intent to commit a crime when he entered the restaurant, he did not commit common law burglary because the restaurant is a commercial establishment (not a dwelling or residence) and the restaurant was open to the public when Ben entered (thus no "breaking" or trespassory entry).

Bob was running late and rushed out of the house to get to work. When Bob began to feel weak and lethargic after driving for 20 minutes, he realized that his blood-sugar level was dropping because he did not take his medication. Bob stopped at a convenience store and realized as he walked inside that he also left his wallet at home. Feeling weak and faint, Bob grabbed a cold bottle of orange juice, gulped it down, and slumped to the floor. As Bob slowly regained his strength and stood up, the store owner asked Bob to pay for the orange juice. Despite describing his medical emergency and promising to return to the store with payment, Bob was arrested. Which of the following would be most correct? a. Because Bob was still inside the store, the asportation requirement for common law larceny was not met b. Bob committed larceny by consuming the orange juice without paying c. Because Bob immediately stated his intention to pay, he did not act with the criminal intent to permanently deprive the owner of possession d. Bob's dire medical emergency negates a specific intent to permanently deprive the owner of the property

b. Bob committed larceny by consuming the orange juice without paying When Bob consumed the bottle of orange juice with full knowledge that he lacked the means to pay for it, the "taking," "asportation," and intent to permanently deprive requirements were met because the bottle of orange juice was permanently gone.

The XYZ Corporation is a defense contractor that manufactures and sells vans to the government. The contract with the government states that XYZ Corporation will supply the government with 300 vans. XYZ's Chief Executive Officer (CEO) Luke tells employees to only give the government 290 vans, but to charge them for 300 vans. CEO Luke has a clear and deliberate intent to defraud the government in order to increase the profits of the company and also the amount of money that he will receive from the company. CEO Luke laughingly asks, "Who will ever count the number of vans we give to them?" Which of the following is most correct? a. Only Luke can be prosecuted for this crime as corporations cannot face criminal liability b. Both XYZ corporations and Luke can be prosecuted for this crime c. Only XYZ Corporation can be prosecuted for this crime d. No one can be prosecuted as theft is never a crime e. Defrauding of only ten vans is an insubstantial amount for a prosecution against XYZ Corporation

b. Both XYZ corporations and Luke can be prosecuted for this crime Both the individual and corporation can be prosecuted (see § 4.01). If the crime committed by the individual is for the benefit of the company and there is the requisite intent, the agent's conduct can be imputed to the company.

Brooke agrees with Susan to sell narcotics and place the proceeds from the sale into a fictitious federal bank account. This is exactly what they do, although unbeknownst to them the sale is to a federal Drug Enforcement Agency (DEA) officer. Brooke and Susan receive three thousand dollars ($3,000) from the sale. Both Brooke and Susan are caught on tape agreeing to the sale of the narcotics to the DEA officer. Brooke and Susan are charged with the crime of conspiracy to sell illegal narcotics. Brooke and Susan argue that they cannot be charged with conspiracy to sell illegal narcotics because they disagreed how the money from the sale would be spent. Which of the following is most correct? a. Brooke and Susan cannot be convicted of conspiracy to sell illegal narcotics if they disagree on how the money would be spent from the sale of the drugs b. Brooke and Susan can be convicted of conspiracy to sell illegal narcotics even if they disagree on how the money would be spent from the sale of the drugs

b. Brooke and Susan can be convicted of conspiracy to sell illegal narcotics even if they disagree on how the money would be spent from the sale of the drugs In looking at a criminal offense of conspiracy to sell illegal narcotics, the government only has to prove the elements of the offense, which in this case would be the elements of an agreement to sell illegal narcotics since drug conspiracies do not require an overt act. A would not be correct because how the money would be used after the sale of the illegal narcotics is not an element of the offense.

True or False? It is proper for a court to give a jury insturction that states "the law presumes that a person intends the ordinary consequences of his or her voluntary acts."

False - In Sandstrom v. Montana, the Supreme Court held that an instruction to the jury that "the law presumes that a person intends the ordinary consequences of his voluntary acts" violated the defendant's due process right to have the government prove all the elements of the crime beyond a reasonable doubt. See § 5.03.

True or False? If only two individuals are involved in the conspiracy, and both individuals agree to commit the act, then prosecutors are required to charge both parties.

False - Prosecutors have the option of having unindicted co-conspirators. There is no requirement to charge all the parties to a conspiracy and it is common for the government to have unindicted coconspirators. True is incorrect because prosecutors often will not indict all of the parties to a conspiracy and allow for a co-conspirator to receive immunity or a lesser charge in return for their cooperation and testimony against a co-conspirator.

True or False? A defendant is convicted of violating a statute that makes it a crime to be intoxicated in a public place. The Supreme Court found this statute to be unconstitutional as a violation of the cruel and unusual punishment clause of the Constitution.

False - This is false because the Supreme Court in Powell v. Texas found this statute to be constitutional. This differs from the case of Robinson v. California where the Supreme Court held that a statute of being addicted to drugs was unconstitutional because it was criminalizing a status or condition.

True or False? If a prosecutor proves that the accused is the "but for" cause of the social harm, the prosecutor has met their burden of proof of the causation element of the offense

False - Typically meeting the "but for" cause is only the first step in proving causation. Prosecutors also need to demonstrate that the accused is the proximate cause (see § 6.02). True is not correct because often intervening causes can limit whether the causation element has been met.

True or False? When a statute is clear and unambiguous, courts should still use the rule of lenity in interpretating the statute

False - When statutes are clear, there is no need to employ the rule of lenity. The rule of lenity is only used when there are two possible constitutional interpretations of a statute. The rule of lenity instructs that when the statute is ambiguous, and there are two possible constitutional interpretations, it should be resolved in favor of the defendant.

True or False? It is a correct statment of law for a court to instruct a jury that one who is reckless in learning the truth can be willfully blind.

False - Willful blindness requires that "(1) the defendant must subjectively believe that there is a high probability that a fact exists and (2) the defendant must take deliberate actions to avoid learning of that fact." In Global-Tech Appliances, Inc. at al v. SEB S.A., the Court held that willful blindness "surpasses reckless and negligence."

True or False? The American Law Institute's Model Penal Code (MPC), drafted by the American Law Institute (ALI) under Herbert Weschler, has not been adopted in its entirety in any jurisdiction.

True - Although states will adopt various provisions of the MPC, no state has taken the entire work and adopted it as a whole.

True or False? In order to have criminal liability in the Model Penal Code, it is necessary that the acts be caused by the accused, even when the statute involves absolute liability.

True - Although the common law does not have a separate rule for causation involving strict liability offenses, the Model Penal Code does. Model Penal Code 2.03(4) provides that "[w]hen causing a particular result is a material element of an offense for which absolute liability is imposed by law, the element is not established unless the actual result is a probable consequence of the actor's conduct."

True or False? Courts have allowed proof of the knowledge needed for a conspiracy conviction to be demonstrated when the accused acted with concious avoidance.

True - Although the government has to prove the defendant knew that he or she was entering into a conspiratorial agreement, that knowledge can be through conscious avoidance. In United States v. Svoboda, the Second Circuit held that a conscious avoidance instruction permits a jury to infer knowledge when there is sufficient evidence that the defendant was aware of a high probability of a fact — the criminal object of the agreement — but consciously avoided confirming that fact. False is incorrect because courts have allowed knowledge to be through conscious avoidance.

True or False? Common law burglary is a specific intent crime

True - Common law burglary requires breaking and entering the dwelling house of another at nighttime with the intent to commit a felony therein. Because it requires both an intent to break and enter and an intent to commit a felony therein, it is considered a specific intent crime. False is not correct because common law burglary is a specific intent crime. (see § 5.01).

True or False? It is possible for join perpetrators of a crime to be both co-conspirators and accomplices.

True - Conspiracy and accomplice liability have similarities in that both have multiple players, and it is possible for both to be charged in some instances where there is both an agreement to commit the offense and an aiding and abetting by the parties in the commission of the crime. One can assist in the commission of a crime, however, but not know the broader agreement to the essential nature of the plan and therefore not be a conspirator. False is incorrect as there can be instances where the parties agreeing to commit an offense are also aiding and abetting in commission of the offense.

True or False? A state is not required to have a specific insanity defense if it allows a defendant to present mental health evidence to negate the mens rea of a crime and also to present this evidence at sentencing.

True - In Kahler v. Kansas, the Court found that Kansas's statute that allowed the defense to present evidence that "as a result of mental disease or defect, [the accused] lacked the culpable mental state required," coupled with permitting a defendant to offer whatever mental health evidence he deems relevant at sentencing, is sufficient to meet due process.

True or False? The Model Penal Code no longer includes a death penalty provision.

True - In October 2009, the American Law Institute withdrew the Model Penal Code provisions referencing the death penalty (MPC § 210.6). Although they withdrew the death penalty provisions in the MPC, they did not vote to abolish the death penalty.

True or False? Coby points a loaded gun at the head of Leslie and pulls the trigger, killing Leslie. A jury can find that Coby had the intent to kill Leslie by looking at the natural and probable consequences doctrine.

True - In reaching its conclusion, the jury may infer that a person intends the natural and probable consequences of his or her actions and a defendant's statements as to his or her intentions are not binding on the jury if the acts demonstrate a contrary intent. See § 5.03.

True or False? In a hate crime, the defendant's motive for committing the crime is an element that must be proven by the prosecution

True - In some states, the motive must be hatred of the group (e.g., hatred or animus toward a particular race). In other states, the motive could be a belief that a stereotypical characteristic of the group member makes him or her more vulnerable to the crime. Jurisdictions vary in which groups are included for a hate crime, but typically hate crimes statutes include animus or stereotyping based on race, ethnicity, religion, and sexual orientation.

True or False? Solicitation is considered an inchoate crime.

True - It is not necessary for the actual crime to be completed. More importantly, if the crime is a completed offense, then it is unlikely that solicitation will be used by the prosecutor. False is incorrect because if the crime were a completed crime, as opposed to an inchoate offense, there would be no need to charge solicitation as the prosecutor could then charge the actual substantive offense.

True or False? Justices may differ as to whether legislative floor debates should be used to interpret a criminal statute.

True - Some justices use a textual approach, limiting their interpretation solely to what is contained in the text of the statute. Others prefer to find the intent of the lawmakers when they drafted the statute and may use legislative history to discern this intent.

True or False? The United States has 5% of the world's population, but 25% of the world's inmates.

True - The United States has one of the highest incarceration rates in comparison to other countries. The mass incarceration rates in this country have been expressed as a concern, and there is a growing movement across the political spectrum to reduce the rate of incarceration

True or False? According to the Model Penal Code, a material element of an offense is "an element that does not relate exclusively to the statute of limitations, jurisdiction, venue, or to any other matter similarly unconnected with (i) the harm or evil, incident to conduct, sought to be prevented by the law defining the offense, or (ii) the existence of a justification or excuse for such conduct

True - The definition of a material element of an offense differs from the definition for an element of the offense. Elements of the offense are all the items that are proved at trial. Material elements are a category within the scope of all of the elements. This distinction is important when one looks at the burden of proof required, who may have that burden of proof, and the rules related to some defenses.

True or False? Both federal and state juries require unanimous jury decisions in criminal cases

True - The federal jury had required unanimous agreement in its criminal decisions. In 2020, the Supreme Court extended that rule for state juries.

True or False? In the 19th century, the prevailing view was that government inducement provided no defense to a criminal charge.

True - True is correct because in the 19th Century entrapment was not a viable defense. Few jurisdictions allowed for a defense premised on government manufactured crime. False is incorrect because, unlike other defenses, entrapment is a relatively new one.

Using the statement and the statutory provisions below, determine whether sexual assault laws were violated. "My name is Greer. My ex-boyfriend, Perry, and I broke up several months ago because he was physically abusive. Last night I woke up to find Perry in my bed kissing me on the back of my neck. I was terrified. I am a very small guy and Perry is twice my size. I asked him to leave, but he insisted that I submit to sexual penetration first. He did not hit me or threaten to physically harm me, but I know from past experience that he has an explosive temper. I was afraid of him, so I submitted. I never consented, but I did not fight back or offer any resistance." Perry does not dispute the facts reported by Greer, but claims the sex was consensual. Which statutory provision could be used to charge Perry with sexual assault? a. "It is sexual assault for a person to have sexual contact or sexual intercourse with another person and the other person, by words or overt actions, has not indicated a freely given agreement or consent to the sexual act or contact. Consent may not be established by the lack of verbal or physical resistance or submission by the victim if the person was subjected to threats, coercion, or the use of force" b. "It is sexual assault if a person has sexual contact or sexual intercourse with another person without express verbal consent and regardless of a prior consensual sexual relationship and causes a physical injury that requires hospitalization or mental impairment requiring psychiatric care" c. "It is sexual assault for a person to have sexual contact or sexual intercourse with another and use physical force, a threat of physical force, or a combination thereof that makes a person unable to physically repel the actor or produces in that person a reasonable fear of death, serious bodily injury or kidnapping" d. "It is sexual assault if a person has sexual contact or sexual intercourse with another person and used or threatened to use a weapon or physical strength sufficient to overcome, restrain, or injure the person or used threats of harm sufficient to coerce or compel submission by the victim"

a. "It is sexual assault for a person to have sexual contact or sexual intercourse with another person and the other person, by words or overt actions, has not indicated a freely given agreement or consent to the sexual act or contact. Consent may not be established by the lack of verbal or physical resistance or submission by the victim if the person was subjected to threats, coercion, or the use of force" There is no affirmative consent by Greer, and the statute provides that lack of physical resistance or the failure to fight back will not suffice to establish consent if there was coercion. Here, coercion could be established by the fact that Perry broke into Greer's apartment, got into Greer's bed, and had sexual contact with him (kissing his neck), all without Greer's knowledge or consent. B is incorrect because, while Greer was afraid of Perry because of past violence, Perry did not use or threaten to use force. C is incorrect because there are no facts to show that Greer suffered the level of physical or psychological harm specified by the statute. D is wrong because Greer states that he submitted to Perry's demands out of fear based on past experiences with Perry, not because Perry used a weapon, threats, or his considerable physical advantage over Greer to restrain or compel Greer

Which of the following is a correct statement? A motion for judgment of acquittal (or a directed verdict) in a criminal case is: a. A request to the court to dismiss some or all of the criminal charges before jury deliberation b. A motion made by the defense or the prosecution prior to jury deliberations c. A motion typically made by the government prior to jury deliberations d. None of the above

a. A request to the court to dismiss some or all of the criminal charges before jury deliberation B and C are wrong because the prosecution would not make a motion to the court to acquit the defendant on criminal charges initiated by the government.

Abe meets Kate at a party. Abe very much admires the necklace that Kate is wearing. He remembers that she said she would be going out of town for the weekend. On Saturday at 10 p.m., Abe, using burglary tools, breaks and enters into the home of Kate with the purpose of taking the necklace. He does not have permission from Kate to enter her home. The necklace, however, is not at home, as Kate took it with her on her out of town trip. Unbeknownst to Abe, while entering the home he triggered a silent alarm, and as he exits the home, the police are waiting to arrest him. Abe is charged with burglary in a jurisdiction that uses the common law. Which of the following is the highest possible crime that Abe can be convicted of? a. Abe can be convicted of burglarly b. Abe cannot be convicted of any crime because he did not complete the crime c. Abe can only be convicted of attempted burglary because he did not take the necklace d. B and C above e. Abe can be convicted of posession of burglarly tools

a. Abe can be convicted of burglarly The elements of common law burglary are all here—the breaking and entering of the dwelling house of another, at nighttime, with the intent to commit a felony therein. One does not need to complete the act. Having the intent to commit a felony is sufficient (see § 5.01).

Which of the following statements is not correct? a. All federal drug conspiracy statutes require an overt act b. All conspiracies require that the co-conspirators have an agreement c. The general conspiracy statute requires an overt act d. Agreements to conspire do not have to be in writing e. To charge a conspiracy it is necessary that you have two or more people.

a. All federal drug conspiracy statutes require an overt act Drug conspiracy statutes, such as 21 U.S.C. § 846, do not require an overt act and the agreement coupled with the intent is sufficient for prosecuting the crime. Thus, it is an incorrect statement to say that all federal drug conspiracy statutes require an overt act. C is a correct statement and therefore an incorrect answer here. The general conspiracy statute, 18 U.S.C. § 371, requires an overt act as an element of the crime. E is a correct statement, and therefore an incorrect answer, because an element of the crime of conspiracy is the requirement of two or more people reaching an agreement, although both parties do not have to be charged. B is a correct statement and therefore an incorrect answer because a key element of the crime of conspiracy is an agreement between the parties. D is a correct statement and therefore an incorrect answer because agreements to conspire do not have to be in writing. They can be a mere nod of the head.

April is homeless and receives public assistance benefits. April received a text stating that her public assistance benefits would be direct deposited in her account at noon. At 11:00AM, April's infant son is hungry and crying persistently. April goes to the QuickieMart to get some baby formula. As she has done twice before, April plans to beg the store manager to "loan" her the formula until noon when she can pay. When April gets in line to speak to the manager the checkout line is long and moving very slowly. April is desperate to feed her baby, so she decides to take the baby formula and come back to pay for the formula when her benefits are deposited. An off-duty police officer sees April leave without paying for the baby formula and stops her. Which of the following would be most correct? a. April committed common law larceny because she left the store without paying b. April has committed larceny only if the government can show that she did not intend to return and pay for the baby formula c. There is no larceny because April's intent was to temporarily--not permanently-deprive the storeowner of possession of the baby formula, and then return to the store and pay for the item d. April has not committed larceny if the police officer takes April back in the store and the store owner agrees to the temporary "loan" arrangement

a. April committed common law larceny because she left the store without paying When April departed the store with the baby formula she took and carried away the property of another with the intent to permanently deprive the rightful owner of that particular can of baby formula (which was to be immediately consumed by her baby). Her intention to pay for the product later without the store owner's agreement to the "loan" does not negate her criminal intent.

Ashley shoots Whitney in the leg, resulting in an ambulance being called and Whitney going to the hospital. While at the hospital, medical tests are performed on Whitney, and it is discovered that Whitney has brain cancer. Whitney dies from the brain cancer three days later. Which of the following is most correct? a. Ashley is not responsible for the death of Whitney because of an independent intervening cause b. Ashley is not responsible for the death of Whitney because of a dependent intervening cause c. Ashley is responsible for the death of Whitney because of an independent intervening cause d. Ashley is responsible for the death of Whitney because of a dependent intervening cause.

a. Ashley is not responsible for the death of Whitney because of an independent intervening cause An independent intervening cause is when the defendant's acts are so unrelated to the social harm that it would be unfair to hold the defendant criminally responsible for the result. It occurs when it is something that is not foreseeable (see § 6.02). Here, it is not foreseeable that being shot in the leg would result in a diagnosis for brain cancer. B is incorrect because it would be unfair to extend criminal liability for an unrelated act. C and D are incorrect because this does not involve a dependent intervening cause. A dependent intervening cause does not relieve one of liability. It is when the act of the defendant was reasonably foreseeable or intended or, in some locales, closely or sufficiently related to the defendant's conduct to make it fair to hold the defendant responsible for the result

Mike is pledging a fraternity. While the fraternity tried to force him to drink alcohol in excess, Mike outsmarted the frat brothers by removing the strong grain alcohol from the jug of "pledge juice" and replacing it with water. When Mike did not appear to be getting drunk fast enough, one fraternity brother gave Mike a slice of pizza that he secretly sprinkled with the illegal drug methamphetamine ("meth"). Mike consumed the pizza slice but felt no immediate effects of the meth. Mike was then tasked with going to the liquor store for more alcohol. Mike drove to the store without incident, but his vision suddenly became blurry and he became dizzy on the way back. Mike tried to pull over to the side of the road but he crashed into another car and severely injured the driver. Blood test showed Mike had meth, but no alcohol, in his system. Mike is charged with criminal recklessness. Which of the following would be most correct? a. Because Mike was impaired due to involuntary intoxication, he would be entitled to introduce evidence that he lacked substantial capacity to conform his conduct at the time of the collision b. The unwitting consumption of meth would constitute "intoxication" under the MPC, but evidence that Mike was given meth without his knowledge would be "immaterial" under the MPC because Mike is charged with criminal recklessness c. Because the MPC only applies to alcohol intoxication, the voluntary or involuntary consumption of the illegal drug methamphetamine would not constitute "intoxication" under the MPC d. None of the above

a. Because Mike was impaired due to involuntary intoxication, he would be entitled to introduce evidence that he lacked substantial capacity to conform his conduct at the time of the collision Mike's consumption of meth was not self-induced, and he would be able to introduce evidence of his involuntary impairment to show that he was not reckless. C is wrong because the definition of "intoxication" is broad enough to include alcohol and other substances capable of causing a "disturbance of mental or physical capacities." B is wrong because under the MPC the intoxication defense does not apply to criminal acts of recklessness, only when there is voluntary intoxication.

Benny received a text message from a friend that Quay, Benny's brother, participated in a bank robbery Friday night. Benny has not spoken to his brother, but heard from another friend that Quay was hiding out at the Motel 6 on the outskirts of town. On Monday, the police asked Benny if he knew Quay's whereabouts. Benny replied: "I have no personal direct knowledge of where my brother is and I don't get involved in his business." When the police later learned that Benny had information that his brother was at the Motel 6, Benny was arrested and charged as an accessory after the fact to the bank robbery. Which of the following is correct? a. Benny has not engaged in sufficient conduct to make him criminally liable as an accessory after the fact b. Because Benny did not participate in the planning or assist during the bank robbery, Benny cannot be an accessory after the fact c. Benny is an accessory after the fact because his evasive response to the police was designed to protect his brother Quay in the aftermath of the bank robbery d. Because there are no facts to establish that Benny shared in the proceeds of the bank robbery, Benny can be an accessory after the fact.

a. Benny has not engaged in sufficient conduct to make him criminally liable as an accessory after the fact In order for Benny to be an accessory after the fact, he would need to engage in conduct to assist Quay hide or escape. Here, Benny merely had second-hand information that he did not pass on to the police (and was not legally obligated to disclose to the police). C is incorrect because the government would have to prove, at a minimum, that Benny's vague answer to the police was a manifestation of his specific intent to help his brother escape or elude capture. B and D are incorrect because Benny could be an accessory after the fact even if he did not help with the robbery or benefit financially from the robbery.

Morgan and Cat both dislike Averill, a member of another gang. Unbeknownst to each other, they both decide to kill Averill. Morgan approaches Averill from the back door of Averill's home, while Cat approaches Averill from the front door. Both Morgan and Cat enter the home at the exact same time and both shoot at Averill at the exact same time. Morgan's bullet goes into Averill's head and Cat's bullet goes into Averill's heart. The coroner testifies that each bullet alone would have killed Averill. The coroner also says that the bullets killed Averill at the same time. Which of the following is most correct? a. Both Morgan and Cat can be convicted of killing Averill b. Neither Morgan nor Cat can be convicted of killing Averill since both shot at the same time c. The prosecutor has to choose which of the two individuals should be charged as two people cannot be convicted for killing one person d. Morgan and Cat can only be convicted of killing Averill if there is a state statute that allows for the prosecution of two individuals e. The only way that the prosecutor can proceed against both Morgan and Cat is if she uses the crime of conspiracy

a. Both Morgan and Cat can be convicted of killing Averill This is called concurrent sufficient causes. Under the doctrine of concurrent sufficient cause, if the shots by each party could have caused the death of the victim, then both can be held liable (see § 6.01).

Dakota places a bomb outside a condemned and vacated building as a protest to the building company's violations of union demands. Unbeknownst to Dakota, homeless individuals come into the building at night to sleep there. On the night of the bombing, three homeless individuals were in the building and two of them were killed as a result of the bomb. Which of the following is most correct? a. Dakota can be convicted for the death of the two individuals if it is reasonably foreseeable that homeless individuals would be sleeping in the building at night b. Dakota can only be convicted if the third individual dies c. Dakota cannot be convicted for the death of the two individuals even if it is reasonably foreseeable that homeless individuals would be sleeping in the building at night

a. Dakota can be convicted for the death of the two individuals if it is reasonably foreseeable that homeless individuals would be sleeping in the building at night This is an example of a dependent intervening cause. Dakota is not relieved of responsibility for a result that is caused by a dependent intervening cause if the dependent intervening cause was reasonably foreseeable. In this case it is reasonably foreseeable that homeless individuals might be sleeping in the building during the night. B is incorrect because, if it is reasonably foreseeable, then one can be held criminally liable for dependent intervening causes. C is incorrect because one can only be held criminally liable for a homicide for those who are actually killed.

Which of the following is an accurate statement about the criminal justice system? a. Even if the defense and procesution agree to plea bargain, the trial judge still conducts a hearing to review the case and make sure the defendant is making an informed decision to plead guilty b. Approximately 50% of all criminal cases are resolved with a guilty plea, and 50% of all criminal cases are resolved by a jury trial c. Most criminal appeals result in the reversal of the conviction and a new trial d. All of the above are true e. None of the above are true

a. Even if the defense and procesution agree to plea bargain, the trial judge still conducts a hearing to review the case and make sure the defendant is making an informed decision to plead guilty Because a guilty plea results in the waiver of several constitutional rights, the trial judge is required to make sure the defendant is making an informed decision to waive these rights

Chris decides to go the shopping mall after work. As soon as he entered the mall, a uniformed police officer approached him and tried to arrest him for stealing merchandise from one of the mall stores. Chris was not the person who stole merchandise. The officer had mistakenly identified the wrong person. The officer informed Chris that he was under arrest and instructed him to place his hands behind his back. Chris yanked away from the officer and refused to comply. The officer then tackled Chris to the ground and forcefully twisted his arm in order to place him in handcuffs. In response, Chris retrieved his licensed firearm from his ankle holster and shot the officer in the shoulder. Chris is charged with felony assault and maintains that he acted in self-defense. Which of the following is correct under the Arizona self-defense statute? a. Even though his arrest was unlawful, Chris did not have the right to use deadly force against the officer b. Regardless of whether the arrest was lawful, Chris had the right to use deadly force after the officer tackled him, choked him, and attempted to taser him c. Because the officer had no legal authority to arrest Chris, Chris had the right to use force to resist arrest and the right to use deadly force after he was tackled to the ground and assaulted by the officer d. Although Chris had the right to resist the unlawful arrest, he does not have a valid self-defense claim because he did not use a proportionate amount of force.

a. Even though his arrest was unlawful, Chris did not have the right to use deadly force against the officer The Arizona self-defense statute states that the use of physical force is not justified to resist an arrest by a peace officer "whether the arrest is lawful or unlawful, unless the physical force used by the peace officer exceeds that allowed by law." Here, the officer engaged in conduct to subdue Chris, but did not use excessive force. B, C and D are wrong because Chris could not use force to resist the arrest.

Jean and Claude have been married for 20 years. Jean is very religious. She has repeatedly told Claude that she does not approve of pornographic materials in their home. One day Jean saw a porn magazine and confronted Claude in the garage. Claude dismissed her concerns and defiantly stated that he would not stop ordering porn. Frustrated, Jean went into the house, retrieved a loaded gun, and returned to the garage. Jean then pointed the gun at Claude and said: "This is your last chance. I will not let you live a life of sin with all of this pornography." Claude ignored Jean and turned to walk away. Jean fired several shots at Claude's head and killed him. What is the most serious crime supported by the facts? a. First degree premeditated murder b. Voluntary manslaughter c. Second degree murder d. Involuntary manslaughter

a. First degree premeditated murder There is sufficient evidence of premeditation and deliberation and specific intent to kill, including Jean leaving the garage, returning with a loaded gun, making her "last chance" statement, and pointing the gun directly at Claude's head. These facts strongly support a finding that Jean's shooting of Claude was not a rash decision made in the heat of passion and without reflection. C is wrong because, due to the evidence of premeditation and deliberation (discussed above), second degree murder is not the most serious crime supported by the facts. B is wrong because it is unlikely that Claude's interest in porn would be legally adequate provocation (i.e., cause a reasonable person to lose self-control and respond violently). D is wrong because this was not an accidental shooting stemming from an act of gross negligence; Jean intended to shoot Claude.

Harry is an FBI agent. Harry and a few of his fellow agents went to the Mile High Dance Club in Denver. When one of his co-workers questioned whether Harry had hip-hop dance moves, Harry began doing the "stanky leg" dance. When Harry ended his dance with a back flip, his service revolver in his ankle holster fell to the floor, discharged, and killed a waitress. What is the most serious charge against Harry supported by the facts and what lesser-included offense instruction would Harry likely be entitled to? a. Harry could be charged with second degree murder and would likely be entitled to a jury instruction on involuntary manslaughter b. Harry could be charged with felony murder and would likely be entitled to a jury instruction on involuntary manslaughter c. Harry could be charged with second degree murder and would likely be entitled to a jury instruction on voluntary manslaughter d. Harry could be charged with first degree murder and would likely be entitled to a jury instruction on second degree murder

a. Harry could be charged with second degree murder and would likely be entitled to a jury instruction on involuntary manslaughter Dancing wildly and flipping upside down in a crowded place when you have a loaded gun strapped to your ankle provides sufficient facts to support a charge of second degree murder based on extreme recklessness or depraved heart murder, especially if the defendant is a law enforcement officer trained in the safety and handling of a firearm. The prosecution would argue that Harry knew and consciously disregarded the extreme risk of death to others in the crowd. Alternatively, the facts would also support a finding by the jury that Harry was criminally negligent or unreasonably unaware of the risk of death and was, therefore, guilty of only involuntary manslaughter. The defense could argue that Harry failed to appreciate the risk that his gun might fall out during a back flip and discharge. B, C, and D are wrong because each of these choices involves at least one intentional homicide as the greater or lesser offense, and there is no evidence that Harry formed a specific intent to kill the waitress.

Hollis recently lost his spouse. Hollis surfs the web looking at naked women engaged in sexual acts. All the sites visited by Hollis are legal sites and none involve child pornography. FBI Agent Jack notices that Hollis put a comment on a site with women engaged in sexual acts and FBI Agent Jack decides to see if Hollis might be interested in purchasing illegal child pornography. FBI Agent Jack contacts Hollis by email and says, "I have some incredible pictures of nude girls that you can view for just $5.00." Hollis, being extremely wealthy, says, "Sure, 5 bucks in nothing." Hollis orders the photos, not knowing that Jack is an FBI agent. This is the first time that Hollis has ever ordered child pornography. Thereafter, a magazine with illegal child pornography is mailed to Hollis. Upon receipt of the photos, the FBI raids the home of Hollis and arrests him for violating federal statute 18 U.S.C. § 2252, for knowingly receiving through the mail a "visual depiction [that] involves the use of minors engaging in sexually explicit conduct." The next day, following the search of Hollis home, he is arrested for violation of § 2252. Which of the following is most correct? a. Hollis is not guilty because as a matter of law he had no predisposition to commit child pornography b. Hollis is not guilty only if there is showing that there was outrageous government conduct c. Hollis is not guilty only if a jury reaches the conclusion that Hollis was not predisposed to commit child pornography d. Hollis is guilty of violating the statute and Hollis has no possible entrapment defense

a. Hollis is not guilty because as a matter of law he had no predisposition to commit child pornography According to Jacobson v. United States (see § 18.02), as a matter of law it is necessary for the government to have evidence that Hollis was predisposed, independent of the government acts and beyond a reasonable doubt, to violate the law by receiving child pornography through the mails. C is incorrect because with evidence lacking to support the government's case, it is a legal as opposed to factual question whether the accused had the predisposition to commit the act. B is incorrect as outrageous government conduct is a separate defense from entrapment, and it is not required that the defendant prove outrageous government conduct to be successful for an entrapment defense. D is incorrect because there is insufficient evidence that Hollis had the predisposition to commit this crime, and therefore a finding of guilty would be improper.

In which one of the following scenario(s) is it most correct to say that the doctrine of transferred intent usually applies? a. Hunter wrongfully fires a gun at Emerson, intending to kill Emerson, but the bullet instead strikes and kills unintended victim Carey, a bystander. Emerson is not injured b. Hunter wrongfully fires a gun at Emerson, intending to kill Emerson, but the bullet instead strikes and kills unintended victim Carey, a bystander. Emerson is not injured. In this jurisdiction, the statute states that prosecutors are not allowed to use transferred intent c. Hunter wrongfully fires a gun at Carey, thinking it is Emerson because Hunter is not wearing glasses that day. Hunter intends to kill the person shot, but finds out later that this person is Carey as opposed to Emerson, the person Hunter wanted to kill. Emerson is not on the scene and is not injured d. All of the above e. Hunter wrongfully fires a gun at Emerson and kills Emerson, the person Hunter intended to kill. The bullet, however, goes through Emerson and also kills Carey who is standing directly behind Emerson

a. Hunter wrongfully fires a gun at Emerson, intending to kill Emerson, but the bullet instead strikes and kills unintended victim Carey, a bystander. Emerson is not injured Transferred intent applies when a defendant intends to inflict a harm on a particular person and instead ends up inflicting the harm on a different person. This is the classic transferred intent scenario, or "bad shot" example, where the accused misses the intended victim and hits the person next to them (see § 5.05). B is incorrect because Hunter had the intent to kill the person killed. The issue was that because of poor sight, that person was not who Hunter thought it was. This is a misidentification as opposed to a misaim. One does not need to use transferred intent here because Hunter has the intent to kill the person aimed at and therefore can be found guilty for purposely killing them. C is incorrect because it is not the best answer. Transferred intent is not needed for the killing of Emerson as the intent to kill Emerson was present. Some courts are reluctant to allow for transferred intent when the original person for whom the crime was intended was in fact harmed. This does not preclude charging Hunter with a crime against Cary, the unintended victim; it just means that in determining the crime to be charged against Cary, the second individual, the doctrine of transferred intent will not be used. D is incorrect because if the legislature has clearly precluded use of the doctrine of transferred intent, then the issue will be resolved by the statute's wording, and transferred intent will not be permitted. E is incorrect because even if a jurisdiction allows for the use of transferred intent when both the victim and bystander are injured or killed, a statute precluding the use of transferred intent, as stated in D above, would not be allowed.

Patti is livid when she comes home and finds that her father has killed her dog for no reason whatsoever. In a state of extreme emotional disturbance, she pulls out a gun and kills her father. Which of the following is most correct? a. In a Model Penal Code jurisdiction, Patti would be able to argue that this should be reduced to manslaughter b. In a Model Penal Code jurisdiction, Patti would not be able to argue that this sluld be reduced to manslaughter

a. In a Model Penal Code jurisdiction, Patti would be able to argue that this should be reduced to manslaughter Some courts have developed a partial responsibility defense to reduce a murder charge to manslaughter. Some contend that the Model Penal Code has adopted this view in permitting murder to be reduced to manslaughter when there is a showing "of extreme mental or emotional disturbance for which there is reasonable explanation or excuse." In State v. Melendez, the court stated that "[t]o sustain his burden of establishing extreme emotional disturbance by a preponderance of the evidence, the defendant must persuade the trier of fact that: (1) the emotional disturbance is not a mental disease or defect that rises to the level of insanity as defined by the penal code; (2) the defendant was exposed to an extremely unusual and overwhelming state, that is not mere annoyance or unhappiness; and (3) the defendant had an extreme emotional reaction to it, as a result of which there was a loss of self-control, and reason was overborne by extreme intense feeling, such as passion, anger, distress, grief, excessive agitation or other similar emotions." One can argue that having her dog killed with no explanation resulted in a state of extreme emotional disturbance with a reasonable explanation. Although she would be able to argue this in a Model Penal Code jurisdiction, she might not be successful it the jury finds that her actions were not reasonable.

Gary, convicted in a Japanese court of having tried to smuggle several pistols, a rifle, and ammunition into Japan, was given a sentence of five years imprisonment by the court in Japan. Shortly thereafter he purchased a gun in the United States and was charged with violating a statute that makes it "unlawful for any person . .. who has been convicted in any court, of a crime punishable by imprisonment for a term exceeding one year . . . to . . . possess any fireman." Which of the following is most correct? a. In interpreting this statute, the court cannot use the conviction from Japan because "in any court" does not include a foreign court b. In interpreting this statute, the court can use the conviction from Japan because "in any court" includes all courts including those outside the United States

a. In interpreting this statute, the court cannot use the conviction from Japan because "in any court" does not include a foreign court The Supreme Court in Small v. United States held that foreign courts would not be included in the term "any court." The Court noted that although the word "any" typically demands a broad interpretation, in defining "any court" in the context of this case, a limited interpretation was proper. The Court stressed the importance of interpreting statutes domestically and not extraterritorially.

Barbara and Tiffany were best friends. They decided to join several other high school seniors in the longstanding tradition of jumping off the ten-story bridge on the last day of classes. When the group of teens arrived at the bridge about a dozen seniors immediately jumped off the bridge and landed safely in the river. When it was Barbara's turn, she stepped over the railing onto the ledge, but froze. Tiffany, who was waiting to make the jump, came up behind Barbara and pushed her off the ledge of the bridge. Barbara panicked and suffered fatal injuries when she landed on rocks under the bridge. What is the most serious charge supported by the facts? a. Involuntary manslaughter b. Second degree murder, extreme recklessness c. Second degree murder, specific intent to kill d. Felony murder

a. Involuntary manslaughter The government would be able to present sufficient evidence that Tiffany's actions amounted to criminal or gross negligence because she should have known that pushing an unsuspecting person off of a tall bridge creates a substantial risk of death or serious bodily harm. C. is wrong because there is no evidence of a specific intent to kill. D is incorrect because Tiffany has not committed an "inherently dangerous" or enumerated felony to support a felony murder charge. B is wrong because there are no facts to support a finding that Tiffany knew and consciously disregarded a known risk of death to Barbara, especially when several other teens had just jumped off of the bridge without incident and Tiffany was ready to jump herself.

Jack and Ezra agree to kill Toby, a member of a rival gang. Unbeknownst to Jack, Ezra is really an undercover police officer who is agreeing to commit this crime solely for the purpose of capturing and charging Jack. Jack and Ezra each have guns and go out to the location where they agreed to kill Toby. Upon arrival at this location, police cars surround the area and Ezra is charged with the crime of conspiracy to kill Toby. Which of the following is most correct? a. Jack can only be convicted of conspiracy to commit murder if the jurisdiction allows for unilateral conspiracies b. Jack cannot be convicted of conspiracy to commit murder because Toby was not killed c. Jack can only be convicted of conspiracy to commit murder if the jurisdiction allows for bilateral conspiracies.

a. Jack can only be convicted of conspiracy to commit murder if the jurisdiction allows for unilateral conspiracies Jurisdictions differ on when a feigning co-conspirator can provide the basis for a conspiracy. A is correct because in a jurisdiction that follows the unilateral approach, such as the Model Penal Code provides in § 5.03(1), there is no requirement that there be more than one participant in the conspiracy so long as "a person" "agrees with such other person or persons" to engage in criminal conduct. Because Jack and Ezra entered into an agreement to kill Toby, if this jurisdiction uses a unilateral theory of conspiracy, even though Ezra was an undercover officer, the conspiracy would be allowed. C is not correct because Ezra is not a real co-conspirator, but rather a government agent. Most jurisdictions follow the traditional bilateral approach reflected in the federal conspiracy statute, 18 U.S.C. § 371, which makes it a crime when "two or more persons conspire" to commit an offense. If the other purported participant is an undercover agent, or only feigns agreement, then there is no conspiracy if these are the only two participants. Thus, because Ezra was an undercover officer, there was no true agreement of both parties here. B is incorrect because conspiracy can be charged even when the underlying crime has not been committed. As an inchoate offense, the agreement can be sufficient.

Jayson suspected that his girlfriend Angi was cheating on him with his best friend, Steven. Jayson secretly set up a camera outside of Steven's house and saw Angl and Steven holding hands and kissing as they walked into Steven's house. Enraged, Jayson raced over to Steven's house with a hunting knife and used the spare key that Steven kept under the doormat. Jayson then charged at Steven and stabbed him multiple times. Steven, though severely injured, managed to take the knife apd lock Jayson in a closet. Which of the following would be most correct? a. Jayson has committed the crime of burglary b. Jayson did not commit the crime of burglary because he used Steven's spare key and did not "break and enter" Steven's home c. Jayson did not commit the crime of burglary because he did not steal any item of property in Steven's house d. Jayson has not committed a burglary because, in his fit of rage, he did not have the requisite criminal intent when he entered Steven's house.

a. Jayson has committed the crime of burglary When Jayson used the key to enter Steven's house without permission, he engaged in a trespassory entry that constitutes the elements of "breaking and entering" for the crime of burglary. Also, Jayson armed himself with a knife and attacked Steven immediately upon entry, strong evidence that Jayson had the intent to commit a felony (armed assault) when he entered the property.

Which of the following scenarios is NOT first degree murder? a. Keith comes home and discovers that Rex has entered his home and is sexually assaulting Keith's young daughter. Keith immediately attacks Rex to stop him, but Rex produces a gun and tries to kill Keith. Keith and Rex struggle over the gun, and Rex is killed when the gun discharges during the struggle b. Randy joins a violent street gang, the Creeps. As part of the initiation, Randy has to kill a member of the rival gang, the Freaks. Randy is brought to the location where the Creeps are holding a member of the Freaks and handed a loaded gun. Randy looks at the Freak, thinks about whether he should walk away, whether he might get caught, and whether being a Creep is worth it. Ultimately, Randy pulls out the gun and fatally shoots the Freak c. Jane's young son was killed when Ethan, a drunk driver, plowed into her car. Ethan managed to avoid criminal charges and get away with only a fine. Jane vowed to get revenge. She became consumed with rage and decided to kill Ethan. Jane went to the college and followed Ethan for several days. Jane then purchased a gun and shot Ethan d. All of the above would be first degree murder

a. Keith comes home and discovers that Rex has entered his home and is sexually assaulting Keith's young daughter. Keith immediately attacks Rex to stop him, but Rex produces a gun and tries to kill Keith. Keith and Rex struggle over the gun, and Rex is killed when the gun discharges during the struggle The facts describe a classic voluntary manslaughter scenario wherein the victim engages in sufficiently provocative behavior that causes the defendant to be enraged and respond violently. B and C both contain sufficient facts to support a finding that the defendant formed a specific intent to kill and killed after premeditation and deliberation.

Logan is charged with the crime of rape with intent to kill. In a common law jurisdiction, which of the following would be accurate? a. Logan has been charged with a specific intent crime b. Logan has been charged with a general intent crime c. Logan has been charged with a strict liability offense d. Common law jurisdiciton use mens rea of purposely, knowingly, recklessly, and negligently so this is neither a specific nor general intent crime e. None of the above

a. Logan has been charged with a specific intent crime Although rape is a general intent crime, when an intent to kill is added it becomes specific intent (see § 5.01). B is incorrect because adding the intent to kill moves the crime of rape, a general intent crime, to one that requires a specific intent. C is incorrect because rape is only strict liability when it is statutory rape. D is incorrect because common law does not require the use of the four levels of mens rea found in the Model Penal Code. E is incorrect because B, C, and D are not correct.

Marjorie is the sole parent of three-year-old Johnnie. Being busy at work lately, she fails to give Johnnie dinner several nights in a row. Johnnie ends up in a hospital for malnutrition. The local child services welfare department becomes aware of this situation and charges Marjorie with a child neglect crime Which of the following is most correct? a. Marjorie can be held criminally liable with the causation element satisfied as her omission to feed Johnnie caused him a harm b. Marjorie cannot be held criminally liable because parents have the sole control of whether to feed their children c. Marjorie can be held criminally liable and there is no need to demonstrate that Johnnie was harmed

a. Marjorie can be held criminally liable with the causation element satisfied as her omission to feed Johnnie caused him a harm An omission to act also requires that there be a correlation to the causation element. A parent has a duty to feed a child, and the social harm of the child being malnourished demonstrates how the omission to act causes the social harm. The doctrine of causation applies to omissions as it does to acts. If the nonaction satisfies "but for" and proximate causation, the person whose failure to act caused the harm may be criminally responsible for that harm.

Marty has a bike that he knows is stolen. Marty listed the bike for sale online for $100. Ken, who has no knowledge or reason to believe the bike is stolen, agrees to buy the bike and sends $100 to Marty. Marty delivers the bike to Ken. The following day the police locate the bike and arrest Ken. Which of the following would be most correct with respect to the transaction between Ken and Marty? a. Marty has committed common law larceny by trick, but Ken has not committed any crime by purchasing and possessing the bike b. Marty has committed the crime of larceny by trick for selling the stolen bike to Ken, and Ken has committed the lesser crime of receiving stolen property c. Marty has committed common law larceny for stealing Ken's money, and Ken has unwittingly committed the lesser crime of receiving stolen property d. Marty has committed the crime of false pretenses for inducing Ken to pay for the stolen bike, but Ken has not committed a property crime by purchasing and possessing the bike.

a. Marty has committed common law larceny by trick, but Ken has not committed any crime by purchasing and possessing the bike Marty's sale of a bike that he knows is stolen constitutes larceny by trick because he induced Ken to voluntarily surrender his property ($100) based on the misrepresentation that Marty had ownership and the right to sell the bike. Ken, however, has not committed a crime in buying or possessing the bike because he did not know the bike was stolen and, thus, lacked the criminal intent required for the crime of receiving stolen property.

Lennox was tried for the crime of stealing multiple pieces of clothes from a department store. At trial, the prosecutor presented several witnesses including the store manager, who identified each piece of clothing and the amount the store was selling those items for. The store manager testified that the total cost of the clothing taken from the store by Lennox was $120. After a trial by jury, Lennox was found guilty of theft over $100. In State A, theft over $100 is a felony punishable for up to five years in prison. The judge sentenced Lennox to 30 days in prison, noting that Lennox had no prior convictions for any crimes. Lennox is upset with this verdict and sentence and decides to hire a new lawyer to appeal the case. Lennox wants this new attorney to argue that the clothing was worth $ 90, and therefore Lennox should not have received a felony conviction. Which of the following is most correct? a. On appeal, the appellate court wil look at the evidence in a light most favorable to the prosecution b. On appeal, the defendant can present new evidence to sway the appellate court that the clothing is worth less than $100 c. On appeal, the appellate court will look at the evidence in a light most favorable to the defense d. On appeal, the defendant can testify as to why the lower court's jury verdict was improper

a. On appeal, the appellate court wil look at the evidence in a light most favorable to the prosecution On appeal, a court looks at the evidence in a light most favorable to the winning party, in this case the prosecution. It is difficult for the defendant-appellant to challenge the sufficiency of the evidence as the defense now has the burden of proof to establish that no reasonable juror could have found the accused guilty. Additionally, all inferences from circumstantial evidence are given to the winning party, the government. B is incorrect because the evidence is looked at in a light most favorable to the winning party, which will almost always be the government. One rare exception will be when the government is appealing a sentencing decision. C is incorrect because one does not get to present new evidence on appeal. If there is a claim of new evidence, the case is sent back to the trial court in order for that court to hear the new evidence. D is incorrect because there is no testimony from the defense on appeal. The appellate argument is made up of legal argument, as opposed to the presentation of evidence. Although it is possible to raise an issue of insufficient evidence to support the verdict, this argument is made by the presentation of legal arguments as opposed to the presentation of testimony.

Doug asks his friend Grey to go into John's house and pick up the TV in the living room and bring it over to Doug's house. Doug tells Grey that the TV is his and that he hurt his back last week at work and can't carry it home. Unbeknownst to Grey, the TV does not belong to Doug, and Doug does not have John's permission to remove it. Grey does as instructed and removes the TV. Which of the following is most correct? a. Only Doug can be convicted of a crime b. Both Grey and Doug can be convicted of a crime c. Only Grey can be convicted of a crime d. Neither Doug nor Grey can be convicted of a crime

a. Only Doug can be convicted of a crime Doug solicited Grey to commit a crime. Grey does not have a mens rea for the commission of the crime, so therefore cannot be charged as Grey is merely an innocent instrumentality. Absent a showing that Grey was willfully blind, Doug is the primary party in the commission of this crime. C is incorrect because Grey did not commit a crime since Grey had no mens rea for stealing the TV. B is incorrect because Grey did not commit a crime since Grey had no mens rea for stealing the TV. D is incorrect because Doug can be convicted for soliciting Grey. Doug can also be convicted for the crime itself as a principle party.

Penny is charged with common law burglary. Penny admits that she broke and entered Harper's home at nighttime but says that she entered the home because she truly believed that there was a fire inside. Penny admits that she knew Harper was out of town and not home but says that she wanted to make sure that there were no pets inside that needed saving. Penny triggered an alarm in the house, and the police arrived and arrested her. Harper was in fact out of town, and no one was in the house when Penny entered it. There was no smoke, and there was no fire. Which of the following is most correct for the burglary charge? a. Only if Penny acted in good faith can her argument that she was mistaken as to there being a fire in the house be successful b. It is not necessary for Penny to have acted in good faith as long as her mistake was not unreasonable c. Only if Penny acted in good faith and if her mistake was reasonable can she successfully argue that she was mistaken as to there being a fire in the house d. Penny can never argue that she was mistaken as to there being a fire in the house e. It is not necessary for Penny to have acted in good faith for her to be successful in her argument that she was mistaken that there was a fire in the house.

a. Only if Penny acted in good faith can her argument that she was mistaken as to there being a fire in the house be successful Because this is common law burglary, specific intent is needed. Mistake of fact requires good faith and can be used irrespective of whether it is reasonable or unreasonable (see § 5.07(A)). C is incorrect because mistake of fact for a specific intent crime only requires good faith, and the reasonableness or unreasonableness of the accused actions is not relevant. Both B and E are incorrect because good faith is a necessary component for mistake of fact. Further, D is incorrect because reasonableness is not necessary with a specific intent crime. D is incorrect because one can argue mistake of fact with specific intent crimes such as common law burglary.

According to the Model Penal Code, when the law provides that purposely is required to establish an element of an offense, such element can be established if a person acts which wich one of the following mens rea? a. Only when a person acts purposely b. When the person acts purposely or knowingly c. When the person acts purposely, knowingly, or recklessly d. When the person acts purposely, knowingly, recklessly, or negligently e. When the person acts knowingly, recklessly, or negligently

a. Only when a person acts purposely Purposely is the highest level of mens rea, so there are no substitutes for it under Model Penal Code § 2.02 (see § 5.02). B is incorrect because knowingly is a lower level of mens rea than purposely. C is incorrect because both knowingly and recklessly are lower levels of mens rea than purposely. D is incorrect because although purposely would be sufficient, the other mens rea terms would not be. E is incorrect because knowingly, recklessly, and negligently are lower levels of mens rea and therefore cannot be substitutes for purposely.

Blake takes a baseball bat and deliberately hits Pat with this bat, intending to injure him. Blake has no legitimate defense for committing this battery on Pat. Pat receives severe injuires resulting from Blake's actions and has a hospital bill of $50,000. Which of the following is most correct? a. Pat can proceed with a civil tort action against Blake to recover money for the hospital bill and other damages, and the State can proceed with a criminal action against Blake b. Pat cannot proceed with a civil tort action to recover damages if the State decided to proceed with a criminal action c. Pat gets to decide if the State can proceed with a criminal action against Blake d. The State cannot proceed with a criminal action if Pat decides to sue Blake in a tort action e. Neither the State nor Blake may proceed with any actions here

a. Pat can proceed with a civil tort action against Blake to recover money for the hospital bill and other damages, and the State can proceed with a criminal action against Blake A tort action for damages and injunctions has a different purpose than a criminal action, which is to remedy the harm to society and to punish the wrongdoer for their criminal actions (see § 2.02). Both actions may proceed simultaneously.

Prosecutors in a criminal case have the burden of proof. Which is the most correct as to the level of proof that prosecutors are required to prove in a criminal case? a. Proof beyond a reasonable doubt of the elements of the crime b. Proof by a preponderance of the evidence of the elements of the crime c. More than a mere scintilla of evidence of the elements of the crime d. Clear and convincing evidence of the elements of the crime e. Proof of probable cause of the elements of the crime

a. Proof beyond a reasonable doubt of the elements of the crime The Fifth and Fourteenth Amendments of the United States Constitution provide for due process, which requires proof beyond a reasonable doubt. States may disagree on which elements will require this level of proof, such as whether proof beyond a reasonable doubt will be necessitated for non-material elements such as venue and jurisdiction. But there is agreement that material elements of the actus reus, mens rea, and causation do require proof beyond a reasonable doubt. B is incorrect as a mere scintilla of evidence is a standard used in civil cases for a moving party to procedure. C is incorrect as this is a standard that is used for criminal matters such as obtaining a search warrant. D and E are incorrect as these are standards of review used in different types of civil matters.

Rob is a guard at the state prison. Ned is an inmate in the prison and a convicted sex offender. Rob and the other guards are well aware that many other inmates want to harm Ned. On Friday, when Ned entered the recreation room, several inmates started calling him names and taunting him. Ned defiantly responded by yelling and cursing at the other inmates. Moments later, the verbal dispute escalated. Rob saw four or five inmates rush over and aggressively confront Ned and then completely surround him. Rob monitored the situation from across the room, but did not try to physically intervene or break up the confrontation. Even after Rob saw the group of inmates pushing and shoving Ned, Rob only yelled for the inmates to "back off" and left the area for his dinner break. Moments later Ned was fatally stabbed by several inmates. The inmates are charged with murder, and Rob is charged with murder as an accomplice. Which of the following is correct under the Model Penal Code? a. Rob is an accomplice b. Rob is not an accomplice because there are no facts to support a finding that Rob knew in advance that the inmates were going to attack Ned c. Rob is not an accomplice because Rob did not participate in the attack on Ned d. Both B and C are correct

a. Rob is an accomplice As a prison guard, Rob had a duty to protect Ned and prevent the other inmates from attacking him. By not intervening and by leaving the recreation room as Ned was under attack, Rob's failure to act made him culpable as an accomplice to the fatal assault. Under the Model Penal Code, § 2.06(3)(a)(iii), a person is an accomplice if "having a legal duty to prevent the commission of the offense, [the person] fails to make proper effort so to do." B, C, and D are incorrect because neither prior knowledge, advance planning, or active participation is required when an individual has a legal duty to act.

Rose was walking in a grocery store when the store manager pointed at Rose while speaking with another store employee. Rose, thinking that she was being laughed at, took out a gun and shot the store manager, killing her. Rose is charged with homicide and Rose's attorney decides to present an insanity defense in a jurisdiction that uses the irresistible impulse test. She has a doctor who will testify that Rose suffers from a disease of the mind. Which of the following is most correct? a. Rose may be successful with this defense if she was unable to control her actions b. Rose may be successful with this defense if she suffered from a disease, the product of which caused her actions c. Rose may be successful with this defense if she did not know that her shooting the store manager was wrong d. Rose will not be successful because the irresistible impulse test is purely cognitive

a. Rose may be successful with this defense if she was unable to control her actions The essence of the irresistible impulse test is when there is an inability to control one's actions. C is not correct because not knowing the conduct is wrong would apply in a jurisdiction that used the M'Naghten test. B is incorrect because suffering from a disease, the product of which caused the actions is the Durham or product test, a test coming from New Hampshire. D is incorrect because the irresistible impulse test is not premised on a cognitive approach, but rather on a volitional approach.

A state statute exists in State A that prohibits gambling machines. The Attorney General in State A issues an opinion saying that video poker machines are not considered gambling machines. The opinion also states that video poker machines may legally be at truck stops within State A. Shelby, who owns several truck stops, puts video poker machines in each of these truck stops. Shelby also sends a political contribution to the Attorney General to make sure the Attorney General is re-elected. A local police officer arrests Shelby for violating the state statute prohibiting gambling machines based upon the video poker machines in the truck stops. In a Model Penal Code jurisdiction, which of the following is Shelby's best defense against this charge? a. The Attorney General is a proper law enforcement official to issue statements on what constitutes a gambling machine for purposes of state law, and that individuals who reasonably rely on those statements cannot be held criminally liable b. Shelby has no defense because one cannot rely on an Attorney General's statement when one makes a political contribution to the Attorney General c. The gambling statute is vague and therefore Shelby cannot be held criminally liable

a. The Attorney General is a proper law enforcement official to issue statements on what constitutes a gambling machine for purposes of state law, and that individuals who reasonably rely on those statements cannot be held criminally liable In § 2.04(3)(b) of Model Penal Code, it states that "[a] belief that conduct does not legally constitute an offense is a defense to a prosecution for that offense based upon such conduct when: ... (b) he acts in reasonable reliance upon an official statement of the law, afterward determined to be invalid or erroneous, contained in (i) a statute or other enactment; (ii) a judicial decision, opinion or judgment; (iii) an administrative order or grant of permission; or (iv) an official interpretation of the public officer or body charged by law with responsibility for the interpretation, administration or enforcement of the law defining the offense." An attorney general typically issues interpretations of law through his or her opinions, so this would provide a strong defense for Shelby (see § 5.07(C)). B is incorrect because it is not the best argument that Shelby could make. The statute may arguably be vague, but it is a stronger argument to say it has been interpreted by the Attorney General in accordance with the defendant's acts. C is incorrect because although possible corrupt conduct may be found applicable, that has no influence on the current charges against Shelby.

A federal criminal statute makes it a felony for a person to transport in interstate commerce any automobile, SUV, truck, van, bus, motorcycle, or other vehicle driven by a person that knows it to be stolen." Aubrey flies an airplane from Kentucky to Florida, knowing that the airplane is stolen from another person. Aubrey's best argument for dismissal of the charges is which of the following? a. The ejusdem generis principle excludes planes from the definition of the statute b. The ex post facto clause precludes this prosecution c. The bill of attainder precludes this prosecution d. The statute is void for vagueness

a. The ejusdem generis principle excludes planes from the definition of the statute When general language follows specific terms in a statute, the principle of ejusdem generis limits the general language to the specific terms. A plane would be outside the prior terms, which were all land vehicles

Which of the following is a true statement regarding criminal attempt under the Model Penal Code? a. The government must prove the defendant had a specific intent to commit the crime b. Attempt liability can be based on criminally negligent conduct under the Model Penal Code, as long as the defendant makes a "substantial step" towards completion of the crime c. Attempt is an inchoate crime. Thus there is no mens rea required, only a "substantial step" towards completion of the crime d. None of the above

a. The government must prove the defendant had a specific intent to commit the crime Attempt is a specific intent crime. Therefore, attempt liability under the Model Penal Code occurs only if the government can prove the defendant had a specific intent to commit the crime and made a substantial step towards completion of the crime. Therefore, B and C are incorrect because they do not accurately state the requisite mens rea for the crime of attempt under the Model Penal Code.

Which of the following is a TRUE statement regarding the power of the trial judge in a criminal case? a. The judge can dismiss a criminal charge if the prosecution has presented no evidence related to an element of the crime b. In a jury trial, if the defendant presents compelling evidence of an alibi, the trial judge can dismiss all charges, even if the prosecutor objects c. In a jury trial, if the judge believes the evidence presented by the government is not persuasive, the judge can dismiss the case d. Even if there is sufficient evidence to support the charges, the judge can dismiss the charges if they feel that circumstances surrounding the defendant's commisison of the crime warrant leniency e. All of the above are true

a. The judge can dismiss a criminal charge if the prosecution has presented no evidence related to an element of the crime The government has to prove each and every element of the crime beyond a reasonable doubt. If the prosecution has failed to present evidence regarding one or more elements of the crime, the trial judge has the power (and responsibility) to dismiss that charge

As Anne Greene walked to the elementary school to pick up her son, she was approached from behind by a man who, in a very demanding and intimidating voice, told her "do not turn around." The man then told Anne that when she picked up her son she needed to convince the school that she has permission to pick up Janie Smith, a little girl in her son's class, because Janie's parents just tested positive for COVID and cannot come to the school. The man said to Anne: "if you try to tell anyone at the school what I am telling you to do I will know because one of my people works at the school and will be watching you." Anne was terrified and felt like her life and the life of her child were in danger. Anne told the man that she would do as he demanded. After Anne convinced the school staff that she had permission to take Janie, Anne took Janie around the corner to a parked van as she was instructed to do. Janie is now being held for ransom because of her father's gambling debts. Anne is facing criminal charges for Janie's abduction. Which of the following would be most correct? a. There is ample evidence to support Anne's duress defense, including the manner that Anne was approached, the demands made, and the fear instilled in Anne b. Anne will not be able to support a duress defense on these facts because she cannot show that she exhausted all possible alternatives before participating in the criminal activity c. Anne does not have a valid duress defense because there was never an explicit or implicit threat to cause death or serious bodily injury to Anne or her son d. Anne will be able to introduce evidence of duress, but only if she can establish that she reasonably believed the man had a weapon when he approached her from behind and gave her orders

a. There is ample evidence to support Anne's duress defense, including the manner that Anne was approached, the demands made, and the fear instilled in Anne On these facts Anne will be able to show that "a person of reasonable firmness" would have been "unable to resist." An explicit threat is not required. There was clearly an implied threat of harm. Anne does not need to exhaust all available alternatives, nor is the use of a weapon required before she can assert a valid duress defense

Bart discovered on the nanny-cam in his apartment that his live-in girlfriend, Gina, was cheating on him with his best friend, Chris. Bart was extremely upset and raced home to confront Gina and Chris. When Bart arrived, it appeared that Gina and Chris had left. Bart saw Chris' leather biker jacket hanging on the back of the bedroom door and decided to burn it on the bed where Chris and Gina had been intimate. Bart cranked up his music and defiantly threw a match on the bed and jammed to the loud music while the jacket and the bed burned. Bart then ended his act of arson by throwing a bucket of water on the bed to extinguish the fire. Unbeknownst to Bart, Gina and Chris were hiding under the bed and became trapped when the fire started. With the loud music, Bart did not hear them screaming for help before they both died. What is the most serious crime against Bart that is supported by the facts? a. Two counts of first degree felony murder b. Two counts of second degree murder, specific intent to kill c. Two counts of second degree murder, extreme recklessness d. Two counts of first degree premeditated murder

a. Two counts of first degree felony murder During the commission of arson—an enumerated or "inherently dangerous" felony—Gina and Chris were unintentionally killed. C is wrong because if Bart did not know Chris and Gina were under the bed, there are insufficient facts to establish that Bart recklessly disregarded an extreme risk of their death. Likewise, B and D are wrong because there is no evidence that Bart acted with a specific intent to kill Gina and Chris, especially when he was unaware that they were under the bed.

Taylor is charged with burglary for breaking and entering the home of Haven at 2 a.m. The prosecution's evidence is that Taylor walked into Haven's house while sleepwalking. Which of the following is most correct? a. Under the Model Penal Code, Taylor is not guilty of burglary b. Under the Model Penal Code, Taylor is guilty of burglary

a. Under the Model Penal Code, Taylor is not guilty of burglary B is incorrect because under the Model Penal Code a bodily movement during unconsciousness or sleep is not considered an act (see § 4.01). So irrespective of whether the other elements of burglary can be met, the actus reus would not be sufficient.

Lorina is in the United States illegally. She works as a live-in nanny and maid for Ava Cantrell. Ava knows that Lorina has no family and no other resources. On Friday, Ava got into a heated argument with Karen, the neighbor across the street. The verbal sparring between Ava and Karen escalated when Ava shoved and slapped Karen. Although Karen was not injured, she called the police and Ava was arrested for misdemeanor assault. After Ava was released she ordered Lorina to set Karen's car on fire. Lorina repeatedly refused, but Ava threatened: "You will do it or I will call the immigration authority, and you will be deported." Ava knew that Lorina had narrowly escaped her home country after a violent gang threatened to kill her when she refused to serve as a drug mule. Lorina begged Ava, but Ava persisted. That night, Lorina threw a brick through the window of Karen's car and tossed a match inside. Lorina was caught and arrested. Which of the following statements by Lorina would most strongly support a duress defense? a. "If I refused Ava's demands, Ava would have kicked me out of her house that night. I would have been living on the streets with no means to support myself" b. "I knew that if Ava called immigration I would be deported and killed like my friend Luisa who was deported back to our home town a few months ago. Luisa survived only three days before she was fatally assaulted by a gang member" c. "Although I never faced physical harm from Ava in the past, after seeing Ava shove and slap Karen, I was afraid that Ava would slap me" d. All of the above would provide strong support for a claim of duress

b. "I knew that if Ava called immigration I would be deported and killed like my friend Luisa who was deported back to our home town a few months ago. Luisa survived only three days before she was fatally assaulted by a gang member" Given Lorina's knowledge of Luisa's fate upon deportation, Lorina had a reasonable fear that she would be deported and executed. A does not support duress because the statement involved mere economic distress and not a fear of bodily harm. C does not provide strong support for a claim of duress because Ava never threatened Lorina and the "force" she feared was a slap, not death or serious bodily harm. A slap is not the degree of force that would coerce a person to engage in criminal conduct.

Which of the following is the standard for criminal attempt under Model Penal Code? a. "Physical proximity" b. "Substantial step" c. "Dangerous proximity" d. "Indespensable element"

b. "Substantial step" A, C, and D each describe various common law approaches to defining the actus reus requirement for criminal attempt.

Jess, stranded on a country road, took out and used a flare gun from the trunk of their car in the hopes of being rescued. Shortly thereafter, Police Officer Jacobs came to the scene and charged Jess with violating a state statute that holds that "any person, not authorized by law, who carries concealed upon the person's self or within any vehicle used or occupied by the person or who is found armed with any dirk, dagger, blackjack, slug shot, billy club, metal knuckles, pistol, or other dangerous weapon shall be found guilty of a misdemeanor." Jess admitted to Officer Jacobs that they had carried the flare gun concealed in the trunk of their automobile prior to using it as an emergency signal. Which of the following is most correct? a. A flare gun designed as an emergency signal is not considered a dangerous weapon because of the principle of expression unis est exclusion alterius b. A flare gun designed as an emergency signal is not considered a dangerous weapon because of the principle of ejusdem generis c. A flare gun designed as an emergency signal is considered a dangerous weapon because of the principle of expressio unis est exclusion alterius d. A flare gun designed as an emergency signal is considered a dangerous weapon because of the principle of ejusdem generis

b. A flare gun designed as an emergency signal is not considered a dangerous weapon because of the principle of ejusdem generis Ejusdem generis applies when you have general language following specific terms in a statute. The general term—"other dangerous weapons"—would therefore be limited by the prior terms. Unlike those prior terms, which are dangerous weapons, a flare gun is used for emergency assistance and not as a weapon. A would be incorrect because the principle of expressio unis est exclusion alterius focuses on whether something is omitted from a list of similar items, allowing one to assume it should be there. Because a flare gun is not the same as the other items listed, this is not a deliberate omission of an item.

Stephanie has joint custody of her wayward teenage son Jeffrey, who stays with his father every other week. Stephanie came home late Friday night to find her home ransacked and her computer, television, and over $5,000 in jewelry taken. Soon after Stephanie arrived, someone emerged from the foyer closet, snatched her purse from her hand and fled. Stephanie's home security video showed that Carter, a friend of her son Jeffrey, broke an upstairs rear window to gain entry into the house. Moments later, Jeffrey used his key to enter the front door with Marcus. Carter and Marcus took the computer and television from the living room and Jeffrey took Stephanie's jewelry from her bedroom. When Stephanie arrived home, Jeffrey and Carter had fled, but Marcus hid and snatched Stephanie's purse. All three teens are in juvenile court in a jurisdiction where each child is only held legally responsible for the specific conduct they personally engage in, but not actions taken by their collaborators or accomplices. Which of the following would be most correct regarding the property crime(s) committed by each individual teen? a. Jeffrey, Marcus, and Carter each committed common law burglary because each teen entered Stephanie's home to commit felony larceny (theft of property over $1,000) b. Carter is the only person that committed burglary; Marcus is the only person that committed robbery. The actions of Jeffrey, Marcus and Carter constitute common law larceny c. Marcus and Carter each committed burglary, but Jeffrey's actions only constitute larceny d. Jeffrey, Marcus and Carter each committed larceny, but neither teen committed burglary or robbery

b. Carter is the only person that committed burglary; Marcus is the only person that committed robbery. The actions of Jeffrey, Marcus and Carter constitute common law larceny Jeffrey lives at the property with Stephanie and, therefore, did not burglarize the property "of another." Carter, however, committed a burglary because he broke and entered Stephanie's home when he climbed through the window to commit felony larceny. Marcus did not "break and enter" the house when he was invited in through the front door by Jeffrey, a resident of the property with a key. Marcus, however, did commit robbery when he used force to snatch the purse from Stephanie's hand. All three teens engaged in trespassory taking of property with an intent to permanently deprive Stephanie of possession.

Coby shoots Darcy in the arm, but Darcy does not die from this wound. Darcy is able to walk, and although in significant pain decides to walk the block to a nearby hospital. On the way to the hospital a bolt of lightning hits Darcy, and Darcy dies instantly. The coroner says that Darcy's injuries from being shot in the arm would not cause one to die. Which of the following is most correct? a. Coby is criminally liable for the homicide of Darcy since, but for Darcy being shot, Darcy would not be headed to the hospital to secure medical aid b. Coby is not criminally liable for the homicide of Darcy

b. Coby is not criminally liable for the homicide of Darcy If a person shoots someone but does not cause life-threatening injuries, the person is not criminally liable for murder if a different person or event later causes the victim's death. This is known as an obstructed cause, and it precludes the initial perpetrator from suffering consequences beyond the crime that he or she committed (see § 6.01).

Skylar is home watching television and drinking beer when there's a knock at the door. Skylar answers the door and invites two police officers into the home. They proceed to arrest Skylar and then forcibly take Skylar onto the nearby highway. Skylar is arrested for violating the following statute: "Any person who, while intoxicated or drunk, appears in a public place . . . and manifests a drunken condition by boisterous or indecent conduct or loud and profane discourse, shall be guilty of a criminal offense." On which of the following is Skylar most likely to succeed in arguing that Skylar did not violate this statute? a. The police officers violated Skylar's rights because they entered the home b. Drunkenness in a public place requires a voluntary act c. The police do not have anything showing that Skylar scored above a .08 on a breathalyzer test d. The police officers violated Skylar's rights because they took Skylar to a nearby highway

b. Drunkenness in a public place requires a voluntary act A voluntary act is necessary for the commission of a crime, unless the statute or act permits an omission to act.

Libby is at a bar with friends and decides to add some ecstasy to her drink. Her friend, Brenda, unaware that Libby's drink was spiked, asked to have a sip. Libby politely refused, telling her that the drink is much too strong for her. A few minutes later when Libby was dancing, Brenda gulped down Libby's drink and ordered Libby a fresh drink. Soon thereafter, Brenda walked home but entered the wrong apartment on the wrong floor. The door to the apartment was open and Brenda saw a man she believed was an intruder and screamed. When the man walked towards Brenda, she grabbed her gun from her purse and fatally wounded the man. Blood tests confirm that Brenda had consumed sufficient ecstasy for her to be impaired. Brenda is charged with second degree murder (specific intent to kill). Which of the following would be most correct? a. Because Brenda decided to consume Libby's drink, her intoxication was voluntary and she would not be able to introduce evidence of her intoxication to prove she could not form the specific intent to kill b. Due to Brenda's involuntary intoxication she can introduce evidence that she lacked "substantial capacity" to either appreciate the wrongfulness of her actions or confirm her conduct "to the requirements of law" c. Because Brenda was unaware of the drugs in Libby's drink, Brenda's intoxication was not self-induced. Thus, she would have immunity from prosecution for the murder charge d. None of the above

b. Due to Brenda's involuntary intoxication she can introduce evidence that she lacked "substantial capacity" to either appreciate the wrongfulness of her actions or confirm her conduct "to the requirements of law" Brenda did not voluntarily consume ecstasy because she was unaware that Libby's drink contained ecstasy. Thus, Brenda would be entitled to present the affirmative defense of involuntary intoxication.

Warren confronted Duran as Duran rolled his cart into the check-out line at Walmart. Warren said to Duran: "Hey, you saw my cart. You just cut the line!" Duran responded, "Look, get over it. You should have stayed with your cart." Warren called Duran a "punk" and pushed his shopping cart, causing it to bump Duran. Duran then pulled out his gun and fired one lethal round into Warren's chest. Duran is charged with murder and claims self-defense. Which of the following is correct statement regarding the trial judge's obligation to give the jury a self-defense jury instruction? a. Duran is entitled to a self-defense jury instruction if he was in lawful possession of the firearm b. Duran could not use deadly force in response to the non-deadly assault by Warren. Thus no jury instruction on self-defense should be given c. Because Duran escalated the confrontation with his words and conduct he is the first aggressor and would not be entitled to a self-defense jury instruction d. Duran can receive a self-defense jury instruction because he had no duty to retreat from Warren's attack under the Castle Doctrine.

b. Duran could not use deadly force in response to the non-deadly assault by Warren. Thus no jury instruction on self-defense should be given The deadly force Duran employed was not proportional to the bump he experienced by the shopping cart. While Duran may have had a subjective fear for his life, his fear was not reasonable because there are no facts to suggest that Warren threatened to harm Duran or was about to use greater force against Duran. Thus, the trial court would be justified in not giving the jury a self-defense instruction. A is wrong because the right to self-defense is not contingent on whether the person is in lawful possession of the weapon. The individual may face charges of weapons possession, but one can still claim self-defense for using the weapon. C is incorrect because Duran's verbal exchange did not make him the first aggressor; Warren became the first aggressor when he assaulted Duran by intentionally bumping him with the shopping cart. D is wrong because this fight took place in Walmart, and the Castle Doctrine is applicable to a person's right not to retreat in their home when faced with a threat to their safety.

When Ellie found out Jake was cheating on her she texted him that she was coming to his house to kill him. Jake called the police to report Ellie's threat. Ellie got her gun and began the eighty-mile trip to Jake's house. Ellie was stopped by the police for speeding after driving only ten miles. When the officer was in the process of writing Ellie a traffic citation he was informed of Jake's police report. The officer asked Ellie if she was planning to kill Jake. Ellie replied, "Yes; as soon as I get to his town, purchase ammunition, and get to his house he will be a dead man." Ellie is arrested for attempted murder. Which of the following is correct? a. Even before leaving her house, Ellie committed criminal attempt because she texted her death threat to kill Jake and armed herself with a gun b. Ellie did not commit criminal attempt c. Ellie committed criminal attempt as soon as she got in her car and began driving to Jake's house with a gun d. Ellie committed criminal attempt when she confirmed to the police officer that she planned to kill Jake.

b. Ellie did not commit criminal attempt While there is abundant evidence of Ellie's specific intent to kill Jake, Ellie was more than 70 miles away from Jake's house and was only in possession of an unloaded gun. She was not "at or near the place contemplated" for the commission of the crime (Jake's house) and she did not possess the means to commit the crime. A, B, and D are wrong because the text message and the statement to the police officer merely confirm that Ellie has formed the specific intent to kill. Ellie's conduct had not yet reached a substantial step.

Emery is engaged in fraudulent land deals. Emery contacts Mackenzie by telephone and proceeds to talk for 15 minutes about how Emery can make Mackenzie $5,000 overnight by just signing one form. Mackenzie, not knowing that Emery is committing fraudulent acts, listens intently and says to Emery, "I'll think it over." After the call, Mackenzie leaves to go out shopping. On the way to the store, Mackenzie's car goes through a railroad crossing and a train hits and Rills Mackenzie. There were no trains within the hour prior to the train hitting Mackenzie's car. The delay caused by Mackenzie leaving home put Mackenzie on the railroad tracks exactly when the train was passing. But for Mackenzie not being delayed by Emery's call, Mackenzie would have made it to the store and been alive. Which of the following is most correct? a. Emery cannot be held criminally liable for Mackenzie's death because Emery is not the actual cause of death b. Emery can be held criminally liable for Mackenzie's death because, "but for" Emery keeping Mackenzie on the telephone talking about a fraudulent deal, Mackenzie would be alive c. Emery cannot be held criminally liable for Mackenzie's death because Emery's involvement was de minimus

b. Emery can be held criminally liable for Mackenzie's death because, "but for" Emery keeping Mackenzie on the telephone talking about a fraudulent deal, Mackenzie would be alive The "but for" cause is only one step in the process of determining the causation element of the crime. It is also necessary to demonstrate that the accused was the proximate cause. Causes that are de minimus, or very minor or remote, do not satisfy the causation element (see § 6.02). A is incorrect because more than an actual or "but for" cause is needed to satisfy the causation element. C is incorrect because although Emery meets the "but for" test, the telephone is too remote from the train hitting Mackenzie to satisfy it being the proximate cause of the crime.

As Mary walked through the dimly-lit parking lot in her office building she notices a man walking behind her. As Mary walked faster, she sensed that the quickened his pace. When the man was about 10 feet behind her, Mary was in a panic. Mary turned and saw the man retrieve an item from his waistband area. Mary thought the man had a gun and intended to harm her. Mary was afraid for her life and pulled her licensed firearm out of her purse. When the man was approximately 5 feet behind her Mary spun around and fatally shot the man. As the man fell to the ground, he dropped the cell phone he had just retrieved from the clip on his belt. The man was unarmed. The man's car was parked next to Mary's car. Mary is charged with murder and claims self-defense. Which of the following is correct? a. Because Mary was subjectively afraid for her safety and the man was the first aggressor, Mary has a very strong self-defense claim b. Even though Mary was subjectively afraid, a jury could find that, based on the information known to Mary at the time, Mary's fear of death or serious bodily harm was not objectively reasonable c. A court is likely to rule that Mary is not entitled to a self-defense instruction based on the fact that she shot an unarmed man and Mary failed to try to retreat before resorting to deadly force, as required by law in most jurisdictions d. Mary was justified in shooting the man in self-defense under the Castle Doctrine

b. Even though Mary was subjectively afraid, a jury could find that, based on the information known to Mary at the time, Mary's fear of death or serious bodily harm was not objectively reasonable The most critical issue will be whether Mary subjectively and reasonably believed that the man was about to attack her. A jury could find that Mary rushed to judgment and simply did not have enough information to reasonably access the threat. A is wrong because the man did not attack or initiate a hostile interaction with Mary and was not, therefore, the first aggressor. C is wrong because most jurisdictions do not impose a duty to retreat. D is incorrect because the Castle Doctrine applies when one is confronted by hostility in their home, but not in a parking lot.

Lee and Randall decide to rob a street vendor. They approach the vendor from behind and, as planned, Randall holds a chloroform cloth over the vendor's mouth to render him temporarily unconscious, but not kill or seriously injure him. Lee then took the vendor's cash and fled. The vendor died as a result of the toxic combination of his prescription medication and the chloroform inhalation. What is the most serious crime supported by these facts? a. First degree premeditated murder b. First degree felony murder c. Second degree murder d. Involuntary manslaughter

b. First degree felony murder This was a death that occurred during the course of a robbery—an enumerated or "inherently dangerous" felony in most jurisdictions. As such, the most serious crime supported by these facts is first degree felony murder. A and C are wrong because there is no evidence of a specific intent to kill. D is wrong because it is not the most serious crime supported the facts.

Luke was informed that his teenage daughter Meghan skipped school. Luke suspected that Meghan had returned home. When Luke arrived at his house, he was surprised to see his friend Jack's truck parked in the driveway. Luke heard Jack's voice when he listened outside Meghan's bedroom door. Luke then quietly eased away from the door, retrieved his gun from the locked box in his bedroom, and loaded ammunition. Luke then kicked in the door of Meghan's bedroom. When Luke saw Jack and Meghan naked in Meghan's bed, he pointed his gun at Jack, ordered Meghan to get out of the bed, removed the safety on the gun, and said: "I may go to hell for this, but you are going to hell right now." Luke fired three shots at Jack's chest and killed him. What is the most serious charge supported by these facts? a. Voluntary manslaughter b. First degree premeditated murder c. Felony murder d. Second degree murder

b. First degree premeditated murder There are sufficient facts to find that Luke made a cool, calm, and calculated decision to kill Jack before the fatal encounter. Specifically, Luke left to retrieve a weapon, loaded his gun, and returned to confront Jack. Also, Luke's statement, "I may go to hell for this," is indicative of the fact that he deliberated in advance on the possible consequences for himself if he killed Jack. Likewise, Luke's words just before shooting ("you are going to hell") evince a specific intent to kill. A is incorrect because, although Luke's sudden discovery of Jack and Meghan's tryst likely provoked Luke to act in the heat of passion, voluntary manslaughter is not the most serious charge supported by these facts, and voluntary manslaughter charges are undercut by Luke's calm and calculated actions before confronting Jack. C is incorrect because Luke has not committed an "inherently dangerous" or enumerated felony. D is incorrect because, although there are facts to support a charge of second degree murder, second degree murder is not the most serious charge supported by these facts.

Angel and Frank agree to burglarize a home later that night. They decide that they will meet at 10:00 PM at the home to be burglarized and that Angel and Frank will enter the home using burglary tools. After the conversation between Angel and Frank, Frank goes home to the family and realizes that this is not a smart move and decides not to show up at 10 PM. Instead, Frank stays home and watches a movie with the kids. Which of the following statements is most correct in a Model Penal Code jurisdiction? a. Frank cannot be charged with conspiracy to commit burglary even if Angel shows up at the home at 10 PM and burglarizes the home b. Frank has an affirmative defense if charged with a conspiracy to commit burglary if Frank thwarts the success of the conspiracy and decides to completely and voluntarily not commit the burglary c. Frank has an affirmative defense if charged with a conspiracy to commit burglary because Frank did not show up at 10 P.M. to burglarize the home

b. Frank has an affirmative defense if charged with a conspiracy to commit burglary if Frank thwarts the success of the conspiracy and decides to completely and voluntarily not commit the burglary Section 5.03 of the Model Penal Code allows for an affirmative defense of renunciation of a conspiracy if "the actor, after conspiring to commit a crime, thwarted the success of the conspiracy, under circumstances manifesting a complete and voluntary renunciation of his criminal purpose." Although it is questionable whether Frank has thwarted the success of the conspiracy here, if Frank has done something to thwart its success, Frank would have an affirmative defense available. A is incorrect, because if Angel shows up, then there has been an overt act in furtherance of the conspiracy and Frank can be held criminally liable. C is incorrect because merely staying away from the scene of the crime is not enough to escape criminal liability.

Garnet walks into a room and deliberately and intentionally shoots and kills Casey. It is clear and there is no doubt that Casey died from Garnet's bullet. Rory walks into the same room an hour later. Casey's dead body still remains on the floor. Rory pulls out a gun and, not realizing that Casey is already dead, deliberately and intentionally shoots Casey's dead body that remains on the floor. Which of the following is most correct? a. Only Garnet can be convicted of murder for killing Casey, and Rory cannot be convicted of any crime. b. Garnet can be convicted of murder, and Rory can only be convicted of attempted murder for shooting Casey c. Both Garnet and Rory can be convicted of murder for killing Casey

b. Garnet can be convicted of murder, and Rory can only be convicted of attempted murder for shooting Casey If the individual is already deceased, then the second person shooting the dead body did not cause the social harm of the killing and cannot be convicted of that crime. Rory, however, did attempt to kill Casey, so Rory can be convicted of attempted murder. A is incorrect because Rory's conduct is punishable. B is incorrect because Rory cannot be convicted of killing an already dead person.

Presley is accused of killing Ricki and is charged with the crime of murder. The police have obtained the following evidence: I. A gun that is registered to Presley left at the scene of the crime II A fingerprint on the gun left at the scene of the crime that matches Presley's fingerprints III. A statement from witness Billie who says that she observed Presley shoot Ricki and that Presley stated at the scene of the crime, "[that's what you get for messing with me" IV. A statement by Presley to a friend after the killing saying, "I killed Ricki, and I ain't sorry I did it" Which of the above are considered circumstantial evidence, as opposed to direct evidence, that demonstrates that Presley had the intent to kill Ricki? a. I b. I and II c. I, II, and III d. All of the above e. None of the above

b. I and II Circumstantial evidence can be anything that allows a reasonable juror to infer a fact that is relevant to the prosecution's case. In this case one is looking at evidence that goes to whether the accused had the mens rea to commit the crime. In contrast, direct evidence generally consists of physical evidence, eyewitness statements, and statements by the defendant that provide direct testimony of the element of the crime, which in this case is the defendant's intent. Both I and II provide evidence that leads to Presley as the one who committed this crime. But neither provides direct evidence of the intent to kill Rikki (see § 5.03). A is incorrect because it omits II, which is a fingerprint of the defendant. This fingerprint does not prove an intent, but it does provide circumstantial evidence from which one can infer intent. C is incorrect because a statement from an eyewitness at the scene of the crime provides direct evidence that Presley had the intent to commit the offense. D is incorrect because a statement from Presley is direct evidence of an intent to commit this crime. E is incorrect because evidence at the scene of the crime that does not directly provide intent of the commission of the crime is circumstantial evidence upon which intent can be inferred.

The police legally enter Merrill's home and search for drugs in a dresser in Merrill's bedroom. They find an illegal controlled substance and charge Merrill with possession of a controlled substance. Which of the following would be Merrill's best arguments against this charge? a. Since he was not in his bedroom at the time the police found the controlled substance, he should be found not guilty because there was no proof of his dominion or control of the room b. If he had no actual knowledge of the existence of the controlled substance in the room and had not placed it there, he should be found not guilty c. Since there is no evidence showing that he intended to sell the drugs, he has not committed any act in violation of the possession statute d. Since this was his bedroom, he should be found guilty of possessing the controlled substance

b. If he had no actual knowledge of the existence of the controlled substance in the room and had not placed it there, he should be found not guilty Most possession statutes do require that one knows of the existence of the illegal substance. A is incorrect because it does not make a difference if he is present when the search occurs to determine whether he had dominion or control. C is incorrect because a possession charge does not require selling the controlled substance. D is incorrect because mere ownership or living in the room does not equate with knowing of the existence of the controlled substance, which is required for possession.

Which of the following facts would be most critical in determining whether a defendant will be charged with first degree premeditated murder versus second degree murder? a. The age of the victim/decedent at the time of death b. If the defendant disclosed her plan to commit the murder in a text message sent the day before c. Whether the defendant used a gun, as opposed to a knife or other weapon, to cause the death d. Whether the defendant has previously committed a murder

b. If the defendant disclosed her plan to commit the murder in a text message sent the day before The text message provides facts to establish that the defendant thought about and planned the murder in advance, which would help prove the elements of premeditation and deliberation needed for a first degree murder charge. While the age of the decedent and the use of a weapon (A and C) may be important factors in a specific case, such factors generally do not dictate whether the murder is first degree or second degree. Likewise, though a defendant's prior murder conviction (D) may be relevant in a specific case, the fact that a defendant has previously committed an unrelated murder will not be dispositive in determining whether the new crime is a first degree or second degree murder.

In the majority of jurisdictions, which of the following statements is correct? a. If the defendant asserts that she acted in self-defense, the defense has the burden to prove that the defendant acted in self-defense b. If the defendant first presents evidence to establish a factual basis for a self-defense claim, the government must then prove that the defendant did not act in self-defense c. If the defendant asserts a claim of self-defense, the defense does not have to present any evidence and the government must prove that the defendant did not act in self-defense d. If the defendant fails to prove that he acted in self-defense, the trial court can refuse to give a self-defense jury instruction.

b. If the defendant first presents evidence to establish a factual basis for a self-defense claim, the government must then prove that the defendant did not act in self-defense The government has the burden of proving that the defendant did not act in self-defense, but the defendant must present enough evidence to establish a factual predicate for each element of self-defense. If the defendant fails to meet the minimal factual foundation to support a self-defense claim, the trial judge can rule that the defendant is not entitled to a self-defense jury instruction. A and D are wrong because the defense is not required to prove the defendant acted in self-defense. C is wrong because the defendant is required to present some evidence before a jury instruction is warranted.

Bart, age 8, went into his father's room in the middle of the night and said he was hungry. His father told him to go back to bed. Later, his father woke to find Bart lying on the floor panting, speaking "gibberish," shaking uncontrollably, and hot with fever. The father, unaware that Bart had ingested some of the methamphetamine (meth) in a duffle bag under the father's bed, tried to calm Bart by rocking him and applying cold compresses. Several hours later, when Bart continued to exhibit symptoms of distress, his father summoned a neighbor for advice. His neighbor advised him to call 911. Although the father immediately rushed Bart to the hospital, it was too late to render the medical care that could have prevented Bart from dying of an overdose. What is the most serious criminal charge supported by these facts? a. First degree premeditated murder b. Involuntary manslaughter c. Second degree murder, intent to kill d. Voluntary manslaughter

b. Involuntary manslaughter Even though the father did not know the child ingested meth, several hours delay in seeking medical treatment for a child exhibiting these extreme symptoms of distress is a gross deviation from a reasonable standard of care. A, C, and D are all wrong because there is no evidence that the father had a specific intent to kill Bart or withheld medical treatment because he wanted Bart to die.

Maria and Perez were aspiring TikTok stars. They made videos of their various exploits and posted them online. They hoped that one of their videos would go viral and launch them into superstardom. Perez proposed putting a book on his chest and having Maria fire a gun at the book. Perez believed the book would stop the bullet. Maria was firmly opposed to the idea until Perez stood a short distance away and fired a bullet into a book propped against a tree. Maria saw that the bullet lodged in the pages of the book and did not puncture the back cover. After days of prodding, Perez finally convinced Maria. With family and friends watching and filming, Perez held a book on his chest. Maria stood several feet away and fired the gun at the book. This tire, however, the bullet went through the book and into Perez's chest. Perez died instantly. What is the most serious charge supported by these facts? a. First degree premeditated murder b. Involuntary manslaughter c. Second degree murder, extreme recklessness d. Voluntary manslaughter

b. Involuntary manslaughter The government would be able to present sufficient evidence that Maria's conduct in firing the gun was criminal or gross negligence because her conduct was a gross deviation from a reasonable standard of care and she was unreasonably unaware of the risk of death posed by shooting directly at Perez and expecting a book to stop a bullet. A is wrong because there is no evidence that Maria had a specific intent to kill Perez or that she premeditated and deliberated killing him prior to the fatal encounter. C is wrong because there are insufficient facts to establish that Maria knew of the risk of death and recklessly disregarded that risk. Such a conclusion is undercut by the fact that Maria and Perez first practiced the stunt under similar conditions. D is wrong because there is no evidence that Perez engaged in any conduct that provoked Maria or that Maria was enraged at the time she fired the shot.

Which of the following statements is NOT true? a. Second degree murder is usually a lesser-included offense of first degree murder b. Involuntary manslaughter is a lesser-included offense of voluntary manslaughter c. Voluntary manslaughter can be a lesser-included offense of second degree murder d. Involuntary manslaughter can be a lesser-included offense of second degree murder

b. Involuntary manslaughter is a lesser-included offense of voluntary manslaughter Voluntary manslaughter is a type of intentional, sudden killing based on adequate provocation. Involuntary manslaughter is an accidental or unintentional killing resulting from criminal negligence. The elements for the two crimes are completely different.

Same facts as Question 20 above. Which of the following new facts would be critical to the government's case in proving that John is guilty of first degree felony murder in the death of Kerri? a. Two weeks before going to Kerri's house, John learned that Kerri was HIV positive and told a co-worker that he planned to kill Kerri b. John broke into Kerri's house on the night of her death and brought the knife he used to fatally stab Kerri c. When John arrived at Kerri's house he brought duct tape and a large tarp to dispose of Kerri's body after killing her d. John was seen waiting in his car outside of Kerri's house for two hours before Kerri came home

b. John broke into Kerri's house on the night of her death and brought the knife he used to fatally stab Kerri First degree felony murder requires that the death occur during the commission of one of the enumerated or "inherently dangerous" felonies. Here, there are sufficient facts to support a finding that John killed Kerri during the commission of a burglary—breaking and entering Kerri's home with the intent to commit a felony offense (murder). A, C, and D are all important pieces of information that would support a finding that John committed first degree premeditated murder, but these facts do not help establish a felony murder charge.

John is standing on a street corner in a city with a statute that makes it a crime to be addicted to the use of narcotics. John has been a heroin user for years and has scab marks and discoloration on the inside of his arm. John is arrested by the local police and charged with the crime of being addicted to the use of narcotics. At the time of his arrest, he did not have any drugs on him. Which of the following is INCORRECT? a. It is unconstitutional to convict John for being addicted to the use of narcotics b. John can be convicted for being addicted to the use of narcotics if he had used narcotics during the past month c. It is unconstitutional to convict John because he is not currently using the narcotics d. John could constitutionally be convicted of a narcotics offense other than being addicted to the use of narcotics if he had possession of illegal drugs on him

b. John can be convicted for being addicted to the use of narcotics if he had used narcotics during the past month B is incorrect because, as a status crime, it would not make a difference whether he had used narcotics in the past if he is being charged with the status of being addicted to the drugs. A is a correct statement of the law in that being addicted to the use of narcotics is a status crime and therefore there is no punishable act. C is also correct because of this being a status crime. D is likewise correct, because, although it would be a different crime than stated in the question, one can be convicted for possession of illegal drugs, and this is not considered a status crime

Cedar is charged with the crime of burglary. Cedar asks counsel to cal indigo as a witness in the trial. Cedar says that witness Indigo will testify that they were together on the evening of the burglary and that they were watching television at Indigo's home. Cedar's attorney, Jordan, finds out that indigo not only does not have a television, but that Cedar was not with Indigo on the evening of the burglary. Cedar finally admits to Jordan that testimony that would be provided by Indigo would be false. Cedar believes that the jury will believe Indigo's testimony and acquit Cedar. Cedar implores Attorney Jordan to therefore call Indigo as witness. a. Jordan should call Indigo as a witness because a defendant has a right to present a defense b. Jordan should not call Indigo because the witness will lie on the witness stand

b. Jordan should not call Indigo because the witness will lie on the witness stand Although a defendant has a right to present a defense, it is improper for an attorney to call a witness that will present false testimony. Although there may be some dispute as to whether counsel is obligated to call a defendant that wishes to present false testimony, there is no dispute that defense counsel may not call witnesses who intend to commit perjury. ABA Model Rules of Professional Conduct, Rules 1.2, 3.3, 3.4.

Which of the following pairings does NOT describe categories or classifications of crimes? a. Felony and misdemeanor b. Judicial and administrative c. Federal and state d. Malum in se and Malum prohibitum

b. Judicial and administrative "Judicial" and "administrative" generally describe types of hearings, not crime categories

Lexi got a frantic call from Darcy, her pregnant younger sister. Darcy was crying uncontrollably and said that her abusive husband Rick had beat her up and dragged her out of the house by her hair, leaving her stranded in the freezing cold without a coat. Darcy said she called 911, but the operator said it would take an hour to get an officer out to their remote location. Lexi arrived at Rick and Darcy's house before the police, but Rick refused to let Lexi in the house. Lexi could clearly hear that her sister Darcy was now inside the house and crying out for Lexi's help. Lexi, concerned that Darcy was being physically abused, started throwing rocks through the windows of the house hoping her actions would force Rick to come outside (and leave Darcy alone) until the police arrived. The plan worked. Rick came outside to confront Lexi, and the two got into a heated argument. Shortly thereafter, the police arrived and escorted Darcy out of the house and arrested Rick for domestic violence. When Rick told the police about the property damage Lexi caused with the rocks, Lexi was also arrested and charged with felony destruction of property. Which of the following would be most correct? a. Given Lexi's reasonable and legitimate concern for Darcv's well-being and Rick's abusive behavior towards Darcy, Lexi has a strong argument that she was under extreme duress when she caused the property damage b. Lexi's best defense would be that she was faced with a choice of evils between breaking some windows or allowing her sister to face death or serious bodily injury before the police arrived c. Lexi can mount a strong duress and necessity defense under these facts d. None of the above is correct

b. Lexi's best defense would be that she was faced with a choice of evils between breaking some windows or allowing her sister to face death or serious bodily injury before the police arrived In the choice between necessity and duress, a necessity defense is much stronger. Lexi reasonably perceives it is necessary to distract Rick until the police arrive in order to prevent "a greater harm or evil" to Darcy, who was already a domestic violence victim prior to Lexi's arrival. The property damage she caused while awaiting the arrival of the police—even if thousands of dollars—was necessary to save her sister's life. The facts simply do not support a finding that Lexi was threatened, coerced, or forced to commit the crime of destruction of property. More accurately, Lexi destroyed the windows out of necessity.

Mark purchased an illegal loaded gun to kill Aden. When Mark saw Aden, he pointed the gun at Aden and pulled the trigger. The gun did not fire because it was inoperable. Aden calls the police and Mark is arrested for criminal attempt. Which of the following is correct? a. Mark has not committed the crime of attempted murder because the gun was inoperable and he could not have killed Aden using that gun b. Mark committed the crime of attempted murder as soon as he purchased the gun with the intent to use the gun to kill Aden c. Mark is guilty of attempted murder because he intended to kill Aden with the gun, aimed the gun at Aden, and pulled the trigger d. Both B and C are correct

b. Mark committed the crime of attempted murder as soon as he purchased the gun with the intent to use the gun to kill Aden Under § 5.01(1) of the Model Penal Code Mark committed an attempt because he engaged in a conduct that would constitute the crime if the circumstances "were as he believed them to be." The fact that the gun was inoperable (unbeknownst to Mark) does not negate the fact that Mark approached Aden with a loaded gun that he thought was operable and tried to kill Aden.

Martin learned that his live-in girlfriend, Gina, was planning to break up with him and move out of the apartment they rented together. Martin went home to try to convince Gina to stay and planned to take her new laptop if she refused. When Martin arrived at their apartment, Gina had her back to him as she sat at her desk typing on her new laptop. When Martin started yelling, Gina stopped typing and stood up to face Martin. Without warning, Martin reached around Gina, snatched her laptop, and said, "You will never see this laptop again." Gina protested and demanded that he return her laptop, but Martin ignored Gina and fled the apartment. Which of the following would be most correct? a. Martin committed the crime of burglary because he entered the apartment with the specific intent to commit felony larceny of Gina's new laptop b. Martin committed the crime of larceny for taking Gina's laptop, but did not commit the crimes of burglary or robbery c. Martin committed the crime of robbery because he snatched Gina's laptop from her possession. d. Both A and C are correct

b. Martin committed the crime of larceny for taking Gina's laptop, but did not commit the crimes of burglary or robbery Martin took and carried away the personal property of another with the intent to permanently deprive her of possession ("you will never see this laptop again.") Because Martin did not take Gina's laptop while it was in her physical possession, his conduct did not meet the force requirement for robbery. Likewise, there was no burglary because Martin entered his own home and did not break and enter the dwelling "of another."

Angi, a grocery store cashier, stole $20,000 from the supermarket where she worked. Her theft was caught on the store security camera and the police sought to arrest Angi for felony larceny. When Angi's mother, Maud, learned of Angi's legal trouble, Maud gave Angi her car and told Angi to go hide out with her Aunt Sue in the country. Maud then called Aunt Sue to tell her that Angi was in danger. Without knowing any details, Aunt Sue immediately agreed to let Angi stay at her house and agreed to protect Angi from harm. Which of the following is correct? a. Both Maud and Aunt Sue acted as accomplices and, along with Angi, can be charged with felony larceny b. Maud is an accessory after the fact, but Aunt Sue has not committed a crime c. Only Maud is an accomplice; Aunt Sue has not committed a crime d. Maud is an accomplice and Aunt Sue is an accessory after the fact.

b. Maud is an accessory after the fact, but Aunt Sue has not committed a crime Maud, with full knowledge that her daughter committed a crime and was "on the run" from police, had the specific intent to assist in her daughter escape and took action to secure a "hideout" or safe harbor for Angi. This level of conduct after the crime has been committed would subject Maud to criminal liability as an accessory after the fact. Aunt Sue, however, was unaware that she was agreeing to harbor a fugitive. Therefore, her actions in allowing Angi to stay at her home to protect Angi from some unspecified danger would not subject her to criminal liability because she did not have the requisite criminal intent.

Which of the following correctly lists the chronological order of the final stages of a criminal trial? a. Presentation of defense case, jury instructions, motion for judgment of acquittal, defense theory instruction b. Motion for judgment of acquittal, presentation of the defense case, defense theory instruction, jury deliberations c. Defense theory instruction, motion for judgment of acquittal, presentation of the defense case, jury instructions d. None of the above

b. Motion for judgment of acquittal, presentation of the defense case, defense theory instruction, jury deliberations Generally, after the prosecution rests, or completes its presentation of its case, the defense will make a motion for judgment of acquittal (MJOA) to argue that the government has not met its burden of proof. If the judge grants the motion, some or all of the charges will be dismissed. If all of the charges are dismissed, the case is over and the defense will not need to present any evidence. If the MJOA is denied, the defense will have an opportunity to present evidence for the defense, and the defense theory instruction would be given during the court's final instructions to the jury before they begin deliberations.

Finley's parents are Jayden and Ainsley. Finley commits a purposeful murder following an altercation with a college classmate. As Finley was 19 years old, Finley was able to secure a gun without the aid of parents Jayden and Ainsley. Finley was away at college when this crime was committed. Parents Jayden and Ainsley raised Finley well and provided Finley with proper mental health care throughout Finley's life. Which of the following is most correct? a. Prosecutors can charge parents Jayden and Ainsley with the homicide committed by Finley because but for them giving birth to Finley this crime would not have been committed b. Prosecutors cannot charge parents Jayden and Ainsley with the homicide committed by Finley because they did not cause this purposeful murder

b. Prosecutors cannot charge parents Jayden and Ainsley with the homicide committed by Finley because they did not cause this purposeful murder A condition is not considered part of the actual cause, so therefore parents Jayden and Ainsley did not cause this homicide (see § 6.01). A is incorrect because the birth of a child is considered a condition and is not considered causation for the purposes of determining the actual cause.

Reed contacts Courtney trying to get Courtney to go on a bank robbery. Courtney says "OK, see you tomorrow-_bank robbery day." The next day, Reed, calls Courtney and spends three hours convincing Courtney that this was really a crazy idea to think about robbing a bank. Reed says, "We really should not do this, we'll get kicked out of school and it will ruin our lives." Courtney finally agrees not to go on a bank robbery and says, "Let's go to a movie, who needs to be involved in the commission of a bank robbery?" Which of the following is most correct in a Model Penal Code jurisdiction? a. Both Reed and Courtney can be convicted of bank robbery b. Reed has an affirmative defense if charged with bank robbery, since Reed persuaded Courtney not to commit the crime c. Only Courtney can be charged with bank robbery since Courtney was ready to commit the offense until Reed talked Courtney out of committing the offense.

b. Reed has an affirmative defense if charged with bank robbery, since Reed persuaded Courtney not to commit the crime Pursuant to Model Penal Code § 5.02(3) renunciation can be an affirmative defense when the actor "after soliciting another person to commit a crime, persuaded him not to do so or otherwise prevented the commission of the crime, under circumstances manifesting a complete and voluntary renunciation of his criminal purpose." Here Reed successfully talked Courtney out of committing the crime, and it appears to be a complete and voluntary renunciation of the crime. A is incorrect because there has been no act in furtherance of the agreement to commit a crime, and even if there was an agreement to commit bank robbery, the substantive offense would be conspiracy to commit bank robbery and not just bank robbery. C is incorrect because Courtney did not do an act in furtherance of any agreement to commit a crime.

Rowan is charged with arson. The government presented five witnesses at Rowan's trial. Which of the following is most correct? a. If Rowan presents no defense, Rowan will be found guilty b. Rowan does not have to present any defense c. The government has the right to call Rowan as a witness in his trial

b. Rowan does not have to present any defense A defendant need not offer any defense. It has long been established that the burden of proof rests with the prosecution, and the defense does not need to present any evidence. A is incorrect as a failure of the defense to present a defense does not necessarily mean that the accused will be found guilty. The prosecution has to prove the elements of the crime beyond a reasonable doubt and a failure to do so results in the defense being successful. B is incorrect as a defendant has the right to remain silent. This includes the right not to be called as a witness against themselves.

The gambling statute in the State of Erewhon provides that if five or more persons conduct, supervise, or participate in gambling activity, including playing poker for money, they are guilty of gambling, a misdemeanor. Sarah plays poker with six friends and wins one hundred dollars ($100.00) at the poker game. Two of the six friends she was playing poker with report Sarah to the local police, who arrest her and charge her with the crime of conspiracy to commit gambling. Which of the following is most correct? a. Sarah would have a Wharton's Rule argument if she is charged with a conspiracy offense b. Sarah will not have a Wharton's Rule argument if she is charged with a conspiracy offense

b. Sarah will not have a Wharton's Rule argument if she is charged with a conspiracy offense When the crime by its very nature requires a set number of people, in this case five, courts have precluded use of conspiracy or made it a presumption that can be used when you have that set number of individuals. When, however, there are more than the required number of individuals set in the statute, Wharton's Rule does not apply. Because this gambling statute speaks to five individuals and Sarah had six friends with her playing poker, Wharton's Rule would not be a defense to the conspiracy charge. A is incorrect because the set number in the statute has been met, so Wharton's Rule would not be a successful argument for Sarah.

Sharon, a nurse at the county jail, was tasked with providing medical care to inmates. One of the inmates, Craig, was a known methamphetamine ("meth") addict. Shortly after Craig was detained at the jail he began vomiting and was unable to eat. Sharon was informed of Craig's condition by the jail guards but did not examine him or render medical care. Two days later, Craig began slurring his speech and, eventually, could only whisper "help me." In response, Sharon walked by and looked at Craig without examining him. At one point Sharon walked past Craig's cell as he was crying out for help and stated, "That's what he gets for using meth." Later that night Craig died, not because of meth withdrawal, but because he was a diabetic and had not received insulin for several days. Craig's diabetes was documented by Sharon on his jail admissions medical record. Also, multiple times in the days before his death Craig's family called or came to the jail with his diabetes medication, but Sharon refused to receive the medication. What is the most serious crime supported by these facts? a. First degree premeditated murder b. Second degree murder c. Voluntary manslaughter d. Involuntary manslaughter

b. Second degree murder The facts show that Sharon had a duty to provide medical care to Craig, knew he was ill, and knew he was a diabetic. The combination of her duty of care and her degree of awareness of his health issues support a finding that Sharon caused Craig's death because she consciously disregarded a known risk of death or serious bodily harm. A and C are incorrect because there are no facts to support a finding that Sharon had a specific intent to kill Craig. D is wrong because the crime of involuntary manslaughter, though supported by these facts, is not the most serious crime supported by these facts.

Same basic facts as Question 22 above. Subsequent investigation revealed one additional fact: the father knew immediately that Bart ingested meth because the father saw the white powder around Bart's mouth and the father told the neighbor: "I can't take Bart to the hospital because they will arrest me for having meth in the house." Based on this new information what is the most serious criminal charge supported by the facts? a. First degree felony murder because Bart died during the course of his father's commission of the felony drug crime b. Second degree murder because the father was aware of the child's dire medical condition and delayed seeking medical treatment c. First degree premeditated murder because the father's knowledge that Bart ingested the meth and his failure to immediately seek medical care support a finding of premeditation and deliberation d. Involuntary manslaughter because the father did not supply Bart with the drugs or leave the drugs in a location where they were easily accessible by Bart

b. Second degree murder because the father was aware of the child's dire medical condition and delayed seeking medical treatment The additional facts reveal that the father was aware at the outset that his minor child (to whom he owed a duty of care) had ingested some quantity of meth and was facing a high risk of serious bodily harm or death. With his degree of awareness of the risk, it was extremely reckless for the father to withhold or delay medical treatment for any reason. A is wrong because a felony drug charge is not an enumerated or "inherently dangerous" felony. C is wrong because the father's knowledge that Bart ingested meth and his desire to avoid criminal liability is not sufficient evidence that the father either wanted Bart to die or premeditated and deliberated killing Bart. D is wrong because gross negligence exists when the father is unreasonably unaware of the risk of death (i.e., he did not know the child ingested meth). Here, knowing that the child was suffering from a meth overdose makes his failure to seek medical care reckless because he consciously disregarded the risk of death.

When leaving Mexico to return home to San Diego, Paolo was confronted by several armed men who demanded that he transport two liquid-filled plastic water bottles into the U.S. The men told Paolo that they would hurt his grandfather in Mexico if he refused. Paolo took the bottles, but he was stopped by border agents when he tried to enter the U.S. The agents were suspicious of the slightly tinted substance in the clear water bottles. When the border agents opened the bottles and sniffed the contents, they concluded, based on their training and experience, that the bottles contained a highly concentrated form of liquid methamphetamine (meth). Because Paolo steadfastly argued that the bottles contained only fruit juice, the agents offered Paolo a chance to prove it by drinking some of the liquid. When Paolo barely took a sip, the agents smirked and directed him to taste the liquid again, then again and then a fourth time. The agents seized both bottles, had the liquid tested, and confirmed that the bottles were filled with meth. By then, Paolo had labored breathing and was sweating profusely. Paolo soon died of a meth overdose. What is the most serious homicide charge against the border agents supported by these facts? a. First degree premeditated murder b. Second degree murder, extreme recklessness c. Second degree murder, specific intent to kill d. First degree felony murder

b. Second degree murder, extreme recklessness The agents were aware of the risk of death or serious bodily injury to Paolo before he drank the liquid. Given their training and experience, in challenging him to taste the liquid multiple times they disregarded a known risk of death or serious bodily harm. A and C are wrong because there is no evidence that the agents had a specific intent to kill Paolo. D is wrong because Paolo was not killed during the commission of an "inherently dangerous" or enumerated felony.

In which of the scenarios below would the judge deny a defense request for a voluntary manslaughter jury instruction? a. A fistfight initiated by the decedent escalated when the decedent pulled out a knife and tried to stab the defendant. The defendant struggled with the decedent to gain control of the knife and then, in a blind rage, stabbed the decedent a dozen times and killed him. b. Several days after a physical altercation between Anne and Marcy, Marcy tracks down Anne, ambushes her in a dark alley and shoots her in revenge c. A mother comes home unexpectedly and discovers the babysitter's boyfriend sexually assaulting her minor child. The mother pulls the man off her child and beats him to death with a heavy ashtray d. A woman is approached from behind by a man who attempts to abduct her. She knocks him unconscious with her heavy purse and then rams her spiked shoe heel into his neck, causing him to die

b. Several days after a physical altercation between Anne and Marcy, Marcy tracks down Anne, ambushes her in a dark alley and shoots her in revenge The government could establish premeditation and deliberation for first degree murder based on the fact that the woman waited "several days" to track down, ambush, and shoot an adversary. A voluntary manslaughter instruction is unlikely because the decedent did nothing to provoke the defendant at the time of the fatal encounter. A, C, and D correctly describe intentional homicides where the defendant acted in the heat of passion and used deadly force in response to adequate provocation by the decedent.

The police found cocaine in Steve's car. Steve now faces a drug charge with a maximum penalty of ten years in prison. Which of the following statements is true? a. Steve is charged with a regulatory offense b. Steve is charged with a felony offense c. Steve is charged with a misdeameanor offense d. None of the above

b. Steve is charged with a felony offense In general, a felony offense is defined as a crime that carries a penalty of one year or more

Gina works as a law clerk for a criminal defense attorney in Ohio. She is assigned to work on an Ohio armed robbery case. What is the best source for Gina to consult in order to determine the elements and the penalty for armed robbery in Ohio? a. The indictment filed in the case b. The Ohio Criminal Code c. The Model Penal Code d. The Ohio State Constitution

b. The Ohio Criminal Code Modern criminal laws are set forth in criminal statutes passed by the state legislative body. Criminal statutes are the most definitive source for determining the definition or elements of a crime, as well as the potential punishment that could be imposed

John, an airline pilot, is charged with willfully failing to file income tax returns. He did not file his tax returns because he believed that wages were not income. He also argued that it was unconstitutional to require individuals to pay income tax. Which of the following is most correct? a. The court should give an instruction that allows the jury to convict the defendant if a reasonable person would believe that wages were not income but reject his argument based on the constitutionality of paying taxes b. The court should give an instruction that allows the jury to consider whether the defendant believed that wages were not income but reject his argument based on the constitutionality of paying taxes c. The court should refuse to give an instruction on whether the accused believed that wages were not income but should accept the defendant's argument on the unconstitutionality of paying taxes

b. The court should give an instruction that allows the jury to consider whether the defendant believed that wages were not income but reject his argument based on the constitutionality of paying taxes In Cheek v. United States the Court held that the statute provides a subjective standard for whether the accused acted in willfully filing his taxes, but the Court rejected the argument on whether taxes are constitutional (see § 5.07 (B)). A is incorrect because it provides an objective standard of determining the defendant's belief, which was reversed in the Cheek case. C is incorrect because it rejects a subjective standard of belief on whether wages are income and also finds paying taxes unconstitutional, both contrary to the Cheek case.

Which of the following statements does not accurately describe the "void for vagueness" doctrine? a. The doctrine provides a person of ordinary intelligence with fair notice of what is prohibited under the law b. The doctrine has identical requirements as the doctrine of overbreadth c. The doctrine is designed to prevent arbitrary and discriminatory enforcement d. The doctrine is rooted in the Fourteenth Amendment's due process clause?

b. The doctrine has identical requirements as the doctrine of overbreadth A is an incorrect answer to this question because this is the statement that is used to describe the void for vagueness doctrine. C is an incorrect answer because one of the policy rationales for the void for vagueness doctrine is to avoid arbitrary and discriminatory enforcement. D is an incorrect answer as the void for vagueness doctrine originates from the due process clause of the Constitution. Although overbreadth can lead to vagueness, it is not identical to that term. Overbreadth pertains to a statute that encompasses too much conduct and where the legislature has failed to narrowly define what is encompassed within the terms of the statute. Overbreadth can be prohibited when the language includes constitutionally protected behavior.

Monroe is charged with a drug offense in State A and found guilty of the crime after a trial by jury. The federal government also brings a drug charge for the same conduct and same drugs against Monroe. Which of the following is most correct? a. The federal government's case will be dismissed as a violation of double jeopardy b. The federal government's case will be allowed because of the Dual Sovereignty Rule c. The federal government's case will be dismissed as a violation of the Dual Sovereignty Rule

b. The federal government's case will be allowed because of the Dual Sovereignty Rule The Dual Sovereignty Rule permits the retrial of a defendant on the same criminal conduct if the prosecuting authority is different. Because the first prosecutor was a state prosecutor and the second is coming from the federal government, there are two separate sovereigns proceeding on the case and therefore allowed under the Dual Sovereignty Rule.

John and Tyron were convicted of violating a state statute that provided: "[a] person commits an offense if he engaged in deviate sexual intercourse with another individual of the same sex." When arrested, the two men were engaged in sexual acts within their private residence. Which is the strongest argument for finding this statute unconstitutional? a. The statute is vague because it fails to define what consitutes deviate sexual intercourse b. The statute violates the right to privacy guaranteed by the Due Process Clause of the Constitution c. The statute violates the right against cruel and unusual punishment under the Eighth Amendment of the Constitution d. The statute is constitutional because prior Court precedent found such conduct to be criminal

b. The statute violates the right to privacy guaranteed by the Due Process Clause of the Constitution

Drew has a medical condition that requires him to take a prescription drug. Early Friday morning, Drew got a refill of his prescription at the pharmacy and took his medication as prescribed. Later that evening, Drew casually sipped one beer at a local bar during a two-hour period. When Drew got into his car to drive home he felt fine and exhibited no signs of intoxication. Ten minutes later, Drew blacked out while driving, slammed his car into another vehicle, and killed the two passengers. Hospital blood tests showed that Drew was not intoxicated, but the tests revealed that the pharmacy gave Drew a much stronger dosage of medicine than was prescribed. As a result, any amount of alcohol would act as a catalyst and cause physical impairment and loss of consciousness. Notwithstanding, Drew is charged with "recklessly operating a motor vehicle in a manner that caused the death of another." Which of the following would be most correct? a. Because Drew is charged with a crime involving recklessness, he will not be able to assert an intoxication defense under the Model Penal Code b. Under the Model Penal Code, because Drew was suffering "pathological intoxication" at the time of the accident due to pharmacy error, Drew would be able to present an intoxication defense c. Under the Model Penal Code, Drew cannot be prosecuted for his actions because he was involuntarily intoxicated by the pharmacy d. Under the Model Penal Code, Drew would need to present evidence of the defense of insanity--not intoxication--to assert that he lacked the capacity to appreciate the wrongfulness of his actions

b. Under the Model Penal Code, because Drew was suffering "pathological intoxication" at the time of the accident due to pharmacy error, Drew would be able to present an intoxication defense Under the MPC § 2.08(4), Drew's intoxication was not "self-induced" because the actual cause of his impairment was the medication error. As such, Drew would be entitled to present evidence that he lacked "substantial capacity to either appreciate the criminality [wrongfulness] or confirm his conduct to the requirements of law." A is wrong because the recklessness restriction applies only to voluntary intoxication. C is wrong because involuntary intoxication is a defense, not a bar to prosecution. D is wrong because § 2.08(3) makes clear that, unlike insanity, intoxication is not a mental disease.

In the case Queen v. Dudley and Stephens, the English court was being asked to decide which of the following questions? a. Whether Dudley and Stephens had killed Richard Parker b. Whether the killing by Dudley and Stephens was a felony and a murder c. Whether Dudley and Stephens were deprived of the right to present a defense d. Whether Dudley and Stephens had committed an actus reus e. Whether Dudley and Stephens had the appropriate mens rea for the crime charged

b. Whether the killing by Dudley and Stephens was a felony and a murder The jurors could not decide whether this killing was in fact a felony and murder, a crime that in the common law of England provided for a death sentence unless commuted by the Crown (Home Office). The two main issues examined by the court were whether the conduct of Dudley and Stephens was in fact a crime and whether the circumstances of being stranded at sea without food constituted a necessity.

Using the statements and the statutory provisions below, determine whether sexual assault laws were violated. "My name is Ramanda. I decided to get completely drunk at my colleague's party. I must have passed out. At some point, I became aware of the fact that this guy I met, Spence, was on top of me and having sexual intercourse with me. I shoved him off, and he ran out of the room. I do not remember ever agreeing to have sex with him. My name is Spence. I had consensual sex with Ramanda at a party. I have produced several text messages that she sent me earlier that night inviting me to have sex with her! She flirted with me all night and led me into her friend's bedroom to have sex. Ramanda suddenly passed out just as was starting penetration, and then woke up later and completely flipped out." Under which statutory provision would the prosecution have the strongest case for prosecuting Spence? a. "It is sexual assault for a person to engage in sexual penetration of another if the person has substantially impaired the other person's power to appraise or control the situation by administering or employing drugs, intoxicants, or other similar means" b. "A person is guilty of sexual ass left if the other person is unconscious and there is no evidence of prior consent" c. "A male is guilty of sexual assault if he has sexual intercourse or sexual contact with a female and he knows or has reason to know that she is unaware that a sexual act is being committed upon her" d. "A person is guilty of sexual assault if the person has sexual contact or sexual intercourse with a person who is under the influence of an intoxicant to a degree which renders the person incapable of giving consent if the defendant has the purpose to have sexual contact with the person while the person is incapable of giving consent."

c. "A male is guilty of sexual assault if he has sexual intercourse or sexual contact with a female and he knows or has reason to know that she is unaware that a sexual act is being committed upon her" Under this statutory provision, Spence can be found guilty if he has "reason to know" that Ramanda was unaware that a sex act was being performed on her. The fact that Spence knew Ramanda passed out before penetration would support a finding that he had reason to know she was unaware of what was happening to her. A is incorrect because, although Ramanda was intoxicated, Spence did not administer the alcohol to Ramanda. B is incorrect because Spence's text messages would provide evidence of prior consent. D is incorrect because the government would have to show that Spence knew Ramanda was too drunk to consent and that he wanted to "purposely" have sex with her when she was too drunk to consent. Because of this level of level of criminal intent required, this statutory provision does not provide the "strongest case" against Spence.

Using the statement and the statutory provisions below, determine whether sexual assault laws were violated. "My name is Sela. I am 18 years old, and I am in the 9th grade at the Gardner Special Education School. I have an intellectual disability, but I wanted to have sex with Barney, the man who drives my school bus. When I told Barney that I wanted to have sex with him he agreed, and I really enjoyed it. Barney is cute and sexy." The video surveillance recording on the bus captured Sela asking Barney, "what does 'sex' mean" and "how do you go to sex" before the two engaged in sexual intercourse." Which statutory provision provides the strongest grounds for charging Barney for engaging in sexual intercourse with Sela? a. "A person is guilty of sexual assault if the person engages in or causes another person to engage in or submit to a sexual act where the person knows or has reason to know that the other person is a minor who is incapable of communicating unwillingness to engage in that act" b. "A person is guilty of sexual assault if the person engages in or causes another person to engage in or submit to a sexual act where the person knows or has reason to know that the other person has a mental disability and does not want to participate in a sexual act" c. "A person is guilty of sexual assault, regardless of consent, if the person engages in or causes another person to engage in or submit to a sexual act and the other person suffers from a disability that is reasonably apparent or known to the actor, and which in fact renders the other person substantially incapable of appraising the nature of the contact involved" d. "A person is guilty of sexual assault if the person engages in or causes another person to engage in or submit to a sexual act when it is reasonably apparent or known to the actor that the other person suffers from mental illness or psychological impairment"

c. "A person is guilty of sexual assault, regardless of consent, if the person engages in or causes another person to engage in or submit to a sexual act and the other person suffers from a disability that is reasonably apparent or known to the actor, and which in fact renders the other person substantially incapable of appraising the nature of the contact involved" Barney knew that Sela suffered from a disability because Sela attends a special education school and is only in the 9th grade, despite being 18 years old. Beyond that, Sela's questions before they had sexual intercourse indicated that she did not understand the meaning of having sex which made Barney "reasonably aware" the Sela was "substantially incapable" of understanding "the nature of the contact" involved in the sex act. A is wrong because that statute applies only to minors, and Sela is 18 years old. B is wrong because Sela consented to sex, and the statute only applies if the person does not want to participate in sexual activity. D is wrong because an intellectual disability is not a form of mental illness or psychological impairment.

Adair is on a business trip to a city in which Adair previously lived. To surprise Chandler, someone Adair previously dated, Adair breaks a window and enters into the home of Chandler at nighttime. Upon entering, Adair finds Chandler in bed with another woman. Chandler calls the police and Adair is thereafter arrested. Which of the following is most correct? a. Adair can be convicted of common law burglary because Adair broke a window and therefore had the specific intent to break and enter the home of Chandler b. Adair can be convicted of trespassing because trespassing is a specific intent crime and Adair had the specific intent to break and enter the home of Chandler c. Adair can be convicted of trespassing because Adair had the general intent to enter Chandler's home d. Adair cannot be convicted of any crimes because Adair did not have a criminal intent

c. Adair can be convicted of trespassing because Adair had the general intent to enter Chandler's home Trespassing is a general intent crime, and Adair entered the premises and therefore trespassed (see § 5.01). A is incorrect because common law burglary is a specific intent crime. Adair did not have the intent to commit a felony therein, but rather to surprise Chandler. B is incorrect because trespassing is a general intent and not a specific intent crime. D is incorrect because Adair did not have Chandler's permission to enter the home, and broke a window to enter the home, therefore committing a crime of trespass.

After calling his sister Kondi for several hours, Alfie decided to go to her house to check on her. As Alfie approached the house, he heard loud arguing, falling objects, and noises consistent with a physical altercation. Knowing that Kondi's husband Harry has a violent temper, Alfie decided to break a window and enter the house. Alfie encountered Kondi and Harry on the floor fighting for control of a gun. Alfie pulled out his gun and demanded that Harry move away from Kondi. When Harry ignored his repeated request, Alfie pulled out his licensed handgun and fatally shot Harry. Home security video showed that when Harry walked in the door Kondi had her handgun trained on him and fired one shot at him before Harry lunged at her and tried to take the gun from her. Alfie entered the home about 60 seconds later. Alfie is charged with Harry's murder and claims he acted to defend his sister. Which of the following is accurate under the Model Penal Code? a. Alfie does not have a valid defense of others claim because Kondi was the first aggressor b. Alfie's mistaken belief that Harry was the first aggressor will likely result in the judge refusing to instruct the jury on Alfie's defense of others claim c. Alfie has a valid defense of others claim if he reasonably believed that Harry was the first aggressor and Kondi was in imminent danger of death or serious bodily harm from Harry d. None of the above is accurate

c. Alfie has a valid defense of others claim if he reasonably believed that Harry was the first aggressor and Kondi was in imminent danger of death or serious bodily harm from Harry Under the Model Penal Code a defendant still has a valid defense of others claim if the defendant uses force during an altercation to assist the first aggressor and the defendant reasonably believes the first aggressor is the victim. A and B are incorrect because the Model Penal Code has rejected the common law "alter ego" rule in favor of the "reasonable belief" standard.

When Sarah (a second-year law student) and Amy (a first-year law student) met in the law school library a couple days before the start of the fall semester, they learned that Amy's law school courses required the same textbooks that Sarah used during her first year of law school. Sarah and Amy reached an agreement for Amy to purchase Sarah's used books for $500. Sarah delivered the books to Amy, and Amy agreed to pay the $500 in two weeks when she received her scholarship money. A few days later, Amy learned that her scholarship award was rescinded because she did not submit the required documentation on time. Amy had no other means to pay for the books, but refused to return them to Sarah. After two months Sarah reported Amy to the police. Which of the following would be most correct? a. Amy has committed the crime of larceny by trick b. Amy has committed the crime of false pretenses c. Amy has committed larceny d. Amy has not committed a property crime because lacked the criminal intent to steal the books when she obtained them. Sarah must seek a civil remedy

c. Amy has committed larceny Even though Amy did not have the intent to permanently deprive Sarah of her property at the time she accepted the books, under the continuing trespass doctrine, her subsequent decision to keep the books without making payment constitutes larceny. Amy has not committed the crimes of larceny by trick or false pretenses because she did not induce Sarah to voluntarily relinquish possession of the books based on lies, fraud, or misrepresentation. Her representations regarding her scholarship were true at the time the sales agreement was made.

Which of the following lists of stages in the criminal adjutication process is in the correct chronological order? a. Grand jury indictment, arraignment, trial, discovery process b. Bail, preliminary hearing, entry of guilty plea, grand jury indictment c. Arrest, bail, trial, sentencing appeal d. Preliminary hearing, arraignment, grand jury indictment, sentencing appeal

c. Arrest, bail, trial, sentencing appeal

Johnson and Smith were both customers in the check-out line at the 7-11 convenience store when they got into a verbal disagreement over whether the other was attempting to "jump" or "cut" the line. The heated verbal dispute escalated when Smith, a white female customer, screamed an extremely offensive racial epithet at Johnson, an African American female. Johnson instantly responded by violently striking Smith in the head with the gallon of milk she was holding. The force of the blow caused Smith to fall backwards and slam her head on the concrete floor. Smith died. Which of the following statements is true? a. Johnson has likely committed felony murder if her vicious and violent assault of Smith with the milk container is a felony and Smith died in the course of Johnson's felonious conduct. b. The most serious crime supported by the facts is voluntary manslaughter because Smith's racist verbal assault would cause a reasonable person to be provoked into a violent response, and Johnson was enraged when she struck Smith c. Based solely on these facts, if Johnson is charged with first degree premeditated murder, the trial judge is likely to grant a defense motion for judgment of acquittal (directed verdict) d. The most serious crime supported by the facts is involuntary manslaughter because Johnson struck Smith in the heat of passion e. None of the above are true

c. Based solely on these facts, if Johnson is charged with first degree premeditated murder, the trial judge is likely to grant a defense motion for judgment of acquittal (directed verdict) Because there are insufficient facts to find that Johnson planned or thought about killing Smith before striking her. It appears that Johnson made a rash decision to hit Smith immediately after Smith made the offensive remark. Therefore, the elements of premeditation and deliberation would not be met, and the trial court would likely grant a defense motion for judgment of acquittal on a first degree premeditated murder charge. A is not correct because an assault—even a felony assault—is generally not included among the "inherently dangerous" or enumerated felony offenses required for felony murder liability. D is not correct because Smith was intentionally struck with the gallon of milk; it was not an accident or an act of gross negligence by Johnson. B is incorrect because the long-standing common law rule in nearly every jurisdiction provides that "mere words" (even offensive words like racial slurs) do not constitute the adequate provocation needed to reduce murder to voluntary manslaughter.

Though married, John has been having a secret affair with his co-worker Kerri for several months. On June 1st, immediately after having unprotected sexual intercourse with John in her bedroom, Kerri informed John that she was HIV positive. John instantly flew into a rage, screamed at Kerri, and then grabbed a knife on the nightstand and fatally stabbed Kerri in the chest. John is charged with first degree premeditated murder. Which of the following is true? a. The judge will likely dismiss the case before trial on the grounds that it was reasonable for John to react suddenly and violently upon learning that he might be infected with HIV. b. Because John's fatal assault on Kerri happened mere seconds after he learned of Kerri's HIV status, the prosecution should be able to get a felony murder instruction. c. Because of the shock of learning that someone you just had unprotected sex with is HIV positive, John would likely be entitled to a voluntary manslaughter instruction. d. Both B and C are correct.

c. Because of the shock of learning that someone you just had unprotected sex with is HIV positive, John would likely be entitled to a voluntary manslaughter instruction. Kerri's disclosure constituted provocative "informational words" that would constitute adequate provocation. John's immediate explosive reaction supports a finding that he acted in the heat of passion. A is incorrect because whether John's reaction was "reasonable" is a factual determination to be decided by the factfinder at trial, not a basis for a pretrial dismissal. B is incorrect because there is no "inherently dangerous" or enumerated felony to support a felony murder charge.

Robert is charged with both conspiracy to commit burglary and burglary. These two charged crimes are for the same home burglary occurring on the same day and time and with the same individuals conspiring to commit this offense. Which of the following is most correct? a. In a Model Penal Code jurisdiction, Robert cannot be convicted of both burglary and the conspiracy to commit burglary b. In the federal system, Robert can be convicted of both the burglary and the conspiracy to commit burglary c. Both A and B

c. Both A and B The Model Penal Code places the prosecutor with the choice of whether to proceed with the underlying crime or the conspiracy. The two offenses merge. A is therefore a correct statement. The majority rule is that the crime of conspiracy is a separate offense from any substantive crimes committed in the course of the conspiracy and therefore one can be convicted of both crimes. This is the position taken by the federal system that commonly charges both the underlying crime and the conspiracy to commit that crime (see § 13.02). B is therefore a correct statement. The most correct statement would therefore be C as it includes both A and B.

Sloan and Noah both want to kill Kelcy. Sloan enters the room where Kelcy is standing and shoots him. The coroner testifies that the gunshot by Sloan would have caused Kelcy to die in about one hour. Fifteen minutes after Sloan shot Kelcy, Noah enters the same room and shoots Kelcy. Kelcy dies instantly. Which of the following is most correct? a. Sloan can only be guilty of attempted murder because Kelcy died as a result of Noah's shooting him b. Only Noah can be guilty of murder c. Both Sloan and Noah can be guilty of murder d. Both A and B above e. Sloan and Noah cannot both be guilty of the death of Kelcy because two people cannot be guilty of killing one person if they act independently of each other

c. Both Sloan and Noah can be guilty of murder Accelerating a result allows holding a person liable for causing a death at a particular point in time before the victim otherwise would have died. Because the coroner states that Sloan's bullet would have caused the death of Kelcy and because Noah's bullet accelerated the death, both can be held criminally liable for this homicide. Acceleration requires specific proof of when the victim would have died to establish the liability of the first defendant, as well as how the second defendant's conduct caused the otherwise inevitable death to occur earlier.

Brooklyn is charged with battery, which in this jurisdiction is defined as an intentional touching or striking of another person against that person's will. Brooklyn was standing outside a house with best friend Skyler. Brooklyn suddenly has a seizure causing Brooklyn's right hand to land on Skyler's eye. Skyer suffers a black eye from this incident, which results in the arrest and charge of battery against Brooklyn. Which of the following would be Brooklyn's best arguments against the battery charge? a. That Skyler only suffered a black eye and that this would not be a proper basis for a battery charge b. Skyler could have moved out of the way and avoided being hit by Brooklyn's right hand c. Brooklyn actd convulsively and thus the act cannot be the basis for battery d. The statute does not require that Brooklyn do an act, it only requires an intent to act e. Brooklyn was negligent, so therefore there are no arguments that can be made against this charge

c. Brooklyn actd convulsively and thus the act cannot be the basis for battery Because Brooklyn acted convulsively, there was no actus reus on the part of the accused.

Steve was the manager of the night cleaning crew, and Helen was a new employee on his staff. Steve entered one of the offices Helen was cleaning, closed the door behind him, and made sexual advances toward Helen. Helen repeatedly rejected Steve's advances and yelled at Steve to stop. When Steve refused to back off and pushed Helen up against a wall, Helen grabbed a paperweight off the desk and bashed Steve in the head. Steve died. When the police arrived, Helen said: "I wasn't afraid of him, and I could have gotten away from him if I tried, but he should not have tried to sexually assault me. He deserved to die, and I am glad he's dead." The government has charged Helen with murder and she claims self-defense. Which of the following statements is true? a. If Helen could have safely gotten away from Steve, she had a duty to do so before resorting to deadly force. b. If Steve was unarmed, Helen did not have the right to use deadly force to repel his sexual advances c. Despite Steve's offensive and illegal sexual advances, if Helen did not fear death or serious bodily harm she did not have the right to use deadly force d. It does not matter whether Steve threatened deadly force or whether Helen was not afraid. Steve's sexual assault justified Helen's use of deadly force in retaliation

c. Despite Steve's offensive and illegal sexual advances, if Helen did not fear death or serious bodily harm she did not have the right to use deadly force Helen's statement makes clear that she did not fatally strike Steve with the paperweight because she feared either death or serious bodily injury from his sexual assault. Helen's statement makes clear that she attacked Steve, not in self-defense, but in retaliation. A is wrong because in most jurisdictions there is no duty to retreat before using force in self-defense. B is wrong because Helen's right to use force—including deadly force, if necessary—to defend herself is not contingent on whether Steve had a weapon. D is wrong because self-defense is only available when a defendant takes action to protect against a danger of death or serious bodily harm. Retaliation is punishment, not self-defense.

Eddie and Michael agreed to commit a robbery at the Apple store. According to their plan, Eddie would enter the store with a gun and take as much inventory as possible in two minutes, and Michael would block the entrance to the store and alert Eddie if the police arrived. The plan worked. Michael and Eddie took over $10,000 in iPads, iPhones, headphones, and related electronics equipment. Which of the following is correct under modern criminal statutes? a. Eddie's actions make him a principle, and Michael is an accessory after the fact b. Eddie's action make him a principle and Michael is an aider and abettor c. Eddie and Michael are both principals in the robbery, despite the fact that Michael never entered the Apple store d. Eddie committed a robbery, and Michael is only guilty of the lesser crime of being an accomplice.

c. Eddie and Michael are both principals in the robbery, despite the fact that Michael never entered the Apple store Eddie and Michael planned in advance and agreed to each play a role in the robbery. They both can be charged with the crime of robbery, regardless of who entered the Apple store to take the property. A is wrong because, unlike an accessory after the fact, a person that provides aid only after the crime has been committed, Michael played an active role before and during the robbery. B is wrong because under modern criminal statutes a person who is an "aider and abettor" is simply an accomplice and is guilty of the same crime as all other accomplices, in this case, robbery. D is wrong because criminal liability as an accomplice is not a separate, lesser crime.

Ellen, Marsha, and Shirley are in the illegal drug business. Ellen manufactures the illegal drugs and gives them to Marsha. Marsha then takes the drugs to Shirley who sells them on the streets to different individuals. Shirley has never met Ellen and has no idea where Marsha gets the drugs. Which of the following is most correct? a. Ellen, Marsha, and Shirley can be convicted for one conspiracy because this is a wheel and spoke type of conspiracy b. Ellen, Marsha, and Shirley cannot be convicted for one conspiracy because this is a chain conspiracy c. Ellen, Marsha, and Shirley can be convicted for one conspiracy because this is a chain and link conspiracy d. Ellen, Marsha, and Shirley cannot be convicted of one conspiracy because they all do not know each other and therefore could not have reached an agreement

c. Ellen, Marsha, and Shirley can be convicted for one conspiracy because this is a chain and link conspiracy This is a classic example of a chain conspiracy. All parties may not know each other, but they are all working toward the same essential nature of the plan, which in this case is to profit from the sale of illegal drugs. A is not correct as this scenario is not a wheel and spoke conspiracy. In a wheel and spoke conspiracy, it is typically one party serving as the hub forming individual conspiracies with each separate party. As opposed to one conspiracy, in a wheel and spoke conspiracy there are multiple conspiracies existing. B is not correct because having a chain conspiracy allows for one to be convicted and the answer provided says they would not be able to be convicted. D is not correct because it is not necessary to know all the parties' identities who are a part of the conspiracy as long as they are all working for the same essential nature of the plan.

Which of the following is a TRUE statement regarding the power of a prosecutor? a. If there is sufficient evidence to justify the defendant's arrest by the police, the prosecutor must file formal criminal charges b. If there is evidence that the defendant is likely to flee the jurisdiction and not return to court, the prosecutor can order the defendant into pretrial detention or require the defendant to pay a money bond to secure their pretrial release c. Even if the defendant is willing to plead guilty to misdemeanor charges that carry a lesser penalty, the prosecutor can reject the defendant's offer to plead guilty d. Even if the prosecutor plans to dismiss a case because they believe the evidence is not strong enough to prove the defendant is guilty, the judge can order that the prosecution proceed so that the jury -- not the prosecutor -- can decide if the defendant is guilty e. If the prosecutor works in a jurisdiction where felony cases are presented to. grand jury, even if the grand jury refuses to indict the defendant, the prosecutor still has the power to advance the felony charges to trial

c. Even if the defendant is willing to plead guilty to misdemeanor charges that carry a lesser penalty, the prosecutor can reject the defendant's offer to plead guilty Prosecutors have the exclusive power to determine whether to engage in or reject any plea bargain

Nancy and Ken have just gone through a nasty divorce. Nancy will not allow Ken to visit his six-year-old step-daughter, Lisa, even though Ken helped raise Lisa since she was a year old. When Ken learned that Nancy and Lisa were at the home of Nancy's parents, Ken decided to kidnap Lisa. Ken climbed up a ladder and used some tools to pry open an upstairs bedroom window. As Ken started to climb in the window, Nancy's father, Nick, came outside and angrily confronted Ken. Ken was startled, lost his balance on that ladder, fell ten feet to the ground and landed on top of Nick. Ken was injured, but Nick died instantly. What is the most serious crime Ken can be charged with based on these facts? a. Involuntary manslaughter b. Second degree mirder, extreme recklessness c. First degree felony murder d. Voluntary manslaughter

c. First degree felony murder The facts support a finding that Nick died during the course of a burglary and kidnapping—both enumerated or "inherently dangerous" felonies—perpetrated by Ken. Therefore, there are sufficient facts to support first degree felony murder. A is incorrect because involuntary manslaughter is not the most serious crime supported by the facts. B and D are incorrect because there is no evidence that Ken had a specific intent to kill Nick.

Garfield and Marcia are co-workers. Garfield wanted to kill Marcia because Marcia received a promotion that Garfield desperately wanted. Garfield ordered an incendiary device to attach to Marcia's car. Garfield's plan was to install the device on Marcia's car and program the device to explode when Marcia started her car the following day. When Garfield received the incendiary device, he took the device to Marcia's house and hid in the bushes on her property while waiting for Marcia to come home. For several minutes, Garfield remained in the bushes reviewing the instructions on how to install the device. Before Garfield could approach Marcia's car to install the device, a police officer arrested Garfield for trespassing and found the incendiary device in his backpack. Which of the following is correct under the Model Penal Code? a. Garfield committed criminal attempt when he ordered the incendiary device with the intent to use it to kill Marcia b. Garfield did not commit criminal attempt until Marcia arrived home c. Garfield committed criminal attempt when he went to Marcia's house with the incendiary device d. Garfield did not commit criminal attempt because he never approached the car and could have had a change of heart before installing the device

c. Garfield committed criminal attempt when he went to Marcia's house with the incendiary device Under § 5.01(2)(a) and (e) of the MPC, Garfield made a substantial step towards completing the crime when he arrived at Marcia's house with the incendiary device and hid in the bushes ("lying in wait") and by being in possession of materials, the incendiary device, that would "serve no lawful purpose." A is wrong because ordering the device is "mere preparation" for the criminal attempt. B is wrong because Garfield had already engaged in a substantial step towards committing the crime when he was lying in wait with the means to commit the crime. D is wrong because Garfield had already engaged in sufficient culpable conduct to constitute a substantial step without approaching Marcia's car.

Luke is an attorney representing Tamar, who has been charged with homicide. Luke believes that Tamar may be incompetent to stand trial. In determining whether Tamar is competent to stand trial, the judge must decide: I. Whether the defendant understands the proceedings against her I. Whether the defendant has the capacity to consult with her attorney with a reasonable degree of understanding of the proceedings III. Whether the defendant was insane at the time of the crime Which of the following is most correct? a. I b. II c. I and II d. I, II, and III

c. I and II Competency to stand trial differs from insanity. It is usually a pretrial determination in which a court ascertains if the accused (a) has the capacity to consult with his or her attorney "with a reasonable degree of understanding of the proceedings," and (b) whether the accused understands the proceedings against him or her. A is incorrect because it only covers one of the two tests for competency. B is also incorrect because it only covers one of the two tests for competency. D is incorrect because insanity is not required for an incompetency finding. Unlike competency, insanity looks at the status of the defendant at the time of the crime and not at the time of the trial.

Which of the following terms are most correctly used to refer to "actual cause?" I. But for cause II. Sine qua non III. Factual cause IV: Proximate cause a. I b. I and II c. I, II, and III d. I, II, III, and IV e. There is no terminology used to describe actual cause

c. I, II, and III Jurisdictions use different terminology to describe the first step of causation, the actual cause (see § 6.01).

Peyton is incarcerated in a prison serving a sentence of twenty years for raping Bryn. Peyton decides to send a letter to Bobbi, asking Bobbi to kill Bryn for giving incriminating trial testimony against Peyton, testimony that led to Payton's conviction. Peyton writes the letter to Bobbi and places it in the prison mail slot. The prison authorities find the letter in their, routine screening of mail leaving the prison facility. Peyton is charged with the crime of solicitation. Which of the following is most correct? a. In a Model Penal Code jurisdiction, Peyton cannot be convicted of solicitation because Bobbi never received the letter b. Peyton can only be convicted of obstruction of justice because solicitation is not a crime c. In a Model Penal Code jurisdiction, Peyton can be convicted of solicitation even though Bobbi never received the letter. d. Peyton can only be convicted of solicitation if Bobbi actually kills Bryn

c. In a Model Penal Code jurisdiction, Peyton can be convicted of solicitation even though Bobbi never received the letter The Model Penal Code in § 5.02(2) states: "(2) Uncommunicated Solicitation. It is immaterial under Subsection (1) of this Section that the actor fails to communicate with the person he solicits to commit a crime if his conduct was designed to effect such communication." Peyton's attempting to send the letter to Bobbi is an attempt to send a communication to kill and the fact that the prison authorities intercepted this letter does not negate Peyton's sending a letter to someone in order to get them to kill someone on their behalf (see § 12.00). A is incorrect because a Model Penal Code jurisdiction allows for uncommunicated solicitations to be a crime. B is incorrect because solicitation is a crime in many jurisdictions, although not all. D is incorrect because solicitation does not require completion of the crime.

Rich and Mike are high school seniors. They each recently viewed dozens of viral videos depicting people doing the popular "hot water challenge." In each video an unsuspecting person is approached from behind and hot water is poured over them as a prank. After the initial surprise and yelling, everyone laughs and the person subjected to the hot water challenge emerges from the prank unharmed. Rich decided to do the hot water challenge with Mike and arranged for several of their friends to be nearby to film the encounter. Rich boiled a pot of water before Mike arrived. Once the water reached the boiling point, Rich removed the pot from the heat and covered it to keep it warm. A few minutes later when Mike had his back turned, Rich dumped the entire pot of water over Mike's head. While Rich thought the water had sufficiently cooled down, the water was still boiling hot when he dumped it on Mike. Mike quickly went into shock and eventually died from complications caused by the severe burns all over his body. What is the most serious charge against Rich supported by these facts? a. First degree premeditated murder: Rich's act of boiling a pot of water (as opposed to using hot tap water) and having the boiling water ready before Mike arrived provides a factual basis to establish the elements of premeditation and deliberation b. Second degree murder: Rich was extremely reckless in not checking the temperature on the water. Dumping a pot of boiling hot water on Mike is sufficient evidence to support a finding of wanton reckless disregard for the risk of death to Mike c. Involuntary manslaughter: Rich was unreasonably unaware of the risk of death or serious bodily harm to Mike. His actions in dumping the water on Mike without first determining whether the water was still boiling hot was a gross deviation from a reasonable standard of care d. Felony murder: The severity of the burns Mike suffered from the boiling hot water constitute a felonious assault that lead to Mike's death

c. Involuntary manslaughter: Rich was unreasonably unaware of the risk of death or serious bodily harm to Mike. His actions in dumping the water on Mike without first determining whether the water was still boiling hot was a gross deviation from a reasonable standard of care There are sufficient facts to establish that Mike was unreasonably unaware of the fact that the water was still boiling hot when he poured it on Mike, and therefore his gross negligence would support involuntary manslaughter charge. A is wrong because there is no evidence that Rich premeditated and deliberated killing Mike or that he wanted Mike to die from the hot water challenge. B is wrong because there is no evidence that Rich was consciously aware (and disregarded) the fact that the water was still boiling hot when he dumped it on Mike. D is wrong because assaultive crimes (even when there are severe injuries) generally are not enumerated or "inherently dangerous" offenses for felony murder liability.

In 2005, the day after Hurricane Katrina devastated the city of New Orleans, scores of residents were trapped and dying in flooded areas. One resident, Jabar, broke into a locked bus depot, "commandeered" a public school bus, and gathered sixty vulnerable and stranded residents, including some infants abandoned on the streets, and drove thirteen hours on the bus to the Houston Astrodome. If Jabar faces charges of criminal trespass and larceny, which of the following would be most correct? a. The severe death and devastation caused by the flooding in the aftermath of Hurricane Katrina would be sufficient to support an affirmative defense of duress under the Model Penal Code b. Under the Model Penal Code, Jabar cannot mount a necessity defense unless he can show that death was imminent--and not just possible--for the people stranded by the flood waters c. Jabar has a strong necessity defense because the criminal conduct he engaged in is the lesser evil when weighed against the very real and imminent threat of death or serious bodily harm faced by himself and the other sixty people on the bus d. Jabar can assert a civil disobedience defense because his act of stealing the school bus was a protest of the poor evacuation planning by city officials and the delayed response from the Federal Emergency Management Agency (FEMA)

c. Jabar has a strong necessity defense because the criminal conduct he engaged in is the lesser evil when weighed against the very real and imminent threat of death or serious bodily harm faced by himself and the other sixty people on the bus This is a classic case of necessity where "physical forces beyond the actor's control rendered illegal conduct the lesser of two evils." Society values saving the lives of sixty people more than the harm caused the trespass and theft of the school bus. There is no duress defense on these facts because no person pressured or coerced Jabar to steal the bus. Likewise, the Model Penal Code does not impose the exacting standards of imminent danger expressed in B. D is wrong because the facts show that Jabar was motivated to act out of survival, not protest.

Jackie is arrested for the crime of burglary and taken to the local county jail. While being booked at the jail, it is discovered that Jackie has possession of an illegal drug, prohibited controlled substance. Jackie is charged with and convicted of possession of a controlled substance on the premises of a county jail. The statute reads, "No person shall knowingly deliver, attempt to deliver, have in his possession, deposit or conceal in or about the premises of any county jail or other county correction facility any controlled substances that has not been authorized by a written prescription of a licensed medical person." Which of the following is most correct? a. Jackie will be successful in arguing that there was no crime because there was no voluntary act b. Jackie will be successful in arguing that because Jackie did not deliver the drugs to another person, the conviction should be overturned c. Jackie will be unsuccessful in arguing that there was no voluntary act, and therefore no crime d. Jackie will have a potentially successful argument to claim on appeal that because they were brought to the jail involuntarily, there is no voluntary act

c. Jackie will be unsuccessful in arguing that there was no voluntary act, and therefore no crime C is correct because possession can be a voluntary act. A is incorrect because possession can be a voluntary act. B is incorrect because the statute allows for possession in addition to delivering or attempting to deliver the controlled substance. D is incorrect because, unlike the Martin case, the statute here specifies a county jail or other correction facility as an element of the crime. So, his not being at the jail voluntarily does not make a difference to his criminal culpability here.

Kerry and Jodie went on a date to a local tavern. They had a good number of drinks and Kerry became intoxicated and asked Jodie if they could go back to Jodie's place. At Jodie's home, Kerry found a kitchen knife and proceeded to stab himself. Jodie was shocked, but did nothing, including not calling for an ambulance or the police. It was clear that Kerry was going to die. If Jodie is charged with negligent manslaughter, what argumenis could the prosecutor make to support a conviction? a. Jodie was negligent in letting Kerry come to her home b. Jodie was negligent in having a kitchen knife in her home that could be found by someone who is drunk c. Jodie failed to seek assistance for Kerry d. There are no valid arguments that can be made as Jodie has no responsibility for Kerry

c. Jodie failed to seek assistance for Kerry In order to have an act by omission, it is necessary to have a duty to act. One of the ways to establish that duty is to assume the care of an individual and keep others from assisting. Since Jodie allowed Kerry to come to her home and failed to call 911 after Kerry stabbed himself, it can be said that she was not allowing others to provide assistance. D is not correct, because there can be a legal duty to care for the individual in this situation.

Kate and June are "porch pirates." They follow Amazon delivery trucks and take packages from homes immediately after delivery. Kate and June have hacked the Amazon delivery software and can determine the contents of the packages delivered to every address before delivery is made. Kate and June hit four homes on March 1st: (1) at 11:00AM they opened an unlocked screened porch door and took a box containing a 40-inch flat screen TV; (2) at 8:00PM they opened an unlocked screened porch door and took a package containing a laptop; (3) at 8:20PM they opened an unlocked screened porch door and took a DVD for the movie "Top Gun"; and (4) at 10:00PM they retrieved a package with two iPad tablets left on the front steps of house. Kate and June are charged with "four counts of common law burglary for making trespassory entry to dwellings to commit felony larceny of property valued over $500." Which of the following would be most correct? a. Kate and June committed four counts of common law burglary b. Kate and June committed two counts of common law burglary and two counts of larceny c. Kate and June committed one count of common law burglary and three counts of larceny d. Kate and June committed four counts of larceny but did not commit common law burglary.

c. Kate and June committed one count of common law burglary and three counts of larceny Common law burglary requires breaking and entering the dwelling of another in the nighttime with the intent to commit a felony. Only the 8:00PM theft of the 40-inch television set meets all of the elements of common law burglary because: the theft happened at night; entering property through an unlocked door constitutes a "breaking"; and there was an intent to commit a felony because they knew in advance that the targeted package contained a new laptop computer (valued over $500). The 11:00AM theft is not a burglary because it did not occur during "nighttime." The 8:20PM theft was not a burglary because a trespassory entry to steal a "Top Gun" DVD (valued under $500) is not a felony larceny. The 10:00PM theft of the iPads left on the front steps of the home did not involve a "breaking" or entry of a dwelling as required for common law burglary.

Kendall and Judy agree to rob a jewelry store. They drive over to their local mall and enter the closest jewelry store. Kendall points a gun at the store owner and asks them to fill up a bag with all the diamonds in the display case. Kendall and Judy then leave the store. As they are leaving, Judy sees a clothing stand inside the mall and says, "I'm going to get myself a new jacket." Judy proceeds to the stand and tells the stand's owner to give her one of the hanging jackets. While Judy is getting the jacket without paying for it, Kendall goes over to Starbucks to get a coffee. Upon leaving the mall, Kendall and Judy are arrested by the local police for the crimes of conspiracy to commit robbery of the jewelry store, robbery of the jewelry store, and theft under $100, for the jacket which was priced at $75 and which Judy obtained without payment. Kendall moves to dismiss the theft charge against her. Which of following is most correct? a. Kendall will be successful with the motion to dismiss the theft charge if the jurisdiction accepts Pinkerton's Rule b. Kendall will be unsuccessful with the motion to dismiss the theft charge because Pinkerton's Rule is accepted in all jurisdictions c. Kendall will be successful with the motion to dismiss the theft charge if the jurisdiction accepts Pinkerton's Rule, but only if Kendall can convince the court that the theft was not in furtherance of the robbery and was not reasonably foreseeable as an outgrowth of the conspiracy to rob the jewelry store

c. Kendall will be successful with the motion to dismiss the theft charge if the jurisdiction accepts Pinkerton's Rule, but only if Kendall can convince the court that the theft was not in furtherance of the robbery and was not reasonably foreseeable as an outgrowth of the conspiracy to rob the jewelry store The Supreme Court's Pinkerton case held that the overt act of one member of a conspiracy can be attributed to other members of the conspiracy. The test first requires that the government prove the existence of a conspiracy, and then that the substantive offense committed by one of the co-conspirators was committed in furtherance of the conspiracy, and that the substantive offense was reasonably foreseeable as a natural "outgrowth of the conspiracy" (see § 13.05). A is incorrect because if Pinkerton's Rule is accepted then this would provide liability to Kendall as opposed to excusing it. B is incorrect because although Pinkerton's Rule is accepted in the federal system, it is rejected in many states.

Laura and a group of friends decided to rent a barn on a remote farm for a sleepover and consume massive quantities of alcohol. In the middle of the night, Laura wandered away from the party and drove a tractor for several miles to the Smith farm. Upon seeing Laura on a trajectory towards his barn, Smith repeatedly yelled at Laura to stop, but Laura looked right at him and kept going. Smith then ran to in front of Laura to try to jump on the tractor and stop it, but Laura ran over him and kept going until she crashed into the barn and lost consciousness. Laura had a blood-alcohol level twice the legal limit. At the hospital, Laura had no memory of ever driving the tractor or crashing. Laura is charged with second degree murder (specific intent to kill). Which of the following would be most correct? a. Under the MPC, Laura will not be able to present an intoxication defense to the second degree murder charges. b. Based on Laura's level of intoxication, she will have a complete defense to the murder charge because she can show that she lacked "substantial capacity" to either appreciate the wrongfulness of her actions or confirm her conduct "to the requirements of law" c. Laura was so intoxicated that she will be able to present an intoxication defense to show that she could not form the criminal intent required to commit the charged offense d. Laura could only assert an intoxication defense if she has evidence that she was involuntarily intoxicated before she got on the tractor.

c. Laura was so intoxicated that she will be able to present an intoxication defense to show that she could not form the criminal intent required to commit the charged offense Voluntary intoxication is not a complete defense to criminal charges, but evidence of voluntary intoxication can be presented to show that Laura did not have the specific intent to kill, an element of second degree murder.

Stacey posted an online listing for the sale of her used laptop for $200. Lena offered to buy the laptop and sent a bogus email to Stacey that appeared to be confirmation from the online site that Lena had paid the $200. Stacey then delivered the laptop to Lena. The following day Stacey learned that the $200 was never deposited in her account, and the fake email she received did not come from the online site. Which of the following would be most correct? a. Lena committed the crime of larceny b. Lena committed the crime of false pretenses c. Lena committed the crime of larceny by trick d. Lena did not commit any property crime because Stacey voluntarily sent the laptop to Lena without verifying payment

c. Lena committed the crime of larceny by trick Stacey voluntarily gave possession of the laptop to Lena based on Lena's intentional false statement regarding payment contained in the bogus email. This is not larceny because Lena did not acquire the laptop by trespassory taking. Likewise, this is not false pretenses because Lena only acquired possession and not the title to the laptop.

Mary just started a new job and was scheduled to receive her first paycheck on Friday. Mary learned Friday morning that she would not be paid until the following Monday due to an administrative error by the company. Mary desperately needs to get medicine that she has not taken for four days, and she has no food to feed her three small children. Mary is new to town and has no friends or family to assist her. As Mary is about to leave the office on Friday, she runs into Ella, the office manager, at the elevator. Ella asks Mary if she could stay and watch her bags for a second while she runs back to her office for a forgotten item. Mary agrees and notices a thick envelope marked "Petty Cash for Deposit." Mary opens it and sees over $5000 in $100 bills. Mary takes one $100 bill and uses it to buy medicine and food. On Monday morning after Mary's paycheck was deposited, Mary immediately withdrew $100 and handed it to Ella. Mary explained her dire personal circumstances, confessed to taking the $100, produced receipts for the food and medicine she purchased, and begged Ella's forgiveness. Mary is arrested for larceny. Which of the following would be most correct? a. Under the Model Penal Code (MPC), Mary can assert a valid necessity defense because she reasonably believed her action in taking the $100 was necessary to avoid a greater harm to herself and her children b. Mary will be able to present evidence of her dire financial circumstances to support a duress defense because external circumstances beyond her control caused her delayed paycheck and put her under extreme pressure and duress c. Mary's dire financial circumstances will not support a common law necessity defense d. Mary can only mount a necessity defense if she can show that she was facing death or serious bodily harm if she did not take the $100

c. Mary's dire financial circumstances will not support a common law necessity defense As sympathetic as Mary circumstances are, courts have consistently held that financial need or "economic necessity" does not support a defense of necessity. Also, no person coerced or threatened Mary to commit larceny, so there is no defense of duress under these facts.

After football practice, Nester and his friends walked home from the local high school. As usual, they passed several food vendors in their urban neighborhood. One of Nester's friends, Cruise, proposed that they all push the rude hot dog vendor's cart over when they pass by him. Nester and the other boys laughed and agreed to Cruise's plan. When the boys were near the vendor, Cruise suggested that Nester just watch because Nester's arm was just injured during football practice. The boys executed their planned attack, and Nester stood about five feet away and loudly mocked the vendor by chanting, "No hotdogs, No peace!" Nester also cheered the boys on by stating, "Guys, squirt some mustard on him!" The vendor's cart was damaged and the vendor suffered severe physical injuries when the cart landed on his leg. The police rounded up all the boys, including Nester, and charged them with felony assault and destruction of property. Which of the following is correct under the Model Penal Code? a. Nester is not an accomplice because he did not engage in any criminal act during the attack on the vendor b. Nester is not an accomplice because he terminated his involvement before the attack (based on his injury) and refrained from committing any acts in furtherance of the attack on the vendor c. Nester is an accomplice because his conduct during the attack on the vendor was sufficient to render him criminally liable for the acts of the group d. None of the above

c. Nester is an accomplice because his conduct during the attack on the vendor was sufficient to render him criminally liable for the acts of the group Nester's conduct during the attack on the vendor constitutes acting "with the purpose of promoting or facilitating the commission of the offense." His words evince both his desire to have the crime succeed and his active coaching of and engagement with the criminal venture. Nester's words establish that he not an innocent bystander. A is incorrect because accomplice liability does not require each accomplice to commit some criminal act. Just as leaving a door unlocked is not, in isolation, a criminal act, such conduct can make a person an accomplice if the individual knows another person will enter the unlocked door to commit a crime. B is wrong because, although Nester did not push the cart on the vendor, he also did not take action to prevent the criminal actions of his cohorts or, as stated in Model Penal Code § 2.06(6)(c), "terminate his complicity prior to the commission of the offense."

Callie owns 50 acres of land. While on her property she pulls out a gun and randomly starts shooting. Callie thinks she is alone on this land and was not aware that Bronte had wandered off onto her property. As a result of Callie's conduct, Bronte is injured. The investigating officer tells the prosecutor that Callie's conduct appears to be a gross deviation from the standard of conduct that a law-abiding person would observe in Callie's situation. The investigating officer also states that she believes from questioning Callie that Callie did not have a conscious object to injure Bronte and that she was not aware that her conduct would result in injuring someone. If the prosecutor believes the investigating officer is correct, which of the following it the highest possible mens rea the prosecutor should use in bringing a charge in a Model Penal Code jurisdiction? a. Purposely b. Knowingly c. Recklessly d. Negligently e. The defendant should not be charged with any crime because this was an unforunate accident

c. Recklessly Under the Model Code § 2.02, one acts recklessly "when he consciously disregards a substantial and unjustifiable risk that the material element exists or will result from his conduct. The risk must be of such a nature and degree that, considering the nature and purpose of the actor's conduct and the circumstances known to him, its disregard involves a gross deviation from the standard of conduct that a law-abiding person would observe in the actor's situation" (see § 5.02). A is incorrect because there is no showing that Callie acted with a conscious object to injure Bronte, which is required for purposely. B is incorrect because there is no showing that Callie is "aware that his conduct is of that nature or that such circumstances exist; and if the element involves a result of his conduct, he is aware that it is practically certain that his conduct will cause such a result." D is incorrect because the question asks for the highest possible mens rea that can be used and recklessly is higher than negligently. E is incorrect because the conduct meets the mens rea under the Model Penal Code.

Nina and her six-month-old baby, Dylan, boarded a plane to visit her parents. Nina had a window seat and held Dylan on her lap. Ashley was seated next to Nina and rudely shoved Nina's arm off the shared armrest when she took her seat. A few minutes later when baby Dylan touched Ashley's arm, Ashley violently swatted the baby's hand away. Nina then instinctively elbowed Ashley and said: "He's a baby! Are you crazy?!" When Nina activated the call button to summon the flight attendant, Ashley abruptly stood up and shouted: "Let me show you what crazy looks like!" Ashley then leaned over and threw three forceful punches at Nina in rapid succession. The first punch struck Nina in the face, and the second punch hit the side of Nina's head. When Nina scrambled to try to get up and defend herself, the third punch hit baby Dylan and slammed his head against the window. Dylan died as a result of the head-injuries he suffered. What is the most serious criminal charge against Ashley supported by the facts? a. Voluntary manslaughter b. Felony murder c. Second degree murder d. First degree premeditated murder

c. Second degree murder Ashley knew Nina was holding an infant child and that Nina and baby Dylan were in a tight confined space in their seats on the plane. Notwithstanding that knowledge, Ashley repeatedly and violently struck Nina and put baby Dylan (who was inches away from each punch) at risk of death or serious bodily injury. These facts are sufficient to find that Ashley consciously disregarded a known risk of serious bodily injury or death to baby Dylan, or extreme recklessness. A is wrong because, although Ashley seems to have been enraged and acting in the heat of passion, there is no evidence of adequate provocation for a voluntary manslaughter charge. C is wrong because there is no "inherently dangerous" or enumerated felony to support felony murder. D is wrong because there is insufficient evidence of premeditation and deliberation.

Bob and Nick are college roommates. Bob is engaged to Maria, but Nick and Maria have started secretly dating. Bob is in the kitchen cutting up vegetables when Nick enters and bluntly informs Bob, "Look, me and Maria are together now. She does not want to be with you any longer. She and I are in love. You can move out if you want." Bob instantly lunged at Nick with the knife he was using and fatally stabbed Nick in the neck and chest while screaming, "I will kill you! I will kill you!" What is the most serious level of homicide supported by these facts? a. First degree premeditated murder b. Involuntary manslaughter c. Second degree murder, specific intent to kill d. First degree felony murder

c. Second degree murder, specific intent to kill Bob's words ("I will kill you") and his use of the knife are sufficient to establish that he acted with a specific intent to kill when he stabbed Nick to death. A is wrong because there is insufficient evidence that Bob premeditated or deliberated in advance of stabbing Nick. B is wrong because Bob's killing of Nick was intentional—not accidental or criminally negligent. D is wrong because assaultive crimes that lead to death generally fall outside of the scope of "inherently dangerous" or enumerated felony offenses.

When Angel looked through the text messages on her boyfriend Alec's cell phone, she came across several text messages from her cousin Connie regarding her sexual liaisons with Alec. Angel, in a fit of rage, drove two hours to Connie's house to confront her. When Connie was not at home, Angel waited outside of Connie's apartment for several hours. When Connie arrived, Angel confronted her, and an argument ensued. Connie tried to walk away, but Angel pointed a knife at Connie and said, "I am going to slash your face up so bad that Alec will not recognize you or want you." Angel lunged at Connie and tried to cut her face. During the struggle for control of the knife, Connie was stabbed in the abdomen and died. What is the most serious level of homicide supported by these facts? a. First degree premeditated murder b. Second degree murder, extreme recklessness c. Second degree murder, specific intent to kill/injure d. Voluntary manslaughter

c. Second degree murder, specific intent to kill/injure Angel confronted Connie with the specific intent to violently assault Connie (slash her face) or inflict serious bodily harm. Angel's stated intention was for Connie to spend her life disfigured and unattractive to Alec. Angel did not have a specific intent to kill. Thus, Angel's specific intent to cause serious bodily harm supports the charge of second degree murder. A is wrong because while there is ample evidence to support premeditation and deliberation (i.e., driving to her house, waiting for her, arming herself prior to the confrontation), there is no evidence that she had a specific intent to kill Connie. B is wrong because there is no evidence that Connie was killed because Angel was reckless or disregarded a risk of death; Angel assaulted Connie intentionally. D is wrong because, although there are facts that support provocation and heat of passion, voluntary manslaughter is not the most serious homicide charge supported by these facts.

Officer Green was on routine patrol when he saw Lee, age 17, walk up behind a woman at the ATM machine, reach around her, and snatch the money she had just withdrawn. Officer Green gave chase and, at gunpoint, ordered Lee to stop. Instead, Lee ran into a building and Officer Green pursued him. Officer Green shot Lee in the leg, severely wounding him, when Lee again refused to stop. Officer Green's police report states: "I did not believe this kid had a weapon or planned to hurt anybody, but he repeatedly refused to comply with my lawful commands. After chasing him for several blocks I shot and wounded him in order to lawfully apprehend him, but I did not shoot to kill." Which of the following is accurate? a. While Officer Green could not use deadly force, firing one non-lethal shot to apprehend Lee was justified under the circumstances. b. Given that Officer Green witnessed Lee commit a robbery and Lee repeatedly resisted arrest, lethal and non-lethal force to apprehend Lee was justified c. Shooting Lee under these circumstances was not a justified use of force d. Both A and B are correct

c. Shooting Lee under these circumstances was not a justified use of force Officer Green deployed deadly force when he shot Lee to apprehend him. If a fleeing suspect poses no threat of danger to the officer or to the public—a fact conceded by Officer Green—the Supreme Court ruled in Tennessee v. Garner that a police officer cannot use deadly force to apprehend the suspect. A is incorrect because shooting someone—regardless of whether the person is killed—is the use of lethal force. C is wrong because it focuses on Lee's criminal conduct, and not whether the unarmed suspect posed a threat to the safety of the officer or the public.

Which of the following crimes requires the least amount of proof by the prosecutor? a. Conspiracy b. Attempt c. Solicitation

c. Solicitation Unlike attempt and conspiracy, no overt act or agreement is required for solicitation. A is incorrect because conspiracy requires an agreement to commit the unlawful act. With some statutes, it also requires proof of an overt act. B is incorrect because although solicitation is similar to a criminal attempt as it is a preliminary step toward the completion of a crime that usually does not come to fruition, solicitation does not require an overt act.

Which of the following punishment theories is focused on making certain that a convicted defendant will refrain from additional criminal conduct because that particular defendant fears additional punishment? a. Rehabilitation b. General Deterrence c. Specific Deterrence d. Retribution e. Isolation

c. Specific Deterrence

The defendant raises "outrageous government conduct" as a defense to his committing the act charged by the government. Which of the following is most correct? a. The court should evaluate this defense using a subjective approach b. The court should evaluate this defense looking first subjectively at the defendant's conduct and only if the defendant was predisposed to commit the crime should the court next examine the conduct of the government officers c. The court should evaluate this defense using an objective approach d. Outrageous government conduct is not a real defense and the defendant can never raise this as a defense

c. The court should evaluate this defense using an objective approach Outrageous government conduct can differ from an entrapment defense. It looks objectively at the government conduct as opposed to whether the accused acted with a criminal predisposition to commit the crime. It is premised on due process. A is not correct because it does not use a subjective approach as is often used with a defense of entrapment. B is not correct as it presents an entrapment defense as opposed to an argument premised on outrageous government conduct. D is not correct because, although an outrageous government conduct defense is a difficult one for the accused to prove, it is a legitimate argument.

Which of the following is an accurate statement regarding an arraignment proceeding in a criminal case? a. Jury selection occurs b. The prosecutor must disclose specific evidence to the defense c. The defendant enters a plea of "guilty" or "not guilty" to the charges in the indictment d. The grand jury begins hearing evidence of the defendant's guilt e. The trial judge makes the initial decision regarding bail

c. The defendant enters a plea of "guilty" or "not guilty" to the charges in the indictment The purpose of the arraignment is to give the defendant formal notice of the charges and allow the defense to inform the court whether the charges will be contested

Campbell is indicted for trespassing on federal property for snowmobiling in a federal park without permission on April 1, 2019. On January 15, 2020, Congress enacted a law that specifies that snowmobiling on federal property can constitute trespassing. If Campbell files a motion to dismiss the indictment, which argument will be the most successful one with the court? a. The indictment should be dismissed because it violates the rule of lenity b. The indictment should be dismissed because it violates the due process clause c. The indictment should be dismissed because it violates the ex post facto clause d. The indictment should be dismissed because it is based on an overbroad statute e. The indictment should be dismissed because of the vagueness doctrine

c. The indictment should be dismissed because it violates the ex post facto clause A is incorrect because this is not an instance of interpreting a statute narrowly to favor the accused. B is incorrect because although one could argue that this is a due process violation, there is a better answer found in C, as this prosecution directly confronts the ex post facto clause of the constitution because the statute did not exist at the time of the alleged criminal act

Bobby broke into R.J.'s house to steal a computer, and he now faces felony burglary charges. The formal criminal charges against Bobby were most likely filed by: a. The trial judge who presides over the case b. The police officer who made the arrest c. The prosecutor d. R.J., the victim e. All of the above

c. The prosecutor Generally the prosecutor has the exclusive power to decide whether to initiate formal charges, especially felony charges

Oscar, a state legislator in the State of Erehwon, is found by a majority of the state legislature to have violated a state statute that reads: "any state legislator who violates a state statute shall be fined or imprisoned, or both, and removed from office." Oscar's best argument to challenge the fifteen-day imprisonment term issued by the state legislature would be which of the following? a. The statute is overbroad b. The state is void for vagueness c. The state statute violates the bill of attainder d. The state statute violates the ex-post facto clause

c. The state statute violates the bill of attainder This is special legislation that declares a person guilty of a crime and subjects the person to punishment without the benefit of a trial or conviction

A statute reads as follows: "Whoever, willfully derails, disables, or wrecks any train, engine, or car, used or operated in interstate or foreign commerce is guilty of train wrecking." If interpreting this statute in a Model Penal Code jurisdiction, which of the following would be most correct? a. The statute requires that the defendant have acted negligently b. The statute requires that the defendant have acted recklessly c. The statute requires that the defendant have acted knowingly d. The statute does not require an intent as this is a strict liability crime

c. The statute requires that the defendant have acted knowingly In the Model Penal Code, willfully is equivalent to knowingly. A is not correct because this is the lowest level of mens rea in the Model Penal Code. To be convicted of the crime, one could not have a mens rea level lower than stated in the statute. B is not correct because this is a lesser level of mens rea in the Model Penal Code than knowingly. To be convicted of the crime, one could not have a mens rea level lower than stated in the statute. D is not correct because the statute requires a mens rea, and therefore is not strict liability

Helen and her husband Don were in the midst of a nasty divorce. When Don began physical abuse, Helen moved out and began staying with her co-worker, Granger. A few days later, Granger agreed to drive Helen back to her house to collect some her things. Helen informed Granger that Don would not be at work and she was in no danger. Upon arrival at the house, Helen insisted that Granger stay in the car and let her know if Don unexpectedly came home. After several minutes, Helen swiftly emerged from the house with some of her belongings. After Granger drove Helen back to his house, Helen disclosed to Granger that she knew Don was going to be at home when she arrived and that she planned to arrive when she knew Don would be exhausted and sound asleep. Helen also informed Granger that she stabbed Don to death before she left the house. In response, Granger said, "Good! I'm glad you did it. Is there anything I can do to help you?" Before Helen could respond, the police arrived and charged both Helen and Granger with murder. Which of the following is correct? a. Granger can be charged with murder as an accomplice to Helen b. Granger was an accessory after the fact because he drove Helen away from the scene of the crime c. There are insufficient facts to support charging Granger as an accomplice d. Granger would be deemed an accessory after the fact because, upon learning that Helen killed Don, Granger offered his assistance to Helen

c. There are insufficient facts to support charging Granger as an accomplice At common law and under the Model Penal Code, accomplice liability requires a specific intent to aid in the commission of the offense. Here, due to Helen's deception, Granger was unaware that Don was in the house and unaware that Helen planned to kill Don. Thus, there are no facts to support a finding that Granger formed the specific intent to assist Helen with Don's murder. In addition to the absence of requisite mens rea, Choice A is incorrect because Granger did not engage in any culpable conduct to aid Helen prior to the commission of the crime. When Granger agreed to give Helen a ride home he did not know that Helen planned to kill Don. B is wrong because Granger's ignorance of the commission of the crime by Helen until after he drove Helen all the way back to his house precludes a finding that he either "drove the getaway car" or helped Helen escape. D is wrong because although Granger was in favor of Helen killing Don and offered to help her after he learned that she killed Don, Granger never had an opportunity to perform any act of support before his arrest that would make him culpable as an accessory after the fact.

A statute is unconstitutional when "men [or women] of common intelligence must necessarily guess at its meaning and differ as to its application." Which of the following is most correct? a. This is because of the ex post facto provision in the Constitution b. This is because of the bill of attainder provision in the Constitution c. This is because of the vagueness doctrine d. This is because of the rule of lenity e. There is nothing that can ever be unconstitutional about a statute

c. This is because of the vagueness doctrine A is incorrect as the ex post facto clause of the Constitution pertains to it being unconstitutional to charge a person with a crime that had not been enacted at the time of the criminal act. B is incorrect because a bill of attainder pertains to legislation that declares someone guilty of a crime without either a trial or conviction. D is incorrect as the rule of lenity pertains to statutes that might have more than one constitutional interpretation and provides that courts should interpret the statute narrowly in favor of the defendant. E is incorrect because statutes can be held unconstitutional.

Robin, a duly authorized licensee in Alabama Alcoholic Beverage Control Board, is charged with violating an Alabama statute for selling alcohol to individuals who were under the legal age in the state of Alabama. The statute provides that it is a misdemeanor "[for any licensee or the board [Alcoholic Beverage Control Board] either directly or by the servants, agents, or employees of the same, or for any servant, agent, or employee of the same to sell, deliver, furnish, or give away alcoholic beverages to any minor." Which of the following is most correct? a. This statute requires that Robin have a mens rea as this is not a strict liability offense b. Robin cannot be convicted of violating this statute because of the Pinkerton Rule c. This statute does not require that Robin have a mens rea as this is a strict liability offense d. Robin cannot be convicted of violating this statute because of Wharton's Rule e. It is impossible to determine whether this statute is strict liability or not as we don't know the actual sentence that Robin received for violation of this statute.

c. This statute does not require that Robin have a mens rea as this is a strict liability offense Although strict liability crimes are generally disfavored, a misdemeanor statute that fails to include a mens rea term and is regulatory in nature is likely to not require a mens rea (see § 5.06). A is not correct as it is unlikely that this offense will require the prosecutor to show that the accused had the intent to commit this offense. B is not correct because while a sentence received can be a factor in analyzing whether a statute requires a mens rea, it is not typically the sole determination. D is incorrect because the Pinkerton Rule relates to criminal liability in federal conspiracy offenses, which is not involved in this case. E is incorrect because Wharton's Rule relates to the impossibility of having a conspiracy charge because the crime itself requires a set number of people, which is not involved in this instance.

The gambling statute in the State of Erewhon provides that if five or more persons conduct, supervise, or participate in gambling activity, including playing poker for money, they are guilty of gambling, a misdemeanor. Sarah plays poker with four friends and wins one hundred dollars ($100.00) at the poker game. Two of the four friends she was playing with report Sarah to the local police who arrest her and charge her with the crime of conspiracy to commit gambling. Which of the following might assist Sarah most in her defense? a. The Independent Felony Rule b. Corroboration Rule c. Wharton's Rule d. The Ireland Rule e. Pinkerton's Rule

c. Wharton's Rule When the crime by its very nature requires a set number of people, in this case five, courts have precluded use of conspiracy or made it a presumption that can be used when you have that set number of individuals. Because this gambling statute speaks to five individuals and Sarah is there with four others, charging conspiracy may be prohibited by Wharton's Rule. A is incorrect because the Independent Felony Rule pertains to felony murder and does not relate to conspiracy. E is incorrect as Pinkerton's Rule pertains to a substantive offense occurring in the course of and foreseeable to the conspiracy even though the co-conspirator may not have committed that act. B is not correct as the Corroboration Rule comes up in cases where the parties are the only two witnesses against each other. D is incorrect because the Ireland Rule pertains to felony murder.

Moses and Remy are both charged with conspiracy to commit dueling. The statute for dueling reads, "Dueling is an engagement in combat between two individuals using swords." Which of the following might assist Moses in his defense? a. The Independent Felony Rule b. Corroboration Rule c. Wharton's Rule d. The Ireland Rule e. Pinkerton's Rule

c. Wharton's Rule When the crime by its very nature requires two people, courts have precluded use of conspiracy. Because the dueling statute specifies two people, charging conspiracy would be prohibited by Wharton's Rule. A is incorrect because the Independent Felony Rule pertains to felony murder and does not relate to conspiracy. E is incorrect as Pinkerton's Rule pertains to when you can charge a substantive offense occurring in the course of and foreseeable to the conspiracy even though the co-conspirator may not have committed that act. B is not correct as the Corroboration Rule comes up in cases where the parties are the only two witnesses against each other. D is incorrect because the Ireland Rule pertains to felony murder.

At common law, in order to have sufficient causation it is necessary that the victim die when? a. Within a year of the perpetrator's act b. Within five years of the perpetrator's act c. Within a year and a day of the perpetrator's act d. Within thirty days of the perpetrator's act

c. Within a year and a day of the perpetrator's act At common law, a defendant could not be charged with a homicide if the victim did not die within a year and a day of the wrongful conduct causing the death. States for the most part have abolished the "year and a day rule," finding in some instances that it was an arbitrary line. Medical expertise allows a more definitive ability to determine the causation between the act and the death (see § 6.02).

Which of the following is the best answer to this question? "The Model Penal Code allows for strict liability when:" a. The crime is considered a violation under the Model Penal Code b. The crime has no jail time c. The crime has a jail time of less than 30 days d. A and B e. A, B, and C

d. A and B The Model Penal Code §2.05 does not favor strict liability and only allows it for violations. Violations, defined in Model Penal Code § 1.04(5), are only allowed for "an offense defined by this Code or by any other statute of this State constitutes a violation if it is so designated in this Code or in the law defining the offense or if no other sentence than a fine, or fine and forfeiture or other civil penalty is authorized upon conviction or if it is defined by a statute other than this Code that now provides that the offense shall not constitute a crime" (see § 5.06). A and B are therefore correct, but since both are correct they are not the best answer. C is not correct because the Model Penal Code does approve of absolute or strict liability when jail time is provided. E is incorrect because although A and B are correct, including C makes the answer incorrect.

If the accused wants to raise that he or she was insane at the time of the crime, which of the following is most correct? a. A State may place the burden of proof of insanity on the defense as an affirmative defense b. A State may have a guilty but mentally ill verdict used in the trial c. A State will always be required to prove that the accused was not mentally ill d. A and B above e. None of the above

d. A and B above States vary on the burden of proof required in insanity cases, with many making insanity an affirmative defense. Several states have moved away from the verdict of "not guilty by reason of insanity" to using a verdict of "guilty but mentally ill." A is a correct statement, but it is not the best answer because B is also correct. B is a correct statement, but it is not the best answer because A is also correct. C is an incorrect statement because many states have moved away from requiring the burden of proof of insanity to be placed on the prosecution and instead have allowed the burden to rest with the defense as an affirmative defense. Since the answer uses the word "required" it is incorrect. E is not correct because A, B, and D all are correct statements.

Denise was walking down the street to go to the corner store when she was accosted by a strange man who blocked her path, called her insulting names, and made sexual comments about her large breasts. Denise ignored him and tried to walk around him, but the man aggressively pursued her. Finally, frustrated and upset, Denise turned to the man and said, "Wait right here!" Denise went back into her house, retrieved a knife and came back outside to confront the man. Denise pointed the knife at the man and said, "I'm warning you. You better back off and leave me alone." The man continued to follow Denise and make verbal insults. When Denise walked faster to get away from the man, he grabbed her coat and said, "I will beat you down if you ever walk away while I am talking to you." When Denise was finally able to break free of the man's grip, she angrily yelled, "I WARNED YOU!" Denise then forcefully plunged the knife into the man in the chest. The man died instantly. Denise is charged with first degree murder. Which of the following facts would support a charge of first degree murder? a. The fact that Denise left the area to get a weapon and then returned to the confrontation b. Denise making the statement "I warned you" immediately prior to stabbing the man c. The fact that Denise threatened the man with the knife before stabbing him d. All of the above

d. All of the above . A (leaving and returning with a knife) is a classic fact used to show that the defendant planned in advance to kill by gathering the means to carry out the crime and returning to confront the soon-to-be victim. C (warning the decedent at knifepoint to leave her alone) is further evidence that Denise considered whether (and under what circumstances) she would use deadly force and did not make a "spur of the moment" rash decision to stab him. B ("I warned you") further shows that Denise thought about killing the man and formed a definite decision to kill him before she stabbed him.

Which of the following WOULD support a complete defense to a charge of murder? a. "While in the grocery store, I saw the man who broke into my apartment and raped me several months ago. He smiled and winked at me. I walked away, but he followed me around the store. When I tried to run out of the store, he grabbed me and dragged me into the back storage area and started to rip off my clothes. I screamed and fought back as hard as I could. I managed to grab a fire extinguisher mounted on the wall. I used the fire extinguisher to bash him over the head until he was unconscious. I didn't mean to kill him, only stop him from sexually assaulting me. I was afraid and felt I had no other choice." b. "When I came home I encountered my wife with a loaded gun pointed directly at my teenage son's head. My son was on his knees looking at his step-mother and pleading for his life. I repeatedly told my wife to put the gun down. She was hysterical and said that my son broke into the house and physically assaulted her again. She was injured and crying. She turned to me and soberly said: You have to choose_-him or me.' When I tried to reason with her she yelled: 'CHOOSE NOW!!' I was under pressure, I had to think fast, so I pulled out my gun and shot her to save our son's life. I had to make a choice." c. "I've had enough. My husband has beat, tortured, and humiliated me for 7 years. I simply could not take it anymore. I was too afraid of him to leave him, and I had nowhere to go. I filed numerous criminal complaints against him, but he always got out of it. Tonight, he came home drunk and grabbed the strap that he always uses to beat me. When he came charging at me, I was afraid I would end up in the emergency room again. So, I grabbed a knife from the counter and plunged it into his chest." d. All of the above

d. All of the above A is correct because the defendant is facing an imminent threat of serious bodily harm, she is subjectively afraid, her fear is objectively reasonable (based on the current and past sexual assaults by the decedent), and she used proportionate force to repel the violent assault. Moreover, the Model Penal Code (§ 3.04(2)(b)) specifically recognizes the use of deadly force as justifiable to protect one from "sexual intercourse compelled by force or threat." B is correct because the defense of others justifies deadly force when the decedent was threatening to use imminent deadly force against another and has the means to do so. C is correct because the drunk decedent was charging at her with the strap he has used to assault her in the past. While he probably could not kill her by striking her with the strap, the right to use deadly force in self-defense exists for those facing a threat of death or a threat of serious bodily harm—the kind of injuries that would send a person to the emergency room.

Which of the following have not been prohibited by the Supreme Court? a. The imposition of a death penalty from the rape of a child where the crime did not result, and was not intended to result, in the death of a victim b. The imposition to a juvenile of a sentence of life without the possibility of parole in a non-homicide case c. The imposition of a mandatory life sentence to those under the age of 18 at the time of the crime d. All of the above have been prohibited by the Supreme Court e. None of the above have been prohibited by the Supreme Court

d. All of the above A is incorrect because the Supreme Court in Kennedy v. Louisiana held that the Eighth Amendment prohibited the imposition of a death penalty "for the rape of a child where the crime did not result, and was not intended to result, in the death of the victim." B is incorrect because in Graham v. Florida the Supreme Court held that it would violate the Eighth Amendment to give a juvenile offender a sentence of life without the possibility of parole in a non-homicide case. C is incorrect because in Miller v. Alabama, the Court held that the Eighth Amendment prohibited giving a mandatory life sentence to those under the age of 18 at the time of the crime.

Bob is facing felony charges for assaulting Frank in a bar fight. Bob claims that Frank started the fight and that he was simply trying to defend himself against Frank's violent attack. Linda saw the fight and support's Bob's version of the events, but refuses to come to court to testify because she "does not want to get involved." Jimmy also saw the fight and will testify that the fight started when Bob threw the first punch, and Frank was just peacefully having a drink and minding his own business. Which of the following statements is true? a. Bob's attorney can subpoena Linda and require her to testify at trial b. Bob has the right to be represented by counsel at trial c. At trial, the defense will be able to cross-examine Jimmy about his account of the events d. All of the above

d. All of the above The Sixth Amendment right to "compulsory process" gives Bob the constitutional right to compel Linda to testify at trial as part of the defense (A). Likewise, if the prosecution presents Jimmy's testimony at trial, Bob would have the right to "be confronted with the witnesses against him," which includes the right to cross-examine Jimmy and challenge his version of the fight (C). Finally, the Sixth Amendment guarantees that Bob would have an attorney or "the assistance of counsel, to present his defense at trial" (B).

Which of the following are considered aggravating factors that can be used in rendering a defendant convicted of murder eligible for capital punishment? a. The murder of a peace officer killed in his performance of their official duties when done to prevent or retaliate for that performance b. The murder of any person (including but not limited to inmates, staff, visitors) occurring at a correctional facility c. The intentional murder of a person involving the infliction of torture d. All of the above

d. All of the above These are three of the aggravating factors coming from the Constitution Project's Death Penalty Initiative. Specific state statutes may offer different aggravating and mitigating circumstances for whether to impose a sentence of death.

Bob hates Dave for sexting his girlfriend Mandy. Bob, Dave, and several other friends got together for the "cinnamon challenge," a viral stunt whereby friends try to consume a tablespoon of cinnamon. Bob agreed to give the cinnamon to each challenger and film their efforts to complete the stunt. When it was Dave's turn, Bob gave Dave cinnamon laced with poison. Just as he accepted the spoon of cinnamon from Bob, Dave looked at his phone on the counter and saw a text from Mandy which stated: "Bob knows. Watch out!" Dave pretended to accidentally trip and spill the cinnamon. Under which of the following standards would Bob's conduct constitute criminal attempt? a. "Probable desistance" b. "Substantial step" c. "Dangerous proximity" d. All of the above

d. All of the above When Bob handed Dave the poison for him to consume he committed the crime of criminal attempt under the common law standards in A and C, as well as the Model Penal Code standard in B.

Andi works for a food delivery service, and she was assigned to deliver Ida's groceries to Ida's house. Ida's groceries were packed and sealed in thermal cooling bags and placed in the trunk of Andi's car. En route to Ida's house, Andi opened the bags and removed a third of the grapes, two of the bananas, and three slices of bread from each of the two bags in Ida's order. Unbeknownst to Andi, there was a theft detection device on Ida's bags that alerted when the bags were opened before being deactivated by Ida's phone. Which of the following would be most correct? a. Andi had not committed a larceny at that point because there was no asportation b. Andi has committed the crime of false pretenses c. Andi has committed the crime of larceny by trick d. Andi has committed the crime of common law larceny

d. Andi has committed the crime of common law larceny Andi's actions constituted a larceny under the common law "breaking bulk" doctrine because Andi was given possession of the bags, but not the contents of the bags. Thus, her actions in removing portions of the groceries from the bags constituted trespassory taking and asportation. Andi has not committed the crimes of larceny by trick or false pretenses because she did not obtain voluntary possession of the bags by fraud or misrepresentation.

Which is not an inchoate crime? a. Criminal attempt b. Conspiracy to commit murder c. Criminal solicitation of murder d. Assault with intent to commit murder

d. Assault with intent to commit murder Assault is a completed crime that is more serious if, in addition to the assault, the individual intended to kill the victim. Thus, this would not be an inchoate crime. A, B, and C are all examples of inchoate crimes.

Angelo decided to try ecstasy for the first time at his friend's party. Angelo did not experience any immediate effects from the drug. After a couple of hours Angelo leaves the party to walk home. The next morning Angelo awoke to find himself handcuffed to a hospital bed, and a police officer informed him that he was charged with statutory rape. Angelo was told that Kindi, a 13-year-old runaway, agreed to have sex with him in exchange for food and shelter. When Kindi could not get Angelo to wake up she thought he was dead and called 911 in a panic. Angelo does not remember anything that happened after he left the party. Which of the following would be most correct? a. Angelo can assert an intoxication defense based on his impairment due to consumption of ecstasy b. Angelo will be able to present an intoxication defense to show that he lacked the specific intent to engage in sex with a minor c. Angelo knowingly introduced the ecstasy intoxicant into his body and cannot assert a valid intoxication defense under the MPC d. Because specific intent is not relevant, Angelo will not be able to assert a valid intoxication defense

d. Because specific intent is not relevant, Angelo will not be able to assert a valid intoxication defense Because statutory rape is a strict liability crime, evidence of voluntary intoxication is immaterial because there is no specific intent requirement for the crime.

Bob works as an Amazon delivery driver. Over the last five months, Bob has falsely reported to Amazon that he delivered items to particular residences, but he actually kept the expensive electronics equipment for himself. Bob's scheme was exposed when several Ring security cameras at customer homes captured Bob driving to a residence, walking towards the home with packages, and then going back to his truck without leaving any packages. His scheme defeated measures taken by Amazon to monitor his deliveries through GPS tracking of his movements on his route. Which of the following would be most correct? a. Bob has committed the crime of larceny b. Bob has committed the crime of burglary c. Bob has committed the crime of larceny by trick d. Bob has committed the crime of embezzlement

d. Bob has committed the crime of embezzlement Bob is embezzling property that his employer voluntarily entrusted to him for delivery. Bob's action would not constitute common law larceny because he did not acquire possession of the goods through a trespassory taking. Likewise, his actions do not fit the elements of larceny by trick or false pretenses because he did not gain possession or title to the goods as a result of a knowingly false misrepresentation. Bob also did not commit a burglary because he approached the homes, but never made a trespassory entry.

Defendant Gary stole two golf clubs from a pro shop in California, a state that has a three-strikes statute. If convicted, this would be Gary's third felony conviction. Which of the following is most correct? a. It would be proper in evaluating this case to consider whether the sentence was proportional b. Both the prosecutor and the judge have the discretion to decide whether the crime is a wobbler c. Only the judge has the discretion to decide whether the crime is a wobbler d. Both A and B e. Gary is not subject to the three strikes rule because he did not steal three golf clubs

d. Both A and B This question is based on Ewing v. California (see § 2.05). The court uses proportionality in looking at California's three-strike law. Both the prosecutor and judge can make the offense a wobbler, which makes it either a felony or misdemeanor. If it becomes a felony, then it is subject to a three-strikes sentence. E is not correct, as the number of golf clubs taken is not important if the activity could still be a felony.

Jake commits a homicide in State A, in the County of X. Which of the following is most correct? a. The prosecution's proof that the homicide occurred in State A is required to show that they have the jurisdiction to proceed on this case b. Unless a court grants a change of venue, the prosecution must proceed with this case in County X in order to meet its venue requirement c. It is only necessary for the prosecution to prove jurisdiction, venue does not have to be proven d. Both A and B above

d. Both A and B above Jurisdiction is the power of a court to hear a case and render a valid decision, while venue concerns the place where a case may be filed or tried.

Same facts as in Question 18. At the conclusion of the evidence in Denise's first degree premeditated murder trial, the defense asks the trial judge to instruct the jury on lesser-included offenses. Which lesser-included offense instructions would Denise be entitled to receive based on the facts presented? a. An involuntary manslaughter instruction b. A second degree murder instruction c. A voluntary manslaughter instruction d. Both B and C

d. Both B and C While Denise's conduct might support a first degree murder charge, C is correct because a jury could find that Denise's act of arming herself and returning to threaten the man with the knife was done out of fear and self-preservation, and not indicative of premeditation and deliberation to kill. B is correct because the decedent's actions in physically grabbing her and threatening to harm her would constitute adequate provocation, and Denise was enraged when she stabbed the man. A is incorrect because the facts do not support a finding that Denise unintentionally stabbed the man.

Trina, a teenage girl, drove up to Marcy's house around midnight and took Marcy's bike from the driveway, just outside Marcy's front door. Marcy repeatedly screamed and ordered the girl to put the bike back, but Trina laughed and proceeded to place the bike in the trunk of the car. Frustrated, Marcy grabbed her gun fatally shot Trina as she attempted to enter the car. Marcy is charged with murder. Marcy maintains that her use of force is justified in defense of property. Which of the following is accurate under the Model Penal Code? a. Marcy was justified in using deadly force to prevent a theft on her property after she warned Trina to stop b. Marcy would have been justified in punching and kicking Trina to get the . bike back from her after repeatedly ordering Trina to stop stealing the bike c. Under these circumstances, Marcy could not use deadly force to prevent the bike theft d. Both B and C are correct

d. Both B and C are correct Under the Model Penal Code (§ 3.06(1)(a)) Marcy could use non-deadly force (punching, kicking) to prevent the theft of property, but she could only use deadly force to prevent the theft of property if the perpetrator threatened deadly force or the use on non-deadly force posed a risk of danger or serious bodily harm.

Nina worked in the shipping and receiving department for a major electronics company. Over a six-month period, Nina falsely reported to the company that several items in the inventory were damaged and could not be repaired. Based on Nina's representations, she was instructed to scrap the items. In fact, Nina knew these items were not damaged and she shared the items with family and friends. In which scenario below has the recipient of electronics Nina wrongly obtained from the company committed the crime of receiving stolen property? a. Nina to Debbie: "My company allows me to buy damaged electronics that they don't want to spend the money to repair. My cousin fixes them like new. I will sell you one of the refurbished smart watches for $300." Debbie bought the smart watch. b. Nina to Amanda: "For $100, I can get you two new 60-inch flat screen televisions that I got from my job. They will not come in a box, there will be no receipt, and you cannot tell anyone that you got them from me. You give me the money, and I will tell you the location of the pick-up." Amanda bought the TVs. c. Nina to Rico: "I am running a scam at work. I file false reports that electronics are damaged and then I sell the goods on the side for a profit. Do you want to buy these blue tooth headphones for $25." Rico buys two sets of headphones. d. Both B and C present scenarios wherein the recipient of the electronics has committed the crime of receiving stolen property.

d. Both B and C present scenarios wherein the recipient of the electronics has committed the crime of receiving stolen property. The crime of receiving stolen property requires the person in possession of the stolen goods to either know the items are stolen or have reasonable cause to believe the items are stolen. In C, Rico agrees to buy goods that he is expressly told are stolen. In B, the incredibly low price and the suspect circumstances surrounding the delivery would give a person reason to believe that the goods are stolen, even though Nina does not expressly tell Amanda that the televisions are stolen. A does not involve the purchaser having any actual knowledge of Nina's theft or having any facts that would give the purchaser a reasonable belief that the items are stolen.

Which of the following causation rules provide a basis for finding criminal liability as opposed to being a basis for not extending criminal liability? a. De minimus b. Obstructed cause c. Independent intervening cause d. Dependent intervening cause

d. Dependent intervening cause A dependent intervening cause ordinarily involves a reasonably foreseeable consequence of an act by the defendant, and therefore it does not relieve the defendant of the resulting responsibility. A is incorrect as causes that are de minimus, very minor, or remote do not satisfy the causation element. B is incorrect because an obstructed cause keeps the initial perpetrator from being held accountable for the criminal act because of some event that obstructs. C is incorrect because an independent intervening cause keeps the perpetrator from being held accountable.

Mary works as a nurse a Mercy Hospital. Kala is a patient in the hospital under Mary's care. About 10 minutes before Mary was scheduled to administer medication to Kala and other patients, Mary learned that her mother was diagnosed with a terminal illness. Because Mary was very upset and distracted when she gathered the medication for the patients, Mary mistakenly grabbed the wrong dosage of medication for Kala. Unaware of her error, Mary entered Kala's room and inserted the needle of medication into Kala's IV. Just as Mary was about to push the plunger to inject the medication, a nurse ran into the room and informed Mary of her error. If Mary had given Kala the medication, Kala would have died of an overdose. Kala called the police and demanded that Mary be arrested for criminal attempt. Which of the following is correct? a. Mary committed criminal attempt at the point that she retrieved the overdose of medication and entered Kala's room  b. Mary committed criminal attempt because she had the specific intent to give Kala the medication and engaged in conduct that constitutes a substantial step towards killing Kala c. Mary committed criminal attempt at the point that she inserted the needle with the overdose amount of medication into Kala's IV d. Despite Mary's substantial step towards killing Kala, Mary has not committed the crime of attempted murder

d. Despite Mary's substantial step towards killing Kala, Mary has not committed the crime of attempted murder Attempt is a specific intent crime. There are no facts to support a finding that Mary wanted to kill Kala or formed a specific intent to kill Kala. Mary is distracted by her own personal circumstances and acting in a manner that is criminally negligent. A, B and C are all incorrect because while Mary engaged in sufficient conduct to meet the actus reus requirement of criminal attempt, she did not have the requisite mental state (mens rea) to commit criminal attempt.

Jennifer asks Ezra to go to Harry's home and pick up a television for her. Jennifer says to Ezra that the key is located under the mat at the front door of Harry's home. Ezra, always happy to lend a hand, goes to Harry's home at night after he gets off from work. Ezra takes the key from under the mat, enters the home, and removes the television. As Ezra is placing the television in his car, the police arrive and arrest him for theft. Ezra explains to the police that he is not stealing the television, but is doing a favor for Jennifer. The police proceed to arrest both Jennifer and Ezra for theft. Which of the following is most correct? a. The prosecution should have charged both Jennifer and Ezra with burglary b. Jennifer cannot be convicted of theft because she did not take anything c. Ezra is not an innocent instrumentality because he should have known better than to listen to Jennifer d. Ezra should not be convicted because he is an innocent instrumentality, but Jennifer can be convicted of theft e. Both Jennifer and Ezra are guilty of theft

d. Ezra should not be convicted because he is an innocent instrumentality, but Jennifer can be convicted of theft Ezra did not have a mens rea for the commission of the crime and therefore cannot be found guilty of the crime (see § 5.06). A is incorrect because Ezra had no mens rea and therefore should not be convicted. There is also a question as to whether there was a breaking, a necessary element of burglary since Harry's key was under the mat, and Ezra was easily able to enter the home. B is incorrect because Jennifer had the mens rea and used Ezra to commit the act for her. C is incorrect because Ezra had no mens rea and therefore was an innocent instrumentality. E is incorrect because only Jennifer is guilty of a crime, while Ezra is an innocent instrumentality.

Olivia, who is unable to take care of herself, is the mother of Frankie. Frankie is charged with elder abuse for taking his mother's Social Security money but not using the money to feed her. Which of the following is most correct? a. If Frankie loves his mother, he cannot be convicted of a crime because he lacks an actus reus b. Frankie cannot be charged with a crime because most states do not have Good Samaritan statutes. c. Frankie can be convicted of a crime because he has a moral duty to care for his mother. d. Frankie can be convicted of a crime because he has a legal duty to care for his mother

d. Frankie can be convicted of a crime because he has a legal duty to care for his mother D is correct because having a legal duty to care for someone and not acting on that duty is considered an omission to act as seen in the case of West v. Commonwealth. A is incorrect because an omission to act can be the basis for the actus reus of a crime. B is incorrect because although it is correct that most states do not have Good Samaritan statutes, an omission to act when there is a legal duty to act can serve as the basis for criminality. C is incorrect because an omission to act needs to be premised upon a legal duty as opposed to a moral duty.

Hans leaves his glasses home when he goes out to deliberately shoot and kill co-worker Solomon, who he is angry with. Hans enters the bar where Solomon typically drinks following work. Hans points his gun at a person sitting on a stool at the bar thinking that it is Solomon and shoots and kills this person. The coroner says that the person iI dead, but it turns out that Hans shot and killed Abraham, a person he did not even know. In most jurisdictions, which of the following is most correct? a. Hans can be held criminally liable under the doctrine of transferred intent b. Hans can be held criminally liable under the doctrine of willful blindness c. Hans can be held criminally liable under the doctrine of ejusdem generis d. Hans can be held criminally liable because he purposely shot and killed the person on the bar stool e. Hans cannot be held criminal liable because of the rule of lenity

d. Hans can be held criminally liable because he purposely shot and killed the person on the bar stool When one purposely shoots and kills someone, they have committed murder. The victim is not an element of the crime, as long as there is a person who is pronounced dead as a result from the purposeful shooting (see § 5.05). A is not correct because there is no need to use transferred intent since the actor shot the individual he intended. B is incorrect because this is not willful blindness, but rather a purposeful killing. C is incorrect because the doctrine of ejusdem generis is a statutory interpretation tool, and there is no statute being interpreted here. E is incorrect because the rule of lenity pertains to interpretating a criminal statute, and a statute is not being interpreted here.

In assessing predisposition, which of the following are factors that a court might consider? I. Whether the defendant readily responded to the inducement offered II. The circumstances surrounding the illegal conduct III. The state of mind of a defendant before the government agents make any suggestion that the defendant shall commit a crime IV. Whether the defendant was engaged in an existing course of conduct similar to the crime for which the defendant is charged V. Whether the defendant regularly attended church or another religious gathering VI. Whether the defendant has refused to commit similar acts on other occasions Which of the following are most correct? a. I, II, and III b. I, III, and VI c. I, II, IIII, and VI d. I, I, II, IV, and VI e. I, II, III, IV, V, and VI

d. I, I, II, IV, and VI The court can weigh many legitimate factors to determine if the accused had the predisposition to commit the offense, or whether the accused was entrapped into doing the crime. Factors included in United States v. Dion were (1) whether the defendant readily responded to the inducement offered; (2) the circumstances surrounding the illegal conduct; (3) the state of mind of a defendant before the government agents make any suggestion that the defendant shall commit a crime; (4) whether the defendant was engaged in an existing course of conduct similar to the crime for which the defendant is charged; (5) whether the defendant had already formed the design to commit the crime for which the defendant is charged; (6) the defendant's reputation; (7) the conduct of the defendant during negotiations with the undercover agent; (8) whether the defendant has refused to commit similar acts on other occasions; (9) the nature of the crime charged; and (10) the degree of coercion which the law officers contributed to instigating the transaction relative to the defendant's criminal background (see § 18.02). A is incorrect because it is limited to three factors, but omits IV and VI. B is incorrect because it omits II and IV, two factors that can go to determining predisposition. C is incorrect because it omits IV. E is incorrect because it includes V, whether the defendant regularly attended church or another religious gathering. This would be an improper factor as race, religion, and gender are not permissible factors in determining criminal culpability.

John, a commercial fisherman who caught undersized fish, failed to bring some of the undersized fish back to shore as ordered by federal authorities. Instead, he told his crew to toss some of the undersized fish overboard. He was convicted of a federal obstruction statute, passed following the collapse of Enron, that penalized the destruction of "tangible objects." In interpreting the term "tangible object," which of the following might the court consider? I. The term "tangible object" as used contextually in the statute II. Dictionary definitions of the term "tangible object" III. The term as used in similar statutes IV. The alter ego doctrine V. The noscitur a sociis canon VI. Ejusdem generis VII. Rule of lenity a. I, II, and III b. I, II, III, and VI c. I, II, III, VI, and VII d. I, II, III, V, VI, and VII e. All of the above

d. I, II, III, V, VI, and VII All of the above answers are different methods of statutory interpretation, with the exception of the alter ego doctrine (IV). The alter ego doctrine pertains to self-defense.

A prosecutor has a unique ethical role in the criminal justice system. According to the ABA ethics rules, a prosecutor in a criminal case shall: I. Refrain from prosecuting a charge that the prosecutor knows is not supported by probable cause II. Make reasonable efforts to assure that the accused has been advised of the right to, and the procedure for obtaining, counsel and has been given reasonable opportunity to obtain counsel III. Not seek to obtain from an unrepresented accused a waiver of important pretrial rights, such as the right to a preliminary hearing IV. Make timely disclosure to the defense of all evidence or information known to the prosecutor that tends to negate the guilt of the accused or mitigates the offense, and, in connection with sentencing, disclose to the defense and to the tribunal all unprivileged mitigating information known to the prosecutor, except when the prosecutor is relieved of this responsibility by a protective order of the tribunal a. I b. II c. I, II, and III d. I, II, III, and IV e. There are no ethical rules that relate to the conduct of a prosecutor

d. I, II, III, and IV

Defendant Bernie is convicted of fraud. Which of the following factors should the court use in determining Bernie's sentence? I. Nature of the Offense II. Character of the Offender III. Protection of Public Interest IV. Race of the Defendant V. Wealth of the Victims a. I b. I and III c. II, III, and V d. I, II, and III e. I, II, III, IV, and V

d. I, II, and III Courts typically will look at the nature of the offense, looking at whether the crime is violent, hurt many individuals, is a felony, and other such factors. Courts also look at the character of the offender, including whether they have a prior criminal history. Courts also look to protect the public and the sentence may reflect a need to isolate the individual, keep the individual away from their present position where they can harm others, or provide some other way to assure that the public is protected.

Which of the following crime(s) require(s) proof of a specific intent? I. Common law rape II. Attempted murder III. Common law burglarly IV. Conspiracy to commit money laundering a. I, II, and III b. III c. II and IV d. II, III, and IV e. All of the above

d. II, III, and IV Common law rape is general intent, but inchoate offenses such as conspiracy and murder require a specific intent (see § 5.01). A is incorrect because common law rape is a general intent crime. B is incorrect because although common law burglary is specific, the answer omits choices II (attempted murder) and III (conspiracy to commit money laundering), which are also specific intent crimes. Inchoate offenses, such as attempt and conspiracy, require specific intent. C is incorrect because it omits common law burglary, which is a specific intent crime. E is incorrect because it includes common law rape, which is a general intent crime.

On Thursday, Dan offered to sell his car to Aaron for $900. Aaron agreed to buy Dan's car on Sunday evening. Aaron worked all weekend for a moving company to earn the $900 and was paid by the moving company in cash. On Sunday, Aaron gave Dan an envelope with his $900 cash earnings, and Dan gave Aaron the keys and the title to the car. Two days later, Dan learned that the $900 in cash was all counterfeit bills. Which of the following would be most correct? a. Aaron committed the crime of common law larceny when he took possession of the car from Dan b. Aaron committed the crime of false pretenses because he acquired possession and title to the car based on the misrepresentation that he paid $900 to Dan c. Aaron committed the crime of larceny by trick because he fraudulently induced Dan to give him the car based on counterfeit money of no value d. If Dan did not know the $900 was counterfeit, he has not committed any property crime with respect to the purchase of the car

d. If Dan did not know the $900 was counterfeit, he has not committed any property crime with respect to the purchase of the car The crime of false pretenses requires criminal intent—specific intent to defraud through a knowing misrepresentation of facts that induces the transfer of property. If Aaron was unaware and had no reason to know that the moving company paid him in counterfeit bills and that he passed counterfeit bills to Dan in exchange for the car, Aaron does not have the intent required to meet the elements of false pretenses. Likewise, proof that Aaron knew the money was counterfeit would be required before Aaron's actions would constitute larceny by trick. Because Dan voluntarily surrendered possession of the car to Aaron, there was no trespassory taking of property as required for common law larceny.

Kelly, a three-year old child, goes to the town's swimming pool with her mother Kristen. The town provides a lifeguard on duty to watch the children in the pool and that day the lifeguard is Tony, who very much dislikes Kristen. Kelly starts to drown in the pool and Tony says to Kristen, "you get her, she's not my problem." If Tony and Kristen are charged with a crime for failing to save Kelly, which of the following is most correct? a. If Kristen is capable of saving Kelly, only she can be charged because she owes a duty as a parent to save her child b. Even if Kristen is capable of saving Kelly, only Tony is responsible for saving Kelly, and therefore only Tony can be charged for failing to save Kelly c. If Kristen is aware that Kelly needs to be saved, and is capable of saving Kelly, she still cannot be chaged with a crime because Tony is there as the lifeguard d. If both Kristen and Tony are capable of saving Kelly, both can be charged with a crime because they both had a duty to act. e. Neither Kristen or Tony can be charged with a crime because one assumes their own risk when they swim in a town's swimming pool

d. If both Kristen and Tony are capable of saving Kelly, both can be charged with a crime because they both had a duty to act A is correct in that a parent does have a duty to act to care for a child. But A is incorrect in that Kristen is not the only one who can be charged as Tony also has a duty to act. B is correct that Tony can be charged, but incorrect in that he is not the only one who can be charged if Kelly's mother Kristen is capable of saving her. C is incorrect because the parental duty does not go away just because a lifeguard is present if the mother is aware of the need to save her child and is capable of doing so. E is also incorrect because tort civil liability is separate from criminal liability.

Cleo, while walking home from work, observes Max and Solomon fighting. Cleo sees that Max punched and injured Solomon. Cleo thinks that it is terrible what Max is doing to Solomon, but decides that to stop to call the police will result in being late for dinner. Cleo fails to stop, fails to call the police, and just keeps walking. As a result of the fight with Max, Solomon dies. Which of the following is most correct? a. In most jurisdictions in the United States, Cleo can be criminally liable because of a duty to assist Solomon b. In most jurisdictions in the United States, Cleo can be criminally liable because Cleo had the ability to intervene and failed to do so c. A and B d. In most jurisdictions in the United States, Cleo cannot be criminally liable because there is no duty to intervene here

d. In most jurisdictions in the United States, Cleo cannot be criminally liable because there is no duty to intervene here Most jurisdictions do not have Good Samaritan laws. Absent a statute or legal duty, there is no legal requirement to assist. Answers A, B, and C are incorrect because there is no duty to intervene, and one's ability to successfully intervene does not come into play absent a statute requiring one to act

Jamie, Raleigh, and Alice are sitting in a coffee shop in State A when they decide to rob a bank located in State B. To get to State B they travel through State C where they purchase guns for the upcoming crime. Some of the guns purchased were not legally obtained. They eventually arrive in State B, commit the bank robbery and leave State B and go to State D where they divide up the money from the robbery. Jamie, Raleigh, and Alice are all charged with a federal crime of conspiracy to commit bank robbery. Which of following is most correct? a. Jamie, Raleigh and Alice should be charged in a federal district court in State A, the location where they reached the agreement to commit the crime b. Jamie, Raleigh and Alice should be charged in a federal district court in State B, the location of the bank robbed c. Jamie, Raleigh and Alice should be charged in a federal district court in State D, the location where they divided the proceeds from the robbery d. Jamie, Raleigh and Alice can be charged in a federal district court in either State A, B, C, or D, and the prosecutor has the discretion to choose the venue for this case e. Jamie, Raleigh and Alice should be charged in a federal district court in State C, the location where they obtained the guns to commit this robbery

d. Jamie, Raleigh and Alice can be charged in a federal district court in either State A, B, C, or D, and the prosecutor has the discretion to choose the venue for this case Prosecutors have wide discretion with respect to venue in conspiracy cases. They can bring the charges in the location of the agreement or any of the acts in furtherance of the conspiracy. In this case, State A is the location of the agreement. State B is the location of the act of actual crime. State C provides the guns, which is an element of the robbery, putting in fear those being robbed. State D is a part of the conspiracy crime as they are dividing up the money.

While Letti was walking across the parking lot to enter Target, she saw Tracey unknowingly drop a piece of paper on the ground. Letti retrieved the document and saw that it was a receipt with a QR code to pick-up a $300 online order from Target. Letti entered Target and observed Tracey frantically searching her purse for the receipt. Letti watched as Tracey ran out of the Target and raced back to her car. Letti then approached the pick-up counter and presented Tracey's receipt. The clerk scanned the QR code on the receipt and gave Letti the package of goods purchased by Tracey. Letti then left Target with the goods. Which of the following would be most correct? a. Letti has committed the crime of robbery b. Letti has committed the crime of embezzlement c. Letti has committed the crime of false pretenses d. Letti has committed the crime of larceny

d. Letti has committed the crime of larceny The receipt that Tracey dropped is lost or mislaid property found by Letti. Letti retrieved the lost receipt and took no action to return it to Tracey, its rightful owner. In using the lost receipt to procure Tracey's property Letti committed larceny.

Using the statement and the statutory provisions below, determine whether sexual assault laws were violated. "My name is Yara. I had sexual intercourse with my neighbor, Jabar, because he threatened to report my grandmother to the immigration authority after I confided in him that she is undocumented. I love my grandmother, and she is the only family I have. So, to save my grandmother from deportation I had sex with Jabar like he wanted. He never threatened or used any physical force on me, but he knew I did not want to do it. I cried the entire time and begged him not to make me do it." Which of the following statutory provisions could be used to charge Jabar with sexual assault? a. "A person is guilty of sexual assault if that person engages in a sexual act with another person and the actor compels or induces the other person to engage in the sexual act by threat of physical injury against the person" b. "A person is guilty of sexual assault if the person engages in or causes another person to engage in or submit to a sexual act and knows that the other person is incapable of declining participation in that sexual act" c. "A person is guilty of sexual assault if the person uses threats or intimidation to cause the other person to fear for their safety or the safety of another" d. None of the above

d. None of the above A is incorrect because Jabar never threatened physical harm to Yara. C is incorrect because in threatening to reveal the secret of the grandmother's immigration status, Jabar did not threaten physical harm against Yara or her grandmother. B is incorrect because, although Yara felt compelled to have sex with Jabar to save her grandmother, she was not "incapable of declining" Jabar's request.

Using the statement and the statutory provisions below, determine whether sexual assault laws were violated. "My name is June. I agreed to perform oral sex on my boss, Lane, because I had no choice. He came into my work area and said that if I did not perform the sex act I would be fired. I did what Lane asked because I have a criminal record, and I am on parole. My parole could be revoked, and I could be sent back to prison if I get fired from my job. Lane knows all of this and took advantage of my situation. I didn't want to do it, but he forced me to. I felt I had no choice." Which statutory provision could be used to bring charges against Lane? a. "A person is guilty of sexual assault if that person engages in a sexual act with another person and the actor compels or induces the other person to engage in the sexual act by threat of physical injury against the person" b. "A person is guilty of sexual assault if the person has sexual contact or sexual intercourse with an individual who is on probation, parole, or extended community supervision if the actor is a probation, parole, or extended supervision agent who supervises the individual, either directly or through a subordinate, and has the ability to influence whether the other person faces a return to confinement" c. "A person is guilty of sexual assault if that person engages in a sexual act with another person who is under criminal justice supervision as a probationer, parolee, sex offender, or on supervised release and the actor has supervisory or disciplinary authority over the person's criminal justice supervision status d. None of the above

d. None of the above A is wrong because Lane did not threaten physical harm if June failed to meet his sexual demand. C is wrong because, although June is under criminal justice supervision (parole), Lane is not June's parole officer or a person with disciplinary authority over June's parole status. B is incorrect because this statutory provision applies only to an administrator within the parole agency who could order or influence June's parole officer to revoke her parole.

Carli learned that after school Alice planned to engage in a physical altercation with Connie, her cheerleading rival. Carli, who also hates Connie, wanted to see Alice beat Connie. Carli decided to go on her own to the parking lot to watch the fight. Carli remained out of sight as Alice and Connie punched, kicked, and slapped each other. Carli filmed the fight and posted the fight online. Alice was later arrested for starting the fight with Connie. The following day the police arrested Carli as an accomplice. Which of the following is correct? a. Going to the fight made her an accomplice because she wanted Alice to win and provided support for Alice by filming the fight b. Carli is an accomplice because she was present at the time of the fight, had prior knowledge of the fight, and took no action to break up the fight or call the police once the fight started c. Going to the fight, without more intent, is insufficient to make Carli an accomplice, but Carli filming the fight and posting it online is sufficient conduct to make Carli an accomplice d. None of the above

d. None of the above Carli did nothing to aid or assist Alice. Mere knowledge of the crime and mere presence at the scene of the crime is insufficient to make Carli an accomplice. A and C are wrong because Carli arrived on her own, stayed out of sight, and did not cheer or encourage Alice during the fight or provide support for Alice. Her mere act of filming the fight and posting the video did not, therefore, make Carli an accomplice to Alice's criminal conduct. B is wrong because Carli was only a classmate to the two girls and did not have a legal obligation intervene to stop the fight.

Mike receives a call from his son who attends the local elementary school a couple of miles away. His son reports that he is hiding under the desk because of a shooter. Mike tells his son that he is on his way. When Mike races outside, he sees a FedEx truck blocking his driveway and the driver across the street delivering packages. Mike tries to summon the driver, but the driver is wearing earphones and cannot hear Mike. Mike notices that keys are in the ignition and the truck is running. Feeling a sense of urgency, Mike jumps in the truck and races to his son's school. While en route, Mike called FedEx and left a message with customer service that he just borrowed one of their trucks, explained the dire circumstances at his son's school, provided his contact information, and promised to return the truck as soon as possible. When Mike received a text that the school was merely conducting an active shooter safety drill he made a u-turn and brought the truck back. Which of the following would be most correct? a. Mike committed the crime of robbery b. Mike committed the crime of common law larceny c. Mike committed the crime of burglary d. None of the above

d. None of the above Mike did not commit a common law larceny because he did not have the intent to permanently deprive the driver or FedEx of the truck, as evidenced by his telephone call and the fact that he returned home with the truck within a few minutes. Likewise, Mike did not commit a robbery because Mike did not take the truck from the physical possession of the FedEx driver. Moreover, Mike did not commit a burglary because he did not break and enter a dwelling.

Kelly wants to kill Maggie because they have been engaged in a heated dispute over a debt. Which of the following constitutes criminal attempt under the Model Penal Code? a. On Monday, Kelly went to the hardware store and purchased zip tie restraints, duct tape, and a tarp to dispose of Maggie's body. Maggie's mother knew of the dispute between Maggie and Kelly and saw Kelly's purchases in the hardware store. After Maggie's mother notified the police, Kelly was arrested b. On Monday, Kelly bought a gun and planned to use it to kill Maggie when she saw her on Thursday at Zumba class. On Tuesday, Kelly bought ammunition, and on Wednesday she purchased a mask to disguise her appearance. Kelly asked Jane to film the fatal encounter. Instead, on Wednesday, Jane called the police to report Kelly's plan c. On Monday, Kelly prepared a detailed checklist of things she needed to do to kill Kelly. When Kelly's husband Bob told the police of Kelly's plan, Kelly was arrested Monday night d. None of the above

d. None of the above The facts in A, B, and C show "mere preparation" to commit the crime of murder, but not a substantial step for criminal attempt.

Which of the following issues would most likely be decided by a judge, not a jury? a. Whether a prosecution witness has given false testimony b. Whether the defendant acted in self-defense c. Whether the defendant has presented a credible alibi defense d. None of the above

d. None of the above The jury—not the judge—makes factual determinations regarding whether a witness is lying, whether evidence is credible and whether they are persuaded by the evidence that the defendant acted in self-defense.

Andy decided to get very drunk during his 40th birthday party at a local bar. Despite pleas from friends and family, Andy insisted on driving himself home. While driving at an excessive speed down a residential street, Andy's vision became blurry and he slammed into a bus shelter where Mary was waiting to catch the bus. Mary suffered severe injuries, but survived. Andy's blood-alcohol level was twice the legal limit at the time of the crash. Andy is charged with attempted murder of Mary. Which of the following is correct? a. Andy has committed the crime of attempted murder because he intentionally drove his car when he knew he was drunk, and then recklessly plowed into the bus shelter b. Andy has committed the crime of attempted murder because his act of driving his car into the bus shelter and nearly killing Mary constitutes a "substantial step" c. Because voluntary intoxication is not a defense, Andy has committed the crime of attempted murder d. None of the above is correct

d. None of the above is correct Andy's drunk driving and speeding is, at most, criminally reckless conduct. While driving his car into the bus shelter where Mary is sitting and nearly killing her meets the actus reus requirement for criminal attempt, there are no facts to support a finding that Andy had the specific intent to kill Mary. A is wrong because intentionally driving when you know you are drunk is reckless and irresponsible, but does not constitute a specific intent to kill. While, as a general rule, voluntary intoxication is not a defense, C is wrong because voluntary intoxication could negate the formation of specific intent required for the criminal attempt. B is wrong because the substantial step for criminal attempt must co-exist with the specific intent to commit the crime.

Which of the following punishment theories is not considered a utilitarian theory of punishment? a. Rehabilitation b. General Deterrence c. Specific Deterrence d. Retribution e. Education

d. Retribution Utilitarian theories focus on accomplishing a good purpose for society. Retribution looks at past conduct, with a goal being to ensure that the individual gets what they deserve for their criminal act. A, B, C, and E are incorrect because they focus on future conduct and stopping individuals in the future from committing the crime.

Sandy was upset and wanted to kill her mother-in-law for interfering in her marriage to Elbert. Sandy bought some tea bags laced with a toxin that, when boiled and combined with the mother-in-law's prescription medication, would kill her. Sandy kept the toxic tea hidden in her purse. The following day Sandy placed water in the tea kettle and retrieved a bag of the toxic tea from her purse. Sandy then had second thoughts and destroyed all of the toxic tea. The following day, Sandy confessed her actions to her husband who reported Sandy to the police. Which of the following is correct under the Model Penal Code? a. Sandy committed the crime of attempted murder when, with the specific intent to kill her mother-in-law, she purchased the toxic tea b. Although Sandy did not serve the toxic tea to her mother-in-law, she committed the crime of attempted murder when she filled the kettle and the tea bags were in her purse c. Sandy committed the crime of attempted murder when, with the specific intent to kill her mother-in-law, she brought the toxic tea home and hid it in her purse d. Sandy has a strong abandonment defense if she is charged with criminal attempt

d. Sandy has a strong abandonment defense if she is charged with criminal attempt Under § 5.01(2)(e) of the MPC Sandy's purchase and possession of the toxic tea would be criminal attempt because Sandy was in possession of material "specifically designed for unlawful use" that could "serve no lawful purpose." However, under § 5.01(4) of the Model Penal Code, when Sandy destroyed all of the toxic tea she "prevented the successful commission of the offense" under circumstances manifesting a "complete and voluntary renunciation" of her criminal purpose.

The Rome Statute of the International Criminal Court provides as one of the grounds for excluding criminal responsibility: "if, at the time of that person's conduct: (a) the person suffers from a mental disease or defect that destroys that person's capacity to appreciate the unlawfulness or nature of his or her conduct, or capacity to control his or her conduct to conform to the requirements of law." Which of the following tests does the Rome statute most closely resemble? a. The M'Naghten Test b. The Irresistible Impulse Test c. The Durham Test d. The Model Penal Code Test

d. The Model Penal Code Test The Model Penal Code or American Law Institute (ALI) test is a combination of the M'Naghten Rule and the "irresistible impulse" test. It states: "(1) A person is not responsible for criminal conduct if at the time of such conduct as a result of mental disease or defect, he lacks substantial capacity either to appreciate the criminality (wrongfulness) of his conduct or to conform his conduct to the requirements of the law ..." A is incorrect because it only includes one part of the ALI test — that he lacks substantial capacity to appreciate the criminality (wrongfulness) of his conduct. C is incorrect because the Rome Statute is not premised on the product test — that a defendant is not criminally responsible if his unlawful act was a product of mental disease or defect. B is incorrect because it only includes one part of the ALI test — to conform his conduct to the requirements of the law.

Victor borrowed money from Giovanni, a loan shark. When Victor could not pay back the $10,000 he owed, Giovanni offered to wipe Victor's debt clean if Victor would kill his ex-wife, Michelle, and her new boyfriend, Sylvester. Giovanni told Victor that if he failed to complete the job by the end of the week, Giovanni would have Victor killed. Giovanni gave Victor two days to think it over. After Victor left, he realized that even if he committed the murders Giovanni would likely still have him killed. Victor arranged to meet with Giovanni the next day and fatally shot Giovanni. Victor then immediately turned himself in to the police and claimed he killed Giovanni under duress and out of necessity. Which of the following would be most correct? a. Because killing Giovanni prevented the death of Michelle, Sylvester, and likely Victor, under the Model Penal Code, Victor has a strong necessity defense b. Victor has strong evidence that he was under duress because Giovanni threatened to kill Victor if Victor did not execute two people c. Victor has a strong necessity defense, but a weak duress defense d. Victor will not be able assert the defense of duress or necessity under these facts

d. Victor will not be able assert the defense of duress or necessity under these facts Under centuries-old common law principles, the defenses of duress and necessity are not available defenses to the crime of murder. Courts have reasoned that all life is equally valuable, and killing one person (even to save yourself) is not the lesser of two evils

Which of the following determinations would most likely be made by a judge? a. Whether the defendant will be permitted to present evidence of self-defense in an assault case b. Whether in a larceny case the defendant will be allowed to present evidence that the property he is alleged to have stolen is, in fact, his own property c. Whether the defendant has adequately proved that he was not present at the time the crime was committed and is entitled to an alibi instruction d. Whether the prosecution has presented sufficient evidence of premeditation and deliberation to support a first degree murder charge e. None of the above

d. Whether the prosecution has presented sufficient evidence of premeditation and deliberation to support a first degree murder charge The judge has the exclusive authority to determine whether, as a matter of law, the prosecution has presented sufficient evidence to support the charges. If not, the judge will grant a motion for judgment of acquittal. A and B are wrong because the judge cannot preclude the defendant from exercising his constitutional right to present a defense. In A and B the proffered evidence would constitute a defense to the charges. C is wrong because the defendant does not have to prove his alibi in order to give an alibi instruction; the defense need only present sufficient evidence to support the instruction (which can consist of the defendant's bare bone assertion that at another location when the crime was committed).

Arlie, an employee of a company on the U.S. Securities Exchange Commission, is charged in the United States with bribing a foreign official in another country in violation of the Foreign Corrupt Practices Act. Arlie wishes to argue that the U.S. does not have jurisdiction to prosecute because the act of bribery occurred outside this country. Which of the following is most correct? a. Arlie will be unsuccessful because, in passing the Foreign Corrupt Practices Act, Congress made clear that extraterritorial prosecutions would be allowed b. Arlie will be unsuccessful because, in passing the Foreign Corrupt Practices Act, Congress created statutes that were focused. on extraterritorial conduct c. Arlie will be successful because prosecutions in the United States require territorial acts in this country d. Arlie will be successful because the extraterritorial prosecutions are not allowed in United States courts e. A and B above

e. A and B above When Congress explicitly states within a statute that it intends the statute to cover acts outside the United States, extraterritorial jurisdiction is clear. This includes statutes that are focused only on extraterritorial conduct, such as the Foreign Corrupt Practices Act (FCPA), which criminalizes international bribery. A is correct, but it is not the best answer because B is also correct. C is incorrect because although normally prosecutions require territorial acts, if Congress explicitly states otherwise as it did in the Foreign Corrupt Practices Act, extraterritorial prosecutions will be allowed. D is incorrect because extraterritorial prosecutions are allowed in the United States when there is a clear intent on the part of Congress to address extraterritorial conduct.

Kim is convicted of harboring an unreasonably loud or disturbing animal in violation of a city ordinance that states "no person shall keep or harbor any animal which howls, barks, or emits audible sounds that are unreasonably loud or disturbing and which are of such character, intensity and duration as to disturb the peace and quiet of the neighborhood or to be detrimental to the life and health of any individual." Kim's dog had barked constantly for an hour and a half and a neighbor said that the dog was barking so loudly that it could be heard over the sound of his lawn mower, and this was inside the house with the windows closed and the air conditioner running. Which of the following is most correct? a. A court must reverse this conviction because this statute is unconsitutionally vague b. A court must reverse this conviction because the rule of lenity requires this result c. A court must reverse this conviction because the statute is overbroad d. A, B, and C e. A court may find that this staute is not vague as applied

e. A court may find that this staute is not vague as applied This question comes from the case Columbus v. Kim, where the defendant argued unsuccessfully that the above statute should be found vague. It is possible that some courts might find this statute vague or overbroad. Some might even use the rule of lenity in interpreting the statute narrowly in favor of the accused. But in the Kim case, the court found that this specific statute as applied was not unconstitutionally vague.

Norbert, who has an extremely low IQ, steals a candy bar from a local shop. Norbert does not understand that it is necessary to pay for items prior to taking them from the shop. Which of the following is most correct? a. In some jurisdictions Norbert will be able to argue successfully that he lacks the mens rea to be convicted of the crime b. In some jurisdictions Norbert will be able to argue successfully that he suffers from diminished capacity c. In some jurisdictions Norbert will only be allowed to argue diminished capacity if his actions rise to the level of insanity d. In some jurisdictions Norbert will only be successful arguing diminished capacity because theft is a specific intent crime e. A, B, C, and D

e. A, B, C, and D Jurisdictions vary on their acceptance of diminished capacity and when and how it can be used. Some jurisdictions allow diminished capacity to negate the mens rea, some as a separate defense, and some only if it rises to the level of insanity. Some also limit it to specific intent crimes, allowing the diminished capacity to reduce it to a general intent crime. A, B, C, and D are, therefore, all correct statements, making E the best answer.

Which of the following can never be an inchoate crime? I. Solicitation II. Attempted theft III. Attempted burglarly IV. Conspiracy a. I b. II and II c. II and Iv d. I, II, III, and IV e. All can be inchoate crimes

e. All can be inchoate crimes The correct answer is E. Solicitation, attempt, and conspiracy are all considered inchoate crimes. Inchoate crimes do not require the completion of the substantive offense. Each crime has different elements, but all allow for prosecution without the completion of the criminal offense. Conspiracy will sometimes be charged when the offense is completed, but prosecutors have the option to charge it when not completed. A is incorrect because solicitation, an offense that is often considered a prelude to a conspiracy charge, is definitely an inchoate offense but is not the best answer since it omits other inchoate offenses. B is incorrect because all attempt charges, regardless of the substantive offense attached to it, can be inchoate offenses. The prosecutor does not have to wait until the theft or burglary is actually completed to charge the offender. C is incorrect because conspiracy and attempted theft can both be inchoate. Both require a specific intent, an essential element of these charges. D is incorrect because the question asks for which can never be inchoate offense and all of these are inchoate crimes.

In defining the word "asportation" as used in a statute, a court should look at which of the following? a. Whether the statute using the term "asportation" has the same language as that used in the Model Penal Code b. Whether the statute was enacted because a prior statute was too limited to secure convictions for the criminality c. How the term was defined in prior cases in that jurisdiction d. What the legislature intended when they drafted the statute e. All of the above

e. All of the above

Under the Model Penal Code, which of the following are not considered voluntary acts? a. A reflex or convulsion b. A bodily movement during unconsciousness or sleep c. Conduct during hypnosis or resulting from hypnotic suggestion d. A bodily movement that otherwise is not a product of effort or determination of the actor either concious or habitual e. All of the above

e. All of the above

Which of the following are accurate about the United States federal sentencing guidelines? I. When initially enacted they were considered to be mandatory II. In sentencing a defendant today in the federal system, a court considers the sentencing guidelines III. In sentencing a defendant today in the federal system, a court considers the nature and circumstances of the offense and characteristics of the defendant IV. In sentencing a defendant today in the federal system, a court considers the need to avoid unwarranted sentence disparities among defendants with similar records who have been found guilty of similar conduct a. I b. I and II c. II, III, and IV d. I, II, and III e. All of the above

e. All of the above

Assistant United States Attorney Lang is trying to decide whether to charge both bank robbery and conspiracy to commit bank robbery. The basic facts are that Joshua and Bonnie went to Bank A in State One and committed a bank robbery. Following the robbery they went to lunch at a local restaurant and discussed how they would split the money from the bank robbery. A patron in the restaurant overheard their conversation. Joshua and Bonnie then went to Bank B in State Two and were about to commit a bank robbery when the alarm in the bank went off and both Joshua and Bonnie were arrested. Which of the following is most correct? a. It would be beneficial for AUSA Lang to charge conspiracy because statements heard by the patron in the restaurant that were made between Joshua and Bonnie would be admissible into evidence b. It would be beneficial for AUSA Lang to charge conspiracy as it would allow the charging of Joshua and Bonnie for the conspiracy to commit the robbery of Bank B c. It would be beneficial for AUSA Lang to charge conspiracy in order to select the venue for proceeding with the crimes. d. It would be beneficial for AUSA Lang to charge conspiracy as there would be a guilt by association in having both defendants tried together e. All of the above

e. All of the above A is correct because Federal Rule of Evidence 801(d)(2)(E) provides an exception to the hearsay rule, that "a statement by a co-conspirator of a party during the course and in furtherance of the conspiracy" is admissible evidence (see § 13.01 and § 13.04). C is correct because prosecutors have great flexibility in choosing the venue in a conspiracy case because jurisdiction can be found wherever an overt act occurred or where the parties formed the agreement (see § 13.01). B is correct because having co-conspirators at the same trial creates a "guilt by association." D is correct because AUSA Lang does not need a completed offense to charge conspiracy. One of the key benefits of using a conspiracy charge is that it is an inchoate offense and therefore there is no need to wait for the completed act. Because A, B, C, and D are all correct, E is the best answer because it includes all of the benefits that the government has if proceeding with a conspiracy charge.

Which of the following is a commond ground for appeal of a criminal conviction? a. The government did not present sufficient evidence to support the criminal charges b. The trial judge erroneously instructed the jury on the elements of the crime c. The trial judge erroneosuly excluded evidence that should have been presented at trial d. The trial judge should have excluded certain evidence that was obtained in violation of the defendant's constitutional rights e. All of the above

e. All of the above A is correct because the government has the burden of proof and the defense will frequently argue that the trial judge should have dismissed the charge(s) because the government failed to meet their burden. B, C, and D are correct because each addresses a challenge to very common legal rulings made by trial judges in criminal cases.

Charlie approaches Lynn at a train station and says, "Any interest in buying some cocaine?" Lynn replies, "Sure." Unbeknownst to Lynn, Charlie is an undercover police officer. Following the sale to Lynn of the cocaine, Charlie arrests Lynn for the felony charge of possession of cocaine. Lynn wants to raise a defense of entrapment. Which of the following is most correct? a. If this is a federal charge, the court will use a subjective view of entrapment b. If an objective view of entrapment is used by a court then a key determination will be whether Charlie's conduct was manufactured by the government c. If this is a Model Penal Code jurisdiction, the court will use an objective view of entrapment d. If a subjective view of entrapment is used by the court then a key determination will be whether Lynn acted with the predisposition to purchase the drugs e. All of the above

e. All of the above All of the above statement are accurate. A is correct because federal courts use a subjective test when examining whether the accused had the predisposition to commit the offense (see § 18.02). C is correct because a minority of states and the Model Penal Code use an objective standard, looking at the police conduct (see § 18.03). D is correct because a subjective view includes looking at whether the accused had the predisposition to commit the crime (see § 18.02). B is correct because an objective view used by the Model Penal Code and minority of states focuses on the police conduct of the accused.

Which of the following might be considered a collateral consequence of a criminal conviction (depending on the crime committed by the defendant)? I. Sex Offender Registration II. Prohibition on Posessing a Firearm III. Barred from Voting IV. Loss of Business or Professional License V. Barred from doing Business with the Government VI. Deportation a. I b. I, II, and III c. I, III, IV, and VI d. I, II, III, and VI e. All of the above

e. All of the above All of these can be collateral consequences of a criminal conviction. Collateral consequences are the restrictions that may be placed upon a defendant as a result of their criminal conviction.

Which of the following sources have United States Supreme Court justices used in supporting their decisions in death penalty cases? I. The U.S. Constitution II. U.S. Supreme Court precedent III. State law IV. International conventions v. The Federalist Papers a. I b. I and II c. I, II, and V d. I, II, III, and V e. All of the above

e. All of the above In Roper v. Simmons, a death penalty case, the Supreme Court looked at the Eighth and Fourteenth Amendments of the Constitution as well as prior Court decisions such as Atkins and Thompson. Justice Kennedy's opinion also looked at how many states were allowing the death penalty for juveniles. His opinion also referenced Article 37 of the United Nations Convention on the Rights of the Child and provisions in other international conventions. Justice Scalia, however, rejected the use of international law in interpreting the United States Constitution. Justice Kennedy also references the Federalist Papers. So, all of the answers were correct, and E was therefore the most correct answer.

Using the statement and the statutory provisions below, determine whether sexual assault laws were violated. "My name is Angelica. I am in the 9th grade, and I will be 14 years old next month. My boyfriend Chris is a senior at my school and he turned 18 years old last month. Chris and I had sexual intercourse this past summer. I agreed to have sex with Chris. I knew what I was doing. He did not force me to do anything." Which of the statutory provisions below could be used to prosecute Chris? a. "A person commits the crime of rape if he or she is 17 years or older and engages in nonconsensual sexual intercourse with another person who is under 15 years of age at the time" b. "A person is guilty of sexual assault if that person engages in a sexual act with another person and the other person has not attained the age of 14 years" c. "A person who has sexual intercourse with a minor is guilty of sexual assault if the minor is less than 17 years old and the actor is at least four years older" d. All of the above e. B and C only

e. B and C only The correct answer is E. B is correct because Angelica would have been 13 years old (under the age of 14), and her consent is irrelevant under this statute. C, loosely based on the Model Penal Code § 213.3, is correct because at the time Chris and Angelica engaged in sexual intercourse he was 17 years old and Angelica was 13 years old (under the age of 17), making her four years younger than Chris, especially because Chris's 18th birthday is a month before Angelica's 14th birthday. A is incorrect because this statutory provision requires a lack of consent, and Angelica consented to intercourse with Chris.

Dewey is convicted of a violation of the federal crime of arson for throwing a Molotov cocktail through a window into the private home of Dewey's cousin in the State of Erehwon. The statute reads as follows: "Whoever maliciously damages or destroys, or attempts to damage or destroy, by means of fire or an explosive, any building, vehicle, or other real or personal property used in interstate or foreign commerce or in any activity affecting interstate or foreign commerce shall be imprisoned for not less than 5 years and not more than 20 years, fined under this title, or both." Which of the following would be Dewey's best argument on appeal? a. A Molotov cocktail is not an explosive device b. Dewey's conviction should be reversed because of the rule of lenity c. Dewey's conviction should be reversed because of ambiguity in the statute d. Dewey's conviction should be reversed because it violates the due process clause e. Dewey's conviction should be reversed because the home is a private residence

e. Dewey's conviction should be reversed because the home is a private residence The applicable case is Jones v. United States, which held that the federal arson statute did not cover owner-occupied dwellings, but rather was limited to "property being used in commerce or in an activity affecting commerce."

Which amendment of the United States Constitution prohibits cruel and unusual punishment? a. First Amendment b. Second Amendment c. Fourth Amendment d. Fifth Amendment e. Eighth Amendment

e. Eighth Amendment The Eighth Amendment states that "excessive bail shall not be required, nor excessive fines imposed, nor cruel and unusual punishment inflicted."

Guinevere is extremely intoxicated, but manages to walk home from the bar where she has been drinking. On the way home she observes two men starting a fire at a local home. They are clearly about to burn down this home. Guinevere ignores the men and keeps walking home. The next day, Guinevere reads in the local newspaper of an arson at this home and how Carey was killed in the fire. These events occurred in a common law jurisdiction that uses degrees of murder. Arson is included as a felony for the purposes of felony murder. Which of the following crimes did Guinevere commit? a. Felony murder b. Voluntary manslaughter c. Involuntary manslaughter d. Guinevere did not commit a crime because she was extremely intoxicated e. Guinevere did not commit a crime and her intoxication was irrelevant

e. Guinevere did not commit a crime and her intoxication was irrelevant Guinevere did not commit an act as there was no legal duty to act. This is not a proper basis for an omission to act because absent a statute, contract, assuming the care of someone, or a family relationship, there is no basis to create a legal duty to act. A, B, and C are incorrect because Guinevere did not do any act to cause a homicide. D is also incorrect because her intoxication is irrelevant to whether she committed a crime

Willful blindness is sometimes referred to as: I. The ostrich instruction II. Deliberate ignorance III. An alternative way to obtain a mens rea of knowingly IV. Conscious avoidance a. I and II b. II c. II and IV d. I, II, and III e. I, II, III, and IV

e. I, II, III, and IV Jurisdictions refer to willful blindness using different terms. All of the terms used here are correct (see § 5.04).

Which of the following is a TRUE statement? a. Once a defendant has been indicted by a grand jury, the defendant is no longer presumed innocent under the law b. The government does not have the burden of proving guilt beyond a reasonable doubt in a misdemeanor case c. Because jury nullification is a crime, jurors can be prosecuted if they disregard the law and do not return a verdict consistent with the evidence presented at trial d. All of the above are true e. None of the above are true

e. None of the above

A police officer is flagged down in the street by Gerald Hunter, the pharmacist who owns the Hunter Community Pharmacy. Hunter frantically informed the officer that a man just jumped over the pharmacy counter, shoved him and his staff aside, and snatched several prescription bottles of oxycodone and several other powerful pain killers. When the police officer asked Hunter which direction the man fled, Hunter responded: "He didn't get away. I shot him when he tried to escape with the drugs. He's dead on the floor behind the pharmacy counter." Which of the following is correct? a. To prevent the man from selling and distributing the powerful drugs and causing death or serious bodily harm to the community, the pharmacist was justified in using deadly force for the protection of others in the community b. The pharmacist can assert a valid self-defense claim because when the man assaulted him and his staff he posed a risk of causing imminent death or serious bodily harm c. Given the fact that multiple bottles of prescription pain killers would have a value of well over a thousand dollars, the man was committing a felony offense. Thus, the use of deadly force was justified d. As the owner of the pharmacy, Hunter was justified in using deadly force in defense of property e. None of the above

e. None of the above A is incorrect because self-defense can only be justified by an imminent threat of harm, not a potential future threat of harm to the community due to the possible effects of drug distribution. B is wrong because the man pushed Hunter and the staff aside to get to the drugs and was shot as he was leaving with the drugs. Therefore, he did not pose a threat of death or serious bodily harm to Hunter or his staff. Moreover, the use of deadly force was grossly out of proportion to the simple assault (shove) by the man. D is incorrect because, in general, deadly force is not permissible to defend or retain property. Also, under the Model Penal Code, deadly force in defense of property is only justified is the assailant threatens physical harm or poses a risk of causing physical harm. C is wrong because the right to use deadly force in defense of property is not contingent on the value of the property stolen, but on whether there is an accompanying threat or risk of physical harm or death.

Marley, a Norwegian worker on a cruise ship, commits a statutory rape of a 12-year-old U.S. citizen. The ship is owned by a Danish company, flies under a German flag, and the rape occurred in waters off the coast of South Africa. A United States prosecutor would like to prosecute Marley in the United States. Which of the following is the strongest argument for jurisdiction in the United States? a. Objective territorial jurisdiction b. Protective jurisdiction c. Nationality jurisdiction d. Universality jurisdiction e. Passive personality jurisdiction

e. Passive personality jurisdiction Passive personality jurisdiction looks at the nationality of the victim. As the victim is a U.S. citizen, this argument will be the strongest for jurisdiction to prosecute in the United States. The different principles of jurisdiction come from international norms, sometimes referred to as the Harvard principles. A is incorrect as objective territoriality principle looks at the location where the crime was committed and also allows countries to reach acts committed outside territorial limits but intended to produce, and producing, detrimental effects within the nation. This is a principle often used with drug sales that have not yet arrived in the United States. C is incorrect because nationality jurisdiction is based on the nationality of the offender. In this case Marley is Norwegian, so this will not assist in obtaining U.S. jurisdiction. B is incorrect as protective jurisdiction is based on the protection of the interests and the integrity of the nation. It would include crimes such as a country hacking into a defense department computer. D is incorrect because universality jurisdiction is used for crimes such as war crimes and crimes against humanity.

State A has adopted the M'Naghten test for insanity cases. Which of the following is most correct about State A? a. State A has adopted an insanity test that is volitional b. State A has a adopted an insanity test that is both volitional and cognitive c. State A has adopted an insanity test that looks at the product of the disease of the mind d. State A has abolished insanity as a defense e. State A has adopted an insanity test that is cognitive

e. State A has adopted an insanity test that is cognitive The M'Naghten test is considered cognitive because it requires proof "that, at the time of the committing of the act, the party accused was labouring under such a defect [of] reason, from disease of the mind, as not to know the nature and quality of the act he was doing; or, if he did know it, that he did not know what he was doing what was wrong" (see § 19.02). A is incorrect because M'Naghten leaves off the volitional aspect, which is found in the irresistible impulse test. C is incorrect because this is not the Durham or product test, a test coming from New Hampshire. B is incorrect because M'Naghten leaves off the volitional aspect. A test that has both the volitional and cognitive would be the test used in a Model Penal Code jurisdiction. D is incorrect because there clearly is an insanity test being used here.

Morgan is charged with stealing equipment from the neighborhood salon. The charges are based on the testimony of Taylor who claims to have seen Morgan put the equipment in her backpack. Prior to trial, the prosecutor learns certain information that would be favorable to the defense. Which pieces of favorable information is the prosecutor constitutionally required to disclose to the defense under the United States Supreme Court ruling in Brady v. Marlyand? a. The barbershop security camera footage on the day of the theft shows Morgan leaving the salon two hours before the times were stolen b. Although Taylor is 100% sure Morgan is the person who stole the equipment, another salon patron who witnessed tht theft reports the thief was 6 inches taller and 30 pounds heavier than Morgan c. In exchange for testifying against Morgan, the government has agreed to dismiss Taylor's pending drug charge, which carries a penalty of 10 years in prison. d. If the value of the stolen equipment is over $500, Morgan faces a felony charge and three years in prison. If the value of the stolen equipment, however, is under $500, Morgan only faces 90 days in jail. Although Morgan is charged with a felony, shortly before trial, the salon owner disclosed to the government that the stolen equipment was very old and worth no more than $400. e. The prosecutor must disclose all of the above

e. The prosecutor must disclose all of the above Under Brady v. Maryland, the Supreme Court ruled that it would be unfair and unjust for a defendant to have to defend against criminal charges without access to exculpatory, impeachment, and mitigation information possessed by the government. Due process requires the prosecution turn over evidence that is favorable to the defense upon request if the evidence relates to the defendant's guilt or innocence or to sentencing.

According to the Model Penal Code, when the law provides that negligence is required to establish an element of offense, such element can be established if a person acts with which of the following mens rea? a. Only when a person acts negligently b. Only when the person acts recklessly or negligently c. Only when the person acts purposely or knowingly d. Only when the person acts purposely, knowingly, or recklessly e. When the person acts purposely, knowingly, recklessly, or negligently

e. When the person acts purposely, knowingly, recklessly, or negligently Negligently is the lowest level of mens rea, so including all of the levels above it are correct according to Model Penal Code § 2.02, which permits one to substitute a higher level of mens rea to prove lower levels of mens rea (see § 5.02). A is incorrect because the word "only" is used, and other higher levels of mens rea can be used. B is incorrect because recklessly or negligently are not the only higher levels of mens rea that could be used to prove negligence. C is incorrect because purposely and knowingly are not the only levels of mens rea that can be substitutes for negligently. D is incorrect because it omits negligently which can be used to prove the accused acted negligently.


Set pelajaran terkait

Med Surge II: Exam 5 Study guide

View Set

Pharmacology Chapter 52:Skin Disorder Topical Drugs

View Set

w3schools CSS quiz, W3Schools HTML Quiz

View Set

Chapter 4- Elimination, immunity

View Set

1.3 Given a scenario, perform server hardware maintenance.

View Set